Exam 1

¡Supera tus tareas y exámenes ahora con Quizwiz!

In which situations does a nurse have a duty to intervene and report? Select all that apply. a. A peer is unable to write behavioral outcomes. b. A health care provider consults the Physicians Desk Reference. c. A peer tries to provide patient care in an alcohol-impaired state. d. A team member has violated the boundaries of a vulnerable patient. e. A patient refuses a medication prescribed by a licensed health care provider

ANS: C, D Both instances jeopardize patient safety. The nurse must practice within the Code of Ethics for Nurses. A peer being unable to write behavioral outcomes is a concern but can be informally resolved. A health care provider consulting the Physicians Desk Reference is acceptable practice.

A patient begins a new program to assist with building social skills. In which part of the plan of care should a nurse record the item Encourage patient to attend one psychoeducational group daily? a. Assessment b. Analysis c. Planning d. Implementation e. Evaluation

ANS: D Interventions are the nursing prescriptions to achieve the outcomes. Interventions should be specific.

A charge nurse is conducting a class on therapeutic communication with a group of newly licensed nurses. Which of the following aspects of communication should the nurse identify as a component of verbal communication? a. Personal space b. Posture c. Eye contact d. Intonation

ANS: D The nurse should identify intonation as a component of verbal communication. Intonation is the tone of one's voice and can communicate a variety of feelings.Personal space is a component of nonverbal communication. Posture is a component of nonverbal communication. Eye contact is a component of nonverbal communication. (ATI ch 3)

A nurse in an outpatient mental health clinic is preparing to conduct an initial client interview. When conducting the interview, which of the following actions should the nurse identify as priority? a. Coordinate care with social services. b. Identify the client's perception of their mental health status. c. Include the client's family in the interview. d. Teach the client about their current mental health disorder.

ANS: B Assessment is the priority action when using the nursing process approach to client care. Identifying the client's perception of their mental health status provides important information about the client's psychosocial history. Options A, C, and D are appropriate actions however they are not the priority. (from ATI ch 1)

A 4 year old child is unable to consider another child's idea about playing house. This situation is an example of which concept of Piaget's theory of cognitive development? 1. Object permanence. 2. Reversibility and spatiality 3. Ego-centrism 4. Formal operations

ANS: 3 According to Piaget, egocentrism occurs during the state of preoperational thought 2-6 y/o. Personal experiences are though to be universal, and the child is unable to accept the differing viewpoints of others.

The nurse states to the client, "You say that you are sad, but you are smiling and laughing." Which describes the purpose of this therapeutic communication technique? 1. To provide suggestions for coping strategies 2. To redirect the client to an idea of importance 3. To bring incongruences or inconsistencies into awareness 4. To provide feedback to the client

ANS: 3 The nurse uses the therapeutic technique of 'confronting' to bring incongruences or inconsistencies into awareness

An 88 y/o client on an in-patient psychiatric unit states, "My children are refusing to visit me. I feel like giving up." This client has a deficient in which of Erikson's stages of development? 1. Initiative vs guilt 2. Industry vs inferiority 3. Identify vs role confusion 4. Integrity vs despair

ANS: 4 During integrity vs despair, late adulthood (65+), an individual reviews lief accomplishments, deals with loss, and prepares for death. A negative outcome of this state is despair and fear of death.

Social Security Disability Insurance (SSDI) benefits typically cover needs based on which one of the following? A. Disability B. Economic need C. Unemployment D. Mental illness

ANS: A (from ch 5 ppt, no rationale provided)

A patient says, "I've done a lot of cheating and manipulating in my relationships." Select a nonjudgmental response by the nurse. a. How do you feel about that? b. Its good that you realize this. c. Thats not a good way to behave. d. Have you outgrown that type of behavior?

ANS: A Asking a patient to reflect on feelings about his or her actions does not imply any judgment about those actions, and it encourages the patient to explore feelings and values. The remaining options offer negative judgments.

Which one of the following most correctly defines a psychiatrist? A. Medical physician B. Psychologist C. Mental health counselor D. Psychiatric practitioner

ANS: A Psychiatrists are state licensed medical doctors who have at least 4 years of additional training in diagnosing and treating psychiatric disorders. Medication prescribing and monitoring is a dominant treatment method used by psychiatrists. (from ch 5 ppt)

A nurse assesses a confused older adult. The nurse experiences sadness and reflects, "The patient is like one of my grandparents . . . so helpless." What feelings does the nurse describe? a. Transference b. Countertransference c. Catastrophic reaction d. Defensive coping reaction

ANS: B Countertransference is the nurses transference or response to a patient that is based on the nurses unconscious needs, conflicts, problems, or view of the world.

A mature, professional couple plans a large wedding in a city 100 miles from their home. Which response is most likely to be associated with this experience? a. Distress b. Eustress c. Acute stress d. Depersonalization

ANS: B Eustress is beneficial stress that will help the couple to focus, problem solve, and successfully plan their wedding (chapter 10, pg 120)

A basic level registered nurse works with patients in a community setting. Which groups should this nurse expect to lead? Select all that apply. a. Symptom management b. Medication education c. Family therapy d. Psychotherapy e. Self-care

ANS: A, B, E Symptom management, medication education, and self-care groups represent psychoeducation, a service provided by the basic level registered nurse. Advanced practice registered nurses provide family therapy and psychotherapy.

An African-American patient says to a Caucasian nurse, "Theres no sense talking. You wouldn't understand because you live in a white world." The nurses best action would be to: a. explain, "Yes, I do understand. Everyone goes through the same experiences." b. say, "Please give an example of something you think I wouldn't understand." c. reassure the patient that nurses interact with people from all cultures. d. change the subject to one that is less emotionally disturbing.ANS:

ANS: B Having the patient speak in specifics rather than globally helps the nurse understand the patients perspective.

A nurse assesses an adult experiencing a crisis. An appropriate question for the nurse to ask to determine situational support is: a. Has anything upsetting occurred in the past few days? b. Who can be helpful to you during this time? c. How does this problem affect your life? d. What led you to seek help at this time?

ANS: B Only the correct answer focuses on situational support. The incorrect options focus on the patients perception of the precipitating event.

A nurse working on an acute mental health unit is caring for a client who has posttraumatic stress disorder. Which of the following findings should the nurse expect? Select all the apply. A. Difficulty concentrating on tasks B. Obsessive need to talk about the event C. Negative self-image D. Recurring nightmares E. Diminished reflexes

ANS: A, C, D Manifestations of PTSD include the inability to concentrate on or complete tasks, include feeling guilty and having a negative self-image, and include recurring nightmares of flashbacks.A client who has PTSD is reluctant to talk about the traumatic even that triggered the disorder, and does not have an obsessive need to talk about the event. A client with PTSD has an increased started reflex and hypervigilance not diminished reflexes. (ATI ch 12)

A patient says, "My disability check will arrive next week, and I plan to give half of it to Praise Be Ministries." The nurse knows this action will result in the patient being unable to buy groceries by the end of the month. Select the nurse's best response. a. "It's your legal right to donate your money according to what you think is important." b. "Let's discuss how much you could donate and still be able to buy your groceries." c. "Why do you want to donate so much money to that organization?" d. "Let's talk to the social worker about appointing a guardian to manage your money."

ANS: B A response including "Why?" is not therapeutic. Responses A and D are not wrong but B is a better response. (question is from the legal issues ppt)

A nurse plans a group meeting for adult patients in a therapeutic milieu. Which topic should the nurse include? a. Coping with grief and loss b. The importance of handwashing c. Strategies for money management d. Staffing shortages expected over the next 3 days

ANS: B A therapeutic milieu provides a healthy social structure within an inpatient setting or structured outpatient clinic. Groups aim to help increase patients' self-esteem, decrease social isolation, encourage appropriate social behaviors, and educate patients in basic living skills, such as good hand washing (chapter 3, pg 31)

A depressed client discussing marital problems with the nurse says, "What will I do if my husband asks me for a divorce?" Which response by the nurse would be an example of therapeutic communication? 1. Why do you think that your husband will ask you for a divorce? 2. You seem to be worrying over nothing. I'm sure everything will be fine. 3. What has happened to make you think that your husband will ask for a divorce? 4. Talking about this will only make you more anxious and increase your depression

ANS 3 The therapeutic technique of 'exploring' along with reflective listening, draws out the client and can help the client feel valued, understood and supported. Exploring also gives the nurse the necessary assessment information to intervene appropriately.

Andrew, a hospice nurse for 5 years and a member of your nursing team, is demonstrating a blunted affect and is not completing patient care documentation in the required time frame. As a peer, what is your best action? A. Avoid mentioning these observations because you are only a peer. B. Ask Andrew what he feels is causing him to fall behind in his work. C. Immediately report your concerns to the nurse manager in charge of your team. D. Take Andrew to lunch and keep the conversation light and humorous.

ANS: B A. A practicing nurse has a legal and ethical responsibility to communicate concerns with peers. *B. Developing a trusting relationship with peers and being honest allows for openness and promotes beneficial dialog. C. At this point, there is nothing to report to the manager because you have just noticed a change in affect and a slower pace than usual. D. Taking Andrew to lunch may be helpful, but honest conversation is what is needed (from ch 10 ppt).

According to Maslow's hierarchy of needs, which of the following clients actions would be considered most basic? 1. A client discusses the need for order in his life and freedom from fear. 2. A client feels lonely and is seeking to share life experiences 3. A client begins to realize his full potential. 4. A client is role-playing a situation with a nurse to practice assertiveness skills

ANS: 1 #1 Discussing order in one's life and freedom from fear relates to Maslow's need hierarchy of safety and security, which is the 2nd most basic need after the client has bet physiological needs

A client exhibits a pattern of terminating relationships with significant others and poor self esteem. Using Sullivan's interpersonal theory, what major developmental state is this client struggling to master? 1. Late adolescence 2. Early adolescence 3. Preadolescence 4. Juvenile

ANS: 1 #1 During the stage of late adolescence, the major developmental task is to establish self-identity; experience satisfying relationships; and work to develop a lasting, intimate, opposite-sex relationship #2 During the state of early adolescence, developmental task is to learn to form satisfactory relationships with individuals of opposite sex and to develop sense of identity #3. During preadolescence , developmental task is to learn to form satisfactory relationships with individuals of the same sex and initiate feelings of affection for another individual #4. During juvenile state, developmental task is to form satisfactory peer relationships.

The nurse states to a client on an in-patient unit, "Tell me what's been on your mind." Which describes the purpose of this therapeutic communication technique? 1. To have the client choose the topic of conversation. 2. To present new ideas for consideration. 3. To convey interest in what the client is saying 4. To provide time for the nurse and client to gather thoughts and reflect

ANS: 1 A 'broad opening' helps the client to choose the topic of the conversation and puts the client in control of the content

Which is a nursing intervention that would promote the development of trust in the nurse-client relationship? 1. Simply and clearly providing reasons for policies and procedures. 2. Calling the client by name and title such as "Mr. Hawkins" 3. Striving to understand the motivations behind the clients behavior. 4. Taking the client's ideas into consideration when planning care.

ANS: 1 By being given simple and clear reasons for policies and procedures, the client can count on consistency from the nurse in the implementation of these policies and procedures. This consistency promotes the development of trust in the nurse-client relationship.

Which initial information gathered by the nurse is most important when assessing Erikson's stages of development? 1. The chronological age of the individual. 2. The developmental age exhibited through behaviors. 3. The time frame needed to complete a successful outcome at a previous stage 4. The implementation of interventions based on developmental age

ANS: 1 Erikson based his psychosocial theory on an individuals chronological age. Although individuals may have some unresolved issues from previous stages, the individual is assessed in a stage based on chronological age.

Which patient statement would lead a nurse to suspect that the developmental task of infancy was not successfully completed? a. I have very warm and close friendships. b. Im afraid to let anyone really get to know me. c. I am always right and confident about my decisions. d. Im ashamed that I didnt do it correctly in the first place.

ANS: B According to Erikson, the developmental task of infancy is the development of trust. The patients statement that he or she is afraid of becoming acquainted with others clearly shows a lack of ability to trust other people. Having warm and close friendships suggests the developmental task of infancy was successfully completed. Believing one is always right suggests rigidity rather than mistrust. Feelings of shame suggest failure to resolve the crisis of Initiative versus Guilt.

Counselors have been sent to a location that has experienced a natural disaster to assist the population to deal with the devastation. This is an example of __________________ prevention. 1. Primary 2. Secondary 3. Tertiary

ANS: 1 Primary prevention reduces the incidence of mental disorders, such as PTSD, within the population by helping individuals to cope more effectively with stress early in the grieving process. Primary prevention is extremely important for individuals who experience any traumatic event such as rape, war, hurricane, tornado, mass shootings.

A student nurses asks the instructor about the Diagnostic and Statistical Manual of Mental Disorders (DSM-5). Which of the following instructor statements are correct? Select all that apply. 1. The DSM-5 lists all psychiatric and general medical diagnoses 2. The DSM-5 allows clinicians to rate disorders along a continuum of severity 3. Conditions that do not meet the DSM-5 criteria are termed 'not elsewhere defined' (NED) 4. Dimensional assessment tools are included in the DSM-5 5. Global Assessment Functioning (GAF) is included in the DSM-5.

ANS: 1, 2, 3, 4 1. The DSM-5 lists all psychiatric and general medical diagnoses 2. DSM-5 allows clinicians to rate disorders along a continuum of severity 3. Conditions that do not meet the DSM-5 are termed 'not elsewhere defined (NED) 4. Dimensional assessment tools are included in DSM-5 #5 GAF was included in DSM-4 multi-axial system. This system is no longer included in the DSM-5.

Which of the following behaviors exemplifies the concept of countertransference? Select all that apply 1. The nurse defends the client's inappropriate behavior to the psychiatrist. 2. The nurse empathizes with the client's loss 3. The nurse subjectively appreciates the clients feelings 4. The nurse is uneasy when interacting with the client 5. The nurse recognizes that the client is emotionally attached to the social worker.

ANS: 1, 3, 4 #1. Defending the client's inappropriate behavior reflects an underlying subjective connection with the client. #2 The expression of empathy toward a client's loss is therpauetic and does not reflect the concept of countertransference #3. Appreciation of the client's feelings must be from an objective, not subjective, point of view. This subjective appreciation reflects the concept of countertransference #4 This uneasiness that the nurse experiences reflects an underlying subjective connection with the client, which is an example of countertransference #3. Emotional attachment by the client toward a health-care team member is an example of transference

Which historical nursing leader helped focus practice to recognize the importance of science in psychiatric nursing? a. Abraham Maslow b. Hildegard Peplau c. Kris Martinsen d. Harriet Bailey

ANS: B Although all these leaders included science as an important component of practice, Hildegard Peplau most influenced its development in psychiatric nursing. Maslow was not a nurse, but his theories influence how nurses prioritize problems and care. Bailey wrote a textbook in the 1930s on psychiatric nursing interventions. Kris Martinsen emphasized the importance of caring in nursing practice.

As part of the stress response, the HPA axis is stimulated. Which structures make up this system? a. Hippocampus, parietal lobe, and amygdala b. Hypothalamus, pituitary gland, and adrenal glands c. Hind brain, pyramidal nervous system, and anterior cerebrum d. Hepatic artery, parasympathetic nervous system, and acoustic nerve

ANS: B As part of the physiologic response of stress, the hypothalamus stimulates the HPA axis, which is made up of the hypothalamus, pituitary gland, and adrenal glands.

Which of the following therapeutic communication exchanges are examples of 'reflection'. Select all that apply. 1. Client: I get sad because I know i'm going to fail in school. Nurse: So you start feeling down every time a new semester begins? 2. Client: I forgot to get my prescription refilled. Nurse: It is important for you to take your medication as prescribed. 3. Client: I hate my recent weight gain. Nurse: Have you considered Overeaters Anonymous? 4. Client: I'm happy that i poisoned my husband. Nurse: You're happy to have poisoned your husband? 5. Client: I really don't know why I'm here. My stupid husband is behind this. Nurse: You seem angry at your husband for bringing you to the hospital"

ANS 1 & 5 Reflection is used when directing back what the nurse understands in regard to the client's ideas, feelings, questions and content. Reflection is used to put the client's feelings in the context of when or where they occur.

A depressed client discussing employment problems with the nurse says, "What will I do if my boss fires me?" Which nursing response would be an example of therapeutic communication? 1. "You seem to be worrying over nothing. I'm sure everything will be fine." 2. "What has happened to make you think that your boss will fire you?" 3. "Talking about this will only increase your depression." 4. "Why do you think that your boss will fire you?"

ANS 2 #1 Incorrect. This is an example of "giving false reassurance" by indicating to the client that there is no cause for fear or anxiety. This impedes any further interaction and expression of feelings by the client and is a nontherapeutic block to communication. #2 Correct. The therapeutic technique of "exploring," along with reflective listening, draws out the client and can help the client feel valued, understood, and supported. "Exploring" also gives the nurse necessary assessment information to intervene appropriately. #3 Incorrect. This is an example of "rejection," which shows contempt for the client's need to voice and express fears and anxiety. This is a nontherapeutic block to communication. #4 Incorrect. This is an example of "requesting an explanation," which requests the client to provide the reasons for his or her thoughts, feelings, and behaviors, which can be an unrealistic expectation. It also may put the client on the defensive. This is a nontherapeutic block to communication.

A client states, "I don't know what the pills are for or why I am taking them, so i Don't want them." Which is an example of the therapeutic communication technique of "giving information"? 1. You must take your medication to get better. 2. The doctor wouldn't prescribe these pills if they were harmful. 3. Do you feel this way about all your medications? 4. This medication will help improve you mood

ANS 4 #1:This is an example of 'giving advice', which is non-therapeutic because the statement does not allow the client to make personal decisions. #2. This is an example of 'defending', which is non-therapeutic because this statement would put the client on the defensive #3 This is an example of 'exploring', which is incorrect because the client has provided you with information by stating, 'i don't know what the pills are for' #4 This nurse is 'giving information' about the therapeutic effect of the client's medication because the nurse has assessed from the clients statement that information is needed

Nursing theorists identify the nurse-client relationship as central to nursing practice. After studying these concepts, the nursing student understands that Peplau is to the phases of the nurse-client relationship as Watson is to: 1. Seven assumptions about the science of caring 2. Cultural care diversity and universality 3. Modeling and role modeling 4. Human energy fields

ANS: 1 # 1 The basis of Waton's theory is the believe that curing disease is the domain of medicine, whereas caring is the domain of nursing. She development seven assumptions about the science of caring, which allows the nurse to deliver integrated holistic care #2 The bases if Leininger's, not Watson's, theory of cultural care diversity and universality is the belief that across cultures there are healthcare practices and beliefs that are diverse and similar. The nurse must understand the clients culture to provide care. #3. Erickson, Tomlin, and Swain, not Watson, developed theories that included modeling and role modeling by the use of interpersonal interactive skills.

A client on an inpatient unit states, "Did they tell you that I killed my cat?" The nurse maintains an open posture, maintains eye contact, and leans toward the client. Which describes the purpose of the nurse's action? 1. To provide time for the nurse and client to gather thoughts and reflect 2. To present new ideas for consideration 3. To take note and expand on a single topic of conversation 4. To have the client choose a topic of conversation

ANS: 1 #1 Correct. The therapeutic communication technique being used here is "silence." The purpose of silence is to provide time for the nurse and client to gather thoughts and reflect. Also, if the topic of conversation is controversial or disturbing, the nurse buys time to avoid a spontaneous negative reaction. #2 Incorrect. Presenting new ideas for consideration is the purpose of the therapeutic communication technique of "suggesting," not "silence." #3 Incorrect. To assist the client to take note of a single idea or even a single word is the purpose of the therapeutic communication technique of "focusing," not "silence." #4 Incorrect. The therapeutic communication technique of a "broad opening," not "silence," is used to help the client initiate the conversation and puts the client in control of the content.

Which an an example of an appropriate psychosexual development? 1. An 18 month-old relieves anxiety by the use of a pacifier. 2. A 5 year old boy focuses on relationships with other boys. 3. A 7 year old girl identifies with her mother. 4. A 12 month of beings learning about Independence and control.

ANS: 1 #1 from birth to 18 months of age, a child is in the oral stage of Freud's psychosexual development. During this state, an infant would attempt to decrease anxiety by finding relief using oral gratification. #2. From age 3-6 years a child is in the phallic stage. During this stage a child is looking to identify with the parent of the same sex and developing her own sexual identity by focusing on genital organs. Focusing on relationships with same-sex peers occur ins during the latency stage, 6-12 years. #3 From 6-12 a child is in latency. During this stage, a child is suppressing sexuality and focusing on relationships with same-sex peers. Identifying with the parent of the same sex occurs in the phallic stage, 3-6. #4 Learning about independence and control is anal phase, 18 months to 3 years old.

Indicating that there is no cause for anxiety is to 'reassuring' as sanctioning or denouncing the client's idea or behaviors is to: 1. Approving/disapproving 2. Rejecting 3. Interpreting 4. Probing

ANS: 1 Approving/disapproving is a nontherapeutic block to communication. It implies that the nurse as the right to pass judgement on whether the client's ideas or behaviors are good or bad. Example: That's good. i'm glad that you...' or 'that's bad. I'd rather you wouldn't...'

The staff observes a new nurse expression anger and distrust while treating a client with a long history of alcohol use disorder. The staff suspects that the nurse is experiencing countertransference. Which statement by the new nurse validates the staff's suspicions? 1. My mother misused alcohol and neglected her family. 2. The client said i had the same disposition as his cranky wife 3. Maybe the client and i can sit down and work out a plan 4. The client refuses to accept responsibility for his alcohol misuse

ANS: 1 In this example, countertranference refers to the nurse's behavioral and emotional response to the client's diagnosis of alcohol use disorder. those feelings may be related to unresolved feelings toward significant others from the nurse's past, or they may be generated in response to transference feelings on the part of the client. The nurse's statements revealing that her mother misused alcohol and neglected the family is evidence of countertransference. Intervention: Have evaluative sessions with the nurse after an encounter with the client, in which the nurse and staff members discuss and compare the exhibited behaviors in the nurse-client relationship. The relationship usually should not be terminated in the face of countertransference.

A client on an in-patient psychiatric unit is exhibiting extreme agitation. Using a behavioral approach, which nursing intervention should be implemented? 1. The nurse should role-play stressful situations to promote adaptive coping 2. The nurse should develop a plan to deal with stressors during a family meeting 3. The nurse should give ordered prn medications to decrease agitation 4. The nurse should discuss emotional triggers that precipitate angry outbursts

ANS: 1 Role playing a stressful situation to assist the client coping would be a behavioral intervention. Discussing emotional triggers that precipitate angry outbursts is an example of an intrapersonal, not behavioral intervention. Developing a plan for dealing with stressors during a family meeting is interpersonal.

A client on an in-patient psychiatric unit has impulse control issues and at times can be irritable and hostile with little regard for others. Using psychoanalytic theory, which describes this behavior? 1. The behavior is id driven 2. The behavior is ego driven 3. The behavior is superego driven 4. The behavior is edo-ideal driven

ANS: 1 The id is the locus of instinctual drives, or the 'pleasure principle'. The client is exhibiting id-driven behaviors that are impulsive and egocentric, and may be irrational.

On an in-patient psychiatric unit, the nurse explores feelings about potentially working with a women who has allowed her husband to abuse her and her children physically and verbally. This interaction would occur in which phase of the nurse-client relationship. 1. Pre-interaction phase 2. Orientation phase 3. Working phase 4. Termination phase

ANS: 1 The pre-interaction phase involves preparation for the first encounter with the client, such as reading previous medical records and exploring feelings regarding working with that particular client. In this example, the nurse obtains information about the client for initial assessment. This also allows the nurse to become aware of an personal biases about the client. Goad; Explore self perception.

A client diagnosed with PTSD is close to discharge. Which client statement would indicate that teaching about the psychosocial cause of PTSD was effective? 1. My experience, how i deal with it, and my support system all affect my disease process. 2. I have learned to try and avoid stressful situations as a way to decrease my emotional pain 3. So, natural opioid released during the trauma caused my body to become 'addicted'. 4. Because of the trauma, I have negative perception of the world and feel hopeless.

ANS: 1 When the client verbalizes understanding of how the experience event, individual traits, and available support systems affect his diagnosis, the client demonstrates a good understanding of the psychosocial cause of PTSD #2 avoiding situations example of of a learned cause of PTSD #3. release of natural opioids during traumatic event is biological #4. Negative perception of world is cognitive cause of PTSD

A client states, "I don't know why I should go to group therapy, so I don't want to go." Which nursing statement is an example of the nontherapeutic communication block of "giving advice"? 1. "If I were you, I would attend groups in order to get better." 2. "The doctor wouldn't prescribe these treatments if they were harmful." 3. "Do you feel this way about all your therapies?" 4. "Let me tell you about how group therapy works."

ANS: 1 #1 Correct. Here the nurse is "giving advice," which is a nontherapeutic block to communication because the statement does not allow the client to make personal decisions. #2 Incorrect. This is an example of "defending," not "giving advice." This statement is a nontherapeutic block to communication because it would put the client on the defensive. #3Incorrect. This is an example of "exploring," but the nurse has changed the topic and ignored the client's concerns. This introduction of an unrelated topic is a nontherapeutic block to communication. #4 Incorrect. This is an example of "giving information," not "giving advice." Here, the nurse has assessed from the client's statement that information is needed. Also "giving information" is a therapeutic, not nontherapeutic, communication technique.

How is the DSM-5 hepful to mental health providers? Select all that apply 1. The DSM-5 provides a common language related to the diagnoses of mental illness 2. The DSM-5lists medications that are appropriate for the treatment of mental disorders 3. The DSM-5 presents standard criteria for the classification of mental disorders 4. The DSM-5 provides an axis system to evaluate clients holistically 5. The DSM-5 presents a comprehensive list of community resources

ANS: 1, 3 DSM-5 provides common language that facilitates a comprehensive and systematic client evaluation. Presents standard criteria for the classification of mental disorders. These criteria facilitate consistence of care.

A nurse is pulled from a med/surg floor to the psychiatric unit. Which of the following clients would the nurse manager assign to this nurse? Select all that apply. 1. A chronically depressed client 2. An actively psychotic client 3. A client experiencing paranoid thinking 4. A client diagnosed with cluster B personality disorder traits 5. A client diagnosed with generalized anxiety disorder.

ANS: 1, 5 #1 Because there is no indication that this client is suicidal, of the 5 clients presented, this client is most appropriate to assign #5. Clients diagnosed with generalized anxiety disorder are typically not in danger to themselves or others and tend not to be manipulative.

Which of the following clients retain the right to give informed consent? Select all that apply 1. A 21 y/o client who is hearing and seeing things that others do not. 2. A 32 y/o client who is diagnosed with severe intellectual development disorder 3. A 65 y/o client declared legally incompetent 4. A 14 y/o client with ADHD 5. An 80 y/o client who wants to participate in a medical research study.

ANS: 1, 5 A diagnosis of psychosis does not mean that a client is unable to consent to treatment. Advance age does not affect a client's ability to give informed consent.

According to Maslow's hierarchy of needs, which situation demonstrates the lowest level of attainment? 1. In individual demonstrates an ability to discuss objectively all points of view and possesses a strong sense of ethics 2. An individual avoids harm while maintaining comfort, order and physical safety 3. An individual establishes meaningful interpersonal relationships and can identity himself or herself within a group. 4. An individual desires prestige from personal accomplishments

ANS: 2 #1 Demonstrating an ability to discuss objectively all points of view and possessing a strong sense of ethics relates to Maslow's need hierarchy of self actualization, the 5th and highest level of attainment. This occurs ater the client has met physiological, safety and security, love and belonging, and self-esteem/esteem of others needs #2 Avoiding harm while maintaining comfort, order and physical safety relates to Maslow's need hierarchy of safety and security, which is the lowest level of attainment after the client has met physiological needs #3. Establishing meaningful interpersonal relationships and identifying oneself within a group relates to love and belonging, the 3rd level of attainment. This occurs after the client has met physiological, safety and security, and love and belonging needs. #4. Desiring prestige from personal accompaniments relates to self-esteem/esteem of others, the 4th level of attainment.

A pediatric resident physician whose neighbor has been admitted to the inpatient psychiatric unit comes to the unit and asks to see the client's chart. Which is the appropriate initial nursing action? 1. Provide the chart for the physician's review. 2. Refuse to provide the client's chart because the physician is not part of the client's treatment team. 3. Contact and notify the client's physician about the incident before letting the physician review the chart. 4. To keep the resident from seeing the client's chart, tell the resident that the nurse needs to transfer some orders from the chart.

ANS: 2 #1 Incorrect. This pediatric resident has no right to review this client's chart because the resident is not part of the client's treatment team. The resident also has a personal relationship with the client, and providing the chart would breach the client's right to confidentiality. #2 Correct. It is appropriate for the nurse to refuse to provide the client's chart because the physician is not part of the client's treatment team. #3 Incorrect. This pediatric resident has no right to review this client's chart. Providing the chart would breach the client's right to confidentiality. #4 Incorrect. This is a passive-aggressive, indirect way to refuse access to the client's chart. A direct approach is more appropriate.

A client has been voluntarily admitted to an inpatient psychiatric unit. After evaluation, the treatment team has determined that the client is a danger to self. The client states, "I now want to leave." Which is the priority nursing intervention? 1. Teach the client the benefits of psychiatric interventions. 2. Notify the treatment team of the client's intentions. 3. Initiate forced medication protocols, and administer antianxiety medications STAT. 4. Provide the client with forms related to leaving "against medical advice" (AMA).

ANS: 2 #1: Incorrect. Teaching the client about the benefits of psychiatric interventions would be inappropriate at this time and would not address the immediate client safety risk. #2: Correct. This is the priority intervention for the nurse. Because the treatment team believes this client is a danger to self, the physician can change the admission status by submitting appropriate court paperwork to hold a client against his or her will for a legally determined period. State law determines the amount of time a client can be held. #3 Incorrect. Only if the client is an immediate danger to self or others can forced medication be administered. #4 Incorrect. If the client is determined to be a danger to self, leaving "against medical advice" (AMA) is not an option, and therefore this intervention is not appropriate.

A nursing student observes an incorrect dosage of medication being given to a client receiving electroconvulsive therapy. To implement the ethical principle of veracity, which action would the nursing student take? 1. Keep the information confidential to avoid harm to others 2. Inform the student's instructor and the client's primary nurse and document the situation 3. Tell only the client about the incident because the decision about actions would be determined only by the client. 4. Because the client was not harmed, the incident would not need to be reported.

ANS: 2 By applying the ethical principle of veracity, the student should tell the truth and report and document the incident. The only limitation to the ethical principle of veracity is when telling the truth would knowingly produce harm. Veracity must be in the context of hospital policy and procedures and within the chain of command.

Which is the goal for the orientation phase of the nurse-client relationship? 1. Explore self-perceptions 2. Establish trust 3. Promote change 4. Evaluate goal attainment

ANS: 2 The establishment of trust is the goal of the orientation phase. During this phase, a contract is established with the client

Which determines the scope of practice for a registered nurse employed in a psychiatric in-patient facility? 1. National Alliance of the Mentally Ill (NAMI) 2. State law, which may vary from state to state 3. Federal law, which applies nationwide 4. National League of Nursing (NLN)

ANS: 2 The legal parameters of professional nursing are defined within each state by the states nurses practice act. NAMI is a group that advocates for clients experiencing mental illness. NAMI does not determine the scope of practice

On an in-patient psychiatric unit, a client states, "I want to learn better ways to handle my anger." This interaction is most likely to occur in which phase of the nurse-client relationship? 1. Pre-interaction phase 2. Orientation phase 3. Working phase 4. Termination phase

ANS: 2 The orientation phase involves creating an environment that establishes trust and rapport. Another task of this phase includes establishing a contract for interventions that details the expectations and responsibilities of the nurse and the client. In this example, the client has built the needed trust and rapport with the nurse. The client now feels comfortable and ready to acknowledge the problem and contract for the intervention. Goal: Establish trust and formulate contract for intervention.

During a recent counseling session with a depressed client, the psychiatric nurse observes signs of transference. Which statement by the client would indicate that the nurse is correct? 1. Thanks for taking my side against the staff. 2. You sure do remind me of my mom 3. Working on problem-solving together makes sense 4. I won't stop drinking just to please my whole family.

ANS: 2 This example of transference occurs when the client unconsciously displaces (or transfers) to the nurse feelings formed toward a person from his or her past. Transference also can take the form of overwhelming affection with unrealistic expectations of the nurse by the client. When the nurse does not meet the expectations, the client may become angry and hostile. Intervention: The nurse should work with the client in sorting out the past from the present, identifying the transference, and reassigning a more appropriate meaning to the nurse-client relationship.

During a recent counseling session with a depressed client, the psychiatric nurse observes signs of transference. Which statement by the client would indicate that the nurse is correct? 1. Thanks for taking my side against the staff. 2. You sure do remind me of my mom 3. Working on problem-solving together makes sense 4. I won't stop drinking just to please my whole family.

ANS: 2 This example of transference occurs when the client unconsciously displaces (or transfers) to the nurse feelings formed toward a person from his past. Transference also can take the form of overwhelming affection with unrealistic expectations of the nurse by the client. When the nurse does not meet the expectations, the client may become angry and hostile. Intervention: The nurse should work with the client in sorting out the past from the present, identifying the transference, and reassigning a more appropriate meaning to the nurse-client relationship.

A client experiencing agitation states, "I'm ready to jump out of my skin." Which nursing response would be an example of the therapeutic communication technique of restating? 1. "What do you mean by 'jumping out of your skin'?" 2. "You're ready to jump out of your skin?" 3. "I hate to hear you make such a comment." 4. "Why do you feel like you are going to jump out of your skin?"

ANS: 2 #1 Incorrect. By asking, "What do you mean by 'jumping out of your skin'?" the nurse is striving to understand that which is vague and incomprehensible and searching for mutual understanding. This is the therapeutic communication technique of clarification, not restating. #2 Correct. By asking, "You're ready to jump out of your skin?" the nurse is using the therapeutic technique of restating. This technique lets the client know that the nurse is engaged in the conversation and encourages the client to continue discussing the topic at hand. #3 Incorrect. By stating, "I hate to hear you make such a comment," the nurse is using the nontherapeutic block to communication of rejecting. By using this block the nurse refuses to consider the client's ideas and may be communicating contempt. #4 Incorrect. By asking, "Why do you feel like you are going to jump out of your skin?" the nurse is using the nontherapeutic communication block of requesting an explanation. By using this intimidating block to communication, the nurse asks the client to provide the reasons for thoughts, feelings, behaviors, and events. This implies that the client must defend his or her behaviors and feelings.

Which elements are included in the nurse-client contract? 1. During the pre-interaction phase, the roles are established. 2. During the orientation phase, the purpose of the interaction is established 3. During the working phase, the conditions for termination are established. 4. During the termination phase, the criteria for discharge are established.

ANS: 2 #1. Roles cannot be established in the pre-interaction phase b/c the nurse and the client have no met #2 During the orientation phase, the purpose of the interaction is established, and this is a component of the nurse-client contract. #3. The conditions for termination are established in the orientation, not the working phase #4. Criteria for discharge are not established in the termination phase. Discharge criteria are determined by the entire treatment team.

The client states, "I'm not sure that the doctor has prescribe the correct medication for my sadness." Which would be a therapeutic nursing response? 1. A lot of clients are nervous about new medications. I'll get you some information about it. 2. So you think that this medication is not right for you? 3. Why do you think that this medication won't help your mood? 4. Your doctor has been prescribing this medication for years, and it really does help people.

ANS: 2 By 'verbalizing the implication' that the client thinks the medication is not good for the clients problem, the nurse puts into words what the nurse thinks that the client is saying. If this implication is incorrect, it gives the client an opportunity to clarify the statement further.

Which is the goal of cognitive-behavioral theory model according to Beck and Ellis? 1. Developing satisfactory relationships, maturity, and relative freedom from anxiety 2. Substituting rational for irrational beliefs and eliminating self-defeating behaviors 3. Facing reality and developing standards for behaving responsibility 4. Reducing bodily tensions and managing stress by biofeedback and relaxation training

ANS: 2 Substituting rational beliefs for irrational beliefs and eliminating self defeating behaviors are goals of cognitive-behavioral theory

Which of the following are examples of the therapeutic communication technique of 'clarification'? Select all that apply. 1. Can we talk more about how you feel about your father? 2. I'm not sure what you mean when you use the word 'fragile' 3. I notice that you seem angry today. 4. How does your mood today compare with yesterday? 5. Can you help me understand what you mean by a 'difficult childhood'?

ANS: 2 & 5 This example of 'clarification' is an attempt by the nurse to check the nurses's understanding of what has been said by the client and helps the client to make his or her thoughts or feelings more explicit

Which of the following are example of non-therapeutic communication blocks? 1. You acted out in group. It made the other clients uncomfortable. 2. Why did you refuse your medication this afternoon? 3. I'm sorry you feel that way. It is a feeling typical of hospitalized clients. 4. You just think that you are not getting better. You'll see. Everything will work out. 5. What I am hearing you say is that everyone is out to get you.

ANS: 2, 3, 4 #2. This is an example of the non therapeutic block of communication of 'requesting an explanation.' If involves asking the client to provide reasons for thoughts, feelings, behaviors, and events. Asking why a client did something or feels a certain way can be intimidating and implies that the client must defend his or her behavior or feelings. #3. This is an example of the non therapeutic block to communication of 'belittling'. This minimizes the clients concerns and causes the client to feel insignificant or important. When one is experiencing discomfort, it is no relief to hear that others are or have been in similar situations. #4. This is an example of the non therapeutic block to communication of 'giving reassurance'. Reassurance by the nurse indicates to the client that there is no cause for anxiety. By devaluing the clients feelings, the client may be discouraged from further expression of feelings.

Which of the following are examples of therapeutic communication techniques? 1. Tell me about your drunk-driving record. 2. How does this compare with the time you were sober? 3. That's good. I'm glad that you think you can stop drinking. 4. I think we need to talk more about your previous coping mechanisms. 5. What led up to your taking that first drink after 5 years sober.

ANS: 2, 4, 5 #2. This is an example of the therapeutic technique of 'encouraging comparisons,' which ask that similarities and differences be noted #4. This is an example of the therapeutic technique of 'focusing', which poses a statement that helps the client expand on a topic of importance #5. This is an example of 'placing the event in time or sequence,' which clarifies the relationship of events in time so that the nurse and client can view them in perspective.

A nursing student states to the instructor, "I am afraid of mentally ill clients. They are all violent." Which of the following statements would the instructor use to clarify this perception for the student? Select all that apply. 1. Even though most mentally ill clients are often violent, there are ways to de-escalate these behaviors 2. A very few clients with mental illness exhibit violent behaviors. 3. There are medications that can be given to clients to prevent violent behaviors 4. Only paranoid clients exhibit violent behaviors 5. There is little difference in violence statistics between clients diagnosed with mental illness and the general population

ANS: 2, 5 It is true that a very few clients with mental illness exhibit violent behaviors. There is little difference in the violence statistics between clients diagnosed with mental illness and general population.

A 25 year old client diagnosed with major depressive disorder remains in his room and avoids others. According to Erkison, what describes this client's developmental task assessment? 1. Stagnation 2. Despair 3. Isolation 4. Role confusion

ANS: 3 #1 Stagnation is the negative outcome of 'adulthood' stage of development, ages 30-65 years. The major developmental task for adulthood is to achieve the life goals established for oneself, which considering the welfare of future generations. #2. Despair is the negative outcome of 'old age' stage of development, ego integrity vs despair. Age 65 until death. This major developmental task is to review one's life and derive meaning from positive and negative events. Through this process, ones needs to achieve a positive sense of self worth. #3 Isolation is the negative outcome of 'young adulthood' stage of development, intimacy vs isolation. This stage ranges from 20-30. The major developmental task for young adulthood is to form an intense, lasting relationship or a commitment to another person, cause, institution or creative effort. #4 Role confusion is the negative outcome of 'adolescence'. Identity vs role confusion. 12 to 20 years. Major developmental task for this stage is to integrate the tasks mastered in the previous stages into a secure sense of self.

A 7 year old boy is active in sports and has received a most improved player award at his basement tournament. According to Erikson, which descries this client's developmental task assessment? 1. Autonomy 2. Identity 3. Industry 4. Initiative

ANS: 3 #1. Autonomy is the positive outcome of 'early childhood'. Autonomy vs shame and doubt. 18 months to 3 years old. Major developmental task for early childhood is to gain some self-control and independence within the environment. #2. Identity is the positive outcome of 'adolescence'. Idenity vs role confusion. 12 to 20 years old. Task for adolescence is to develop a sense of confidence, emotional stability and a view of oneself as a unique individual. #3. Industry is the positive outcome of 'school age', 6 to 12 years old. Industry vs inferiority. Major developmental task for school age si to achieve a sense of self confidence by learning, competing, performing successfully, and receiving recognition from significant others, peers and acquaintances. #4. Initiative is the positive outcome for 'late childhood'. Initiative vs guilt'. 3 to 6 years. Developmental task for late childhood is to develop a sense of purpose and the ability to initiate and direct one's own activities.

The statement, "Growth involves resolution of critical tasks through the eight stages of the life cycle," is a concept of which therapeutic model? 1. Interpersonal. 2. Cognitive-behavioral. 3. Intrapersonal. 4. Psychoanalytic.

ANS: 3 #1: Interpersonal theories assume that development occurs in stages related to experiencing different types of relationships #2. Cognitive-behavior theories assume that individuals have the potential for rational and irrational thinking, which alters behaviors. 3. Erikson's developmental theory assumes that intrapersonal growth involves resolution of critical tasks throughout eight stages in the life cycle. Eriksons's theory is classified as an intrapersonal theory. #4. Psychoanalytic theories assume that individuals are motivated by unconscious desires and conflicts

A client has been placed in seclusion because the client has been deemed a danger to others. Which is the priority nursing intervention for this client? 1. Have little contact with the client to decrease stimulation 2. Provide the client with privacy to maintain confidentiality 3. Maintain contact and assure the client that seclusion will maintain the clients safety. 4. Teach the client relaxation techniques and effective coping strategies to deal with anger.

ANS: 3 It is important to maintain contact with the client and assure the client that seclusion is a way to maintain the client's safety. Seclusion, when appropriate, should be implemented in a matter of fact manner, focusing on the client's behavior and the consequences of the behavior. When the client is in seclusion, he is not in a readiness state to learn. When hostility and threatening behavior are under control,, #4 would be appropriate intervention.

The nurse helps a client practice various techniques of assertive communication and gives positive feedback for improvement of passive-aggressive interactions. This intervention would occur in which phase of the nurse-client relationship. 1. Pre-interaction phase 2. Orientation phase 3. Working phase 4. Termination phase

ANS: 3 The working phase includes promoting the client's insight and perception of reality, problem-solving, overcoming resistant behaviors, and continuously evaluating progress toward goal attainment. In this example, the client works toward better communication and is guided and encouraged with positive feedback from the nurse. Goal: promote client change.

While talking about an abusive childhood, a client addicted to heroin suddenly blurts out, "I hate my doctor." Which client statement would indicate that transference is taking place? 1. The doctor has told me that his son recovered, and I will also. 2. I don't care what anyone says, I don't have a problem I can't handle. 3. I'd bet my doctor beat and locked his son in a closet when he was a boy. 4. I'm going to stop fighting and start working together with my doctor.

ANS: 3 This is an example of tranference, which occurs when the client unconsciously displaces (or transfers) to the physician feelings formed toward a person from his past. Transference also can take the form of overwhelming affection with unrealistic expectations of the physician by the client. By accusing the doctor of abusing his son, the client is transferring his feelings of hate from the client's father to the doctor. Intervention: The physician should work with the client to sort out the past from the present, identify the transference, and reassign a most appropriate meaning to the physician client relationship.

The nurse is interviewing a client admitted to an in-patient psychiatric unit with a diagnosis of depressive disorder. Which is the primary goal in the assessment phase of the nursing process for this client? 1. To build trust and rapport 2. To identify goals and outcomes 3. To collect and organize information 4. To identify and validate the medical diagnosis

ANS: 3 The primary goal in the assessment phase of the INT is to collect and organize data, which would be used to identify and prioritize the client's problems.

Which client situation is an example of normal ego development? 1. A client calls out in pain to get his needs met. 2. A client complains of poor self-esteem because of punishments from his past. 3. A client exhibits the ability to assert himself without anger or aggression. 4. A client feels guilty about wanting to have sexual relations outside of marriage.

ANS: 3 The ego is considered the 'reality principle' and beings to develop between 4 and 6 months of age. The ego experiences the outside world and then adapts and responds to it. The ego's main goal is to maintain harmony between the id and superego. The ability to assert oneself without anger or aggression is an example of a healthy ego development.

Which of the following statements are examples of the therapeutic communication technique of 'focusing'. Select all that apply 1. You say you're angry, but I notice that you're smiling. 2. Are you saying that you want to drive to Hawaii? 3. Tell me again about Vietnam and your feelings after you were wounded 4. I see you staring out the window. Tell me what you're thinking 5. Yesterday you described your relationship with your mom. Let's continue that topic.

ANS: 3 & 5 This is an example of the therapeutic communication technique of 'focusing'. The nurse uses focusing to direct the conversation to a particular topic of importance or relevance to the client

Which of the following physician situations reflect the ethical principle of veracity? Select all that apply. 1. The physician decides not to tell the client about the stage 4 cancer diagnosis because it would take away hope. 2. The physician volunteers at a free clinic to provide care for the indigent. 3. The physician provides and explains to the client the positive results of the client's HIV test. 4. The physician refuses to give out confidential information over the telephone. 5. The physician reviews the risks, benefits, and interactions of the client's newly prescribed medication.

ANS: 3, 5 Answer 1 is incorrect because in this scenario, the physician has withheld information. Because veracity refers to the duty to always be truthful and not mislead, this is not reflective of this ethical principle. Answer 2 is incorrect because this situation is reflective of beneficence, not veracity. Beneficence is the duty to benefit or promote the good of others. Answer 3 is correct because the physician is not misleading the client and is providing truthful data; the physician is implementing the ethical principle of veracity. Answer 4 is incorrect because this situation is reflective of nonmaleficence—the duty to do no harm, such as divulging a professionally held confidence—not veracity. Answer 5 is correct because the physician is not misleading the client and is providing truthful data; the physician is implementing the ethical principle of veracity.

Which of the following are examples of the non therapeutic block to communication of 'giving reassurance'. Select all that apply. 1. That's good. I'm glad that you... 2. Hang in there, every dog has his day. 3. Don't worry, everything will work out. 4. I think you should... 5. I'm sure you can beat this addiction.

ANS: 3, 5 This is an example of the non therapeutic block to communication of 'giving reassurance.' The use of this block indicates to the client that there is no cause for anxiety. This block involves giving the client a false sense of confidence and devaluing the clients feelings. If also may discourage the client from further expression of feelings if the client believes those feelings would only be downplayed or ridiculed.

Which of the following clients does not have the ability to refuse medications or treatments? Select all that apply. 1. An involuntarily committed client 2. A voluntarily committed client 3. A client who has been deemed incompetent by the court. 4. A client who has a diagnosis of antisocial personality disorder 5. A client who is an imminent danger to himself.

ANS: 3, 5 When a client is declared incompetent, the client has a mental disorder resulting in a defect in judgement, and this defect makes the client incapable of handling personal affairs. A guardian is appointed. The guardian makes decisions for the client, and the client loses the right to refuse medications. When a client is declared an imminent danger to self other others, he loses the ability to refuse medications or treatments. Involuntary commitment results in substantial restriction of the rights of the individual, but protection against loss of liberty and due process is retrained. Involuntarily committed clients can refuse medications, unless they are in imminent danger to themselves or others.

A client on an inpatient psychiatric unit has been raised in an extremely religious and judgmental family. The client exhibits uncontrolled guilt and poor self-esteem. Applying psychoanalytic theory, which is true? 1. The client is experiencing guilt as a result of inherited tendencies from parents. 2. The client is having developmental issues associated with industry versus inferiority. 3. The client had an inconsistent mother figure during separation-individuation. 4. The client is experiencing poor self-worth as a result of an overdeveloped superego.

ANS: 4 #1 Incorrect. Inherited tendencies are not part of psychoanalytic theory. Biological theory describes the etiology of mental illness based on genetic predisposition. #2 Incorrect. Because age is not presented in the question, it is impossible to determine the client's stage of development according to Erikson. The age range for the developmental conflict of industry versus inferiority is 6 to 12 years. #3 Incorrect. An inconsistent mother figure during separation-individuation is part of Mahler's theory of object relations, not psychoanalytic theory. Inconsistency during subphase 3 of separation-individuation, called rapprochement, can cause self-esteem and abandonment issues. #4 Correct. According to psychoanalytic theory, struggles with uncontrolled guilt and poor self-esteem arise from the overdevelopment of the superego. The superego, which develops between the ages of 3 and 6, internalizes the values and morals set forth by primary caregivers. When these caregivers are rigid and judgmental, overdevelopment can occur, leading to uncontrolled guilt and poor self-esteem.

Which therapeutic communication technique is an example of "making an observation"? 1. "You say you're happy, but I notice that you're frowning." 2. "Are you saying that you want to be seen in the outpatient clinic?" 3. "Tell me again about your divorce and your feelings during that time." 4. "I see you are wringing your hands. Tell me what you're thinking."

ANS: 4 #1 Incorrect. This is an example of "confrontation," not "making an observation." The nurse uses a therapeutic technique of "confrontation" to bring incongruence or inconsistencies into awareness. #2 Incorrect. This is an example of "clarification," not "making an observation." The nurse uses the therapeutic technique of "clarification" to check the understanding of what has been said by the client and helps the client make his or her thoughts or feelings more explicit. #3 Incorrect. This is an example of "focusing," not "making an observation." The nurse uses the therapeutic technique of "focusing" to direct the conversation on a particular topic of importance or relevance to the client. #4 Correct. When the nurse notices what the client is doing, in this case "wringing the hands," the nurse is using the therapeutic communication technique of "making an observation."

Which therapeutic communication exchange is an example of "restating"? 1. Client: "I get depressed because I know I'm going to lose the game."Nurse: "So, you start feeling sad every time you play a game?" 2. Client: "I forgot to attend group therapy this morning."Nurse: "Attending all therapies may reduce your symptoms." 3. Client: "I hate my passive nature."Nurse: "Have you considered assertiveness training?" 4. Client: "I'm happy that I divorced my wife."Nurse: "You were happy to have divorced your wife?"

ANS: 4 #1 Incorrect. This is an example of "reflection," not "restating." This technique is used when directing back what the nurse understands in regard to the client's ideas, feelings, questions, and content. "Reflection" is used to put the client's feelings in the context of when and where they occur. #2 Incorrect. When the nurse gives valuable information to the client, the nurse is using the therapeutic technique of "informing," not "restating." #3 Incorrect. Providing suggestions for coping strategies is a way that the nurse assists the client to consider alternative options. This is the therapeutic technique of "suggesting," not "restating." #4 Correct. This is an example of "restating." Restating is a therapeutic technique that lets the client know that the nurse is listening and wants to understand what the client is saying. When restating, the nurse uses the client's exact words or variations of these words.

Which assessment is most important when evaluating signs and symptoms of mental illness? 1. The decreased amount of creativity a client exhibits. 2. The inability to face problems within one's life 3. The intensity of an emotional reaction 4. The client's social and cultural norms

ANS: 4 It is important when assessing for mental illness that social and cultural norms be evaluated. The context of cultural norms determines if behaviors are considered acceptable or aberrant. Belief in reincarnation can be acceptable in one culture and considered 'delusional' in another

On an in-patient psychiatric unit, the goals of therapy have been met, but the client cries and states, "I have to keep coming back to therapy to handle my anger better." This interaction occurs in which phase of the nurse-client relationship. 1. Pre-interaction phase 2. Orientation phase 3. Working phase 4. Termination phase

ANS: 4 The termination phase occurs when progress has been made toward attainment of mutually set goals, a plan for continuing care is mutually established and feelings about termination are recognized and explored. In this example, the nurse must establish the reality of separation and resist repeated delays by the client because of dependency needs. Goat: Evaluate goal attainment and ensure therapeutic closure.

A client states to the nurse, "I'm thinking about ending it all." Which response be the nurse would be an example of therapeutic communication? 1. Your attitude will hamper your recovery. 2. Wasn't your wife just here during visiting hours? 3. Why would you want to do something like that? 4. You must be feeling very sad right now.

ANS: 4 This is the therapeutic technique of "attemping to translate words into feelings," by which the nurse tries to find clues to the underlying true feelings and at the same time validates the client's statement. The nurse might then explore and delve more deeply by responding: Can you tell me more about this sadness you feel?

A client states, "They're putting rat poison in my food." Which intervention would assist this client to be medication adherent while on the in-patient psychiatric unit? 1. Remind the client that the psychiatrist ordered the medication for him. 2. Maintain the same routine for medication administration 3. Use liquid medication to avoid cheeking 4. Keep medications in sealed packages and open them in front of the client

ANS: 4 When a client is exhibiting paranoia, it is important for the nurse to take further actions to encourage adherence. Presenting the client with medicating that is labeled and sealed shows that no one has tampered with the medication and my assist with medication adherence.

In which situation does a health-care worker have the duty to warn a potential victim? 1. When clients manipulate and split the staff and are a danger to self. 2. When clients curse at family members during visiting hours 3. When client exhibit paranoid delusions and auditory or visual hallucinations 4. When clients make specific threats toward someone who is identifiable.

ANS: 4 When a client makes specific threats toward someone who is identifiable, is it the duty of the health-care worker to warn the potential victim. The nurse should bring this information to the treatment team and document the report

A client is developing a sense of identity and learning to form relationships with persons of the opposite gender. According to Sullivan's theory, this client would be assessed at what stage of development? 1. Childhood (18 months- 6) 2. Juvenile (6-9) 3. Preadolescence (9-12) 4. Early adolescence (12-14)

ANS: 4 #1 Childhood stage, an individual learns to experience a delay in personal gratification without undue anxiety #2. Juvenile stage, individual learns to form satisfactory peer relationships #3. 'Preadolsecense' individual learns to form satisfactory relationships with persons of the same gender, initiating feelings of affection for another person. #4 During Sullivan's 'early adolescence' stage, an individual develops a sense of identity and learns to form relationships with persons of the opposite gender.

A client is admitted for alcohol detoxification states, "I don't think my drinking has anything to do with why I am here in the hospital. I think I have problems with depression." Which statement by the nurse is the most therapeutic response? 1. I think you really need to look at the amount you are drinking and consider the effect on your family. 2. That's wrong. I disagree with that. Your admission is because of your alcohol abuse and not for any other reason. 3. I'm sure you don't mean that. You have to realize that alcohol is the root of your problems. 4. I find it hard to believe that alcohol is not the problem because you have recently lost your job and your drivers license.

ANS: 4 When using the therapeutic communication technique of 'voicing doubt'', the nurse expresses uncertainty as to the reality of what is being communicated.

Which is an example of an interpersonal intervention for a client on an in-patient psychiatric unit? 1. Assist the client to note common defense mechanisms and coping skills that are being used. 2. Discuss 'acting-out' behaviors and assist the client in understanding why they occur. 3. Ask the client to record thoughts he or she is having before acting-out behaviors occur. 4. Ask the client to acknowledge one positive person in his life to assist the client after discharge.

ANS: 4 interpersonal theory states that individual behavior and personality development are the direct reslt of interpersonal relationships. The identification of a positive relationship is an intervention that reflects interpersonal theory.

Number the following nursing interventions as they would proceed through the phases of the nurse-client relationship? 1. Plan for continued care 2. Promote client's insight 3. Examine personal biases 4. Formulate nursing diagnoses

ANS: 4, 3, 1, 2 First, in the pre-orientation phase of the nurse-client relationship, the nurse would examine any personal biases. Second, in the orientation phase, the nurse would formulate nursing diagnostic statements. Third, in the working phase, the nurse would attempt to promote client insight. Fourth, in the termination phase, the nurse would plan for continued client care.

A patient who immigrated to the United States from Honduras was diagnosed with schizophrenia. The patient took an antipsychotic medication for 3 weeks but showed no improvement. Which resource should the treatment team consult for information on more effective medications for this patient? a. Clinical algorithm b. Clinical pathway c. Clinical practice guideline d. International Statistical Classification of Diseases and Related Health Problems (ICD)

ANS: A A *clinical algorithm* is a guideline that describes diagnostic and/or treatment approaches drawn from large databases of information. These guidelines help the treatment team make decisions cognizant of an individual patients needs, such as ethnic origin, age, or gender. A clinical pathway is a map of interventions and treatments related to a specific disorder. Clinical practice guidelines summarize best practices about specific health problems. The ICD classifies diseases.

A patient says, "Please don't share information about me with the other people." How should the nurse respond? a. I won't share information with others without your permission, but I will share information about you with other staff members. b. A therapeutic relationship is just between the nurse and the patient. It's up to you to tell others what you want them to know. c. It really depends on what you choose to tell me. I will be glad to disclose at the end of each session what I will report to others. d. I cannot tell anyone about you. It will be as though I am talking about my own problems, and we can help each other by keeping it between us.

ANS: A A patient has the right to know with whom the nurse will share information and that confidentiality will be protected. Although the relationship is primarily between the nurse and patient, other staff members need to know pertinent data. The other incorrect responses promote incomplete disclosure on the part of the patient, require daily renegotiation of an issue that should be resolved as the nurse-patient contract is established, and suggest mutual problem solving. The relationship must be patient centered.

Which action by a psychiatric nurse best supports a patients right to be treated with dignity and respect? a. Consistently addressing a patient by title and surname. b. Strongly encouraging a patient to participate in the unit milieu c. Discussing a patients condition with another health care provider in the elevator. d. Informing a treatment team that a patient is too drowsy to participate in care planning.

ANS: A A simple way of showing respect is to address the patient by title and surname rather than assuming that the patient would wish to be called by his or her first name. Discussing a patients condition with a health care provider in the elevator violates confidentiality. Informing a treatment team that the patient is too drowsy to participate in care planning violates patient autonomy. Encouraging a patient to participate in the unit milieu exemplifies beneficence and fidelity.

A nurse assesses that a patient is suspicious and frequently manipulates others. Using the Freudian theory, these traits are related to which psychosexual stage? a. Oral b. Anal c. Phallic d. Genital

ANS: A According to Freud, each of the behaviors mentioned develops as the result of attitudes formed during the oral stage, when an infant first learns to relate to the environment. Anal stage traits include stinginess, stubbornness, orderliness, or their opposites. Phallic stage traits include flirtatiousness, pride, vanity, difficulty with authority figures, and difficulties with sexual identity. Genital stage traits include the ability to form satisfying sexual and emotional relationships with members of the opposite sex, emancipation from parents, and a strong sense of personal identity.

A community mental health nurse has worked with a patient for 3 years but is moving out of the city and terminates the relationship. A new nurse who begins work with this patient will: a. begin at the orientation phase. b. resume the working relationship. c. enter into a social relationship. d. return to the emotional catharsis phase.

ANS: A After the termination of a long-term relationship, the patient and new nurse usually have to begin at ground zero, the orientation phase, to build a new relationship. If termination is successfully completed, then the orientation phase sometimes progresses quickly to the working phase. Other times, even after successful termination, the orientation phase may be prolonged.

A bill introduced in Congress would reduce funding for the care of people diagnosed with mental illnesses. A group of nurses write letters to their elected representatives in opposition to the legislation. Which role have the nurses fulfilled? a. Advocacy b. Attending c. Recovery d. Evidence-based practice

ANS: A An advocate defends or asserts anothers cause, particularly when the other person lacks the ability to do that for himself or herself. Examples of individual advocacy include helping patients understand their rights or make decisions. On a community scale, advocacy includes political activity, public speaking, and publication in the interest of improving the individuals with mental illness; the letter-writing campaign advocates for that cause on behalf of patients who are unable to articulate their own needs.

Which scenario best demonstrates empathetic caring? a. A nurse provides comfort to a colleague after an error of medication administration. b. A nurse works a fourth extra shift in 1 week to maintain adequate unit staffing. c. A nurse identifies a violation of confidentiality and makes a report to an agency's privacy officer. d. A nurse conscientiously reads current literature to stay aware of new evidence-based practices.

ANS: A Caring is evidenced by empathic understanding, actions, and patience on another's behalf; actions, words, and presence that lead to happiness and touch the heart; and giving of self while preserving the importance of self. Comforting is a part of caring, which includes social, emotional, physical, and spiritual support (chapter 1, pg 6)

A nurse is teaching a client about stress-reduction techniques. Which of the following client statements indicates understanding of the teaching? a. "Cognitive reframing will help me change my irrational thoughts to something positive." b. "Progressive muscle relaxation uses a mechanical device to help me gain control over my pulse rate." c. "Biofeedback causes my body to release endorphins so that I feel less stress and anxiety." d. "Mindfulness allows me to prioritize the stressors that I have in my life so that I have less anxiety."

ANS: A Cognitive reframing helps the client look at irrational cognitions (thoughts) in a more realistic light and to restructure those thoughts in a more positive way.Biofeedback, rather than progressive muscle training, uses a mechanical device to promote voluntary control over autonomic functions.. Physical exercise, rather than biofeedback, causes a release of endorphins that lower anxiety and reduce stress. Priority restructuring, rather than mindfulness, teaches the client to prioritize differently to reduce the number of stressors. (ATI ch 9)

A Mexican-American patient puts a picture of the Virgin Mary on the bedside table. Under which section of the assessment should the nurse document this behavior? a. Culture b. Ethnicity c. Verbal communication d. Nonverbal communication

ANS: A Cultural heritage is expressed through language, works of art, music, dance, ethnic clothing, customs, traditions, diet, and expressions of spirituality. This patients prominent placement of the picture is an example of expression of cultural heritage. Verbal and nonverbal communications apply to all areas of an assessment.

A nurse says, "When I was in school, I learned to call upset patients by name to get their attention; however, I read a descriptive research study that says that this approach does not work. I plan to stop calling patients by name." Which statement is the best appraisal of this nurses comment? a. One descriptive research study rarely provides enough evidence to change practice. b. Staff nurses apply new research findings only with the help from clinical nurse specialists. c. New research findings should be incorporated into clinical algorithms before using them in practice. d. The nurse misinterpreted the results of the study. Classic tenets of practice do not change.

ANS: A Descriptive research findings provide evidence for practice but must be viewed in relation to other studies before practice changes. One study is not enough. Descriptive studies are low on the hierarchy of evidence. Clinical algorithms use flow charts to manage problems and do not specify one response to a clinical problem. Classic tenets of practice should change as research findings provide evidence for change.

A nursing assistant says to the nurse, "The schizophrenic in room 226 has been rambling all day." When considering the nurse's responsibility to manage the ancillary staff, which response should the nurse provide? a. "It is more respectful to refer to the patient by name than by diagnosis." b. "Thank you for informing me about that. I will document the behavior." c. "It is not unusual for schizophrenics to do that. It's just part of their illness." d. "You have a difficult job. I'm glad you are so accepting of our patients' behaviors."

ANS: A Diagnoses classify disorders that people have, not the person. For this reason, it is important to avoid use of expressions such as "a schizophrenic" or "an alcoholic." The nurse has a responsibility to educate the coworker (chapter 2, pg 14)

A nurse is planning care for the termination phase of a nurse-client relationship. Which of the following actions should the nurse include in the plan of care? a. Discussing ways to use new behaviors b. Practicing new problem-solving skills c. Developing goals d. Establishing boundaries

ANS: A Discussing ways for the client to incorporate new healthy behaviors into life is an appropriate task for the termination phase. Practicing new problem-solving skills is an appropriate task for the working phase. Developing goals is an appropriate task for the orientation phase. Establishing boundaries is an appropriate task for the orientation phase. (ATI ch 5)

The assumption most useful to a nurse planning crisis intervention for any patient is that the patient: a. is experiencing a state of disequilibrium. b. is experiencing a type of mental illness. c. poses a threat of violence to others. d. has a high potential for self-injury.

ANS: A Disequilibrium is the only answer universally true for all patients in crisis. A crisis represents a struggle for equilibrium when problems seem unsolvable. Crisis does not reflect mental illness. The potential for self-violence or other-directed violence may or may not be a factor in crisis.

A nurse is preparing an educational seminar on stress for other nursing staff. Which of the following information should the nurse include in the discussion? a. Excessive stressors cause the client to experience distress. b. The body's initial adaptive response to stress is denial. c. Absence of stressors results in homeostasis. d. Negative, rather than positive, stressors produce a biological response.

ANS: A Distress is the result of excessive or damaging stressors, such as anxiety or anger.Denial is part of the grief process. The body's initial adaptive response to stress is known as the fight‑or‑flight mechanism. Individuals need the presence of some stressors to provide interest and purpose to life. Both positive and negative stressors produce a biological response in the body. (ATI ch 9)

Which statement shows a nurse has empathy for a patient who made a suicide attempt? a. You must have been very upset when you tried to hurt yourself. b. It makes me sad to see you going through such a difficult experience. c. If you tell me what is troubling you, I can help you solve your problems. d. Suicide is a drastic solution to a problem that may not be such a serious matter.

ANS: A Empathy permits the nurse to see an event from the patients perspective, understand the patients feelings, and communicate this to the patient. The incorrect responses are nurse centered (focusing on the nurses feelings rather than the patients), belittling, and sympathetic.

While talking with a patient with severe depression, a nurse notices the patient is unable to maintain eye contact. The patients chin lowers to the chest while the patient looks at the floor. Which aspect of communication has the nurse assessed? a. Nonverbal communication b. A message filter c. A cultural barrier d. Social skills

ANS: A Eye contact and body movements are considered nonverbal communication. Insufficient data are available to determine the level of the patients social skills or whether a cultural barrier exists.

A day shift nurse contacts a nurse scheduled or night shift at home and says, "Our unit is full and there are eight patients in the emergency department waiting for a bed." The night shift nurse replies, "Thanks for telling me. I am calling in sick." Which type of problem is evident by the night shift nurse's reply? a. Ethical problem of fidelity b. Legal problem of negligence c. Legal problem of an intentional tort d. Violation of the patients' right to treatment

ANS: A Fidelity is an ethical principle that involves maintaining loyalty and commitment to patients (chapter 6, page 62)

A patient is having difficulty making a decision. The nurse has mixed feelings about whether to provide advice. Which principle usually applies? Giving advice: a. is rarely helpful. b. fosters independence. c. lifts the burden of personal decision making. d. helps the patient develop feelings of personal adequacy.

ANS: A Giving advice fosters dependence on the nurse and interferes with the patients right to make personal decisions. Giving advice also robs patients of the opportunity to weigh alternatives and to develop problem-solving skills. Furthermore, it contributes to patient feelings of personal inadequacy. It also keeps the nurse in control and feeling powerful.

Planning for patients with mental illness is facilitated by understanding that inpatient hospitalization is generally reserved for patients who: a. present a clear danger to self or others. b. are noncompliant with medications at home. c. have no support systems in the community. d. develop new symptoms during the course of an illness.

ANS: A Hospitalization is justified when the patient is a danger to self or others, has dangerously decompensated, or needs intensive medical treatment. The incorrect options do not necessarily describe patients who require inpatient treatment.

A patient with major depressive disorder has lost 20 pounds in one month has chronic low self-esteem and a plan for suicide. The patient has taken an antidepressant medication for 1 week. Which nursing intervention is most directly related to this outcome: Patient will refrain from gestures and attempts to harm self? a. Implement suicide precautions. b. Frequently offer high-calorie snacks and fluids. c. Assist the patient to identify three personal strengths. d. Observe patient for therapeutic effects of antidepressant medication.

ANS: A Implementing suicide precautions is the only option related to patient safety. The other options, related to nutrition, self-esteem, and medication therapy, are important but are not priorities.

A patient tells the nurse at the clinic, "I haven't been taking my antidepressant medication as directed. I leave out the midday dose. I have lunch with friends and don't want them to ask me about the pills." Select the nurses most appropriate intervention. a. Investigate the possibility of once-daily dosing of the antidepressant. b. Suggest to the patient to take the medication when no one is watching. c. Explain how taking each dose of medication on time relates to health maintenance. d. Add the following nursing diagnosis to the plan of care: Ineffective therapeutic regimen management, related to lack of knowledge.

ANS: A Investigating the possibility of once-daily dosing of the antidepressant has the highest potential for helping the patient achieve compliance. Many antidepressants can be administered by once-daily dosing, a plan that increases compliance. Explaining how taking each dose of medication on time relates to health maintenance is reasonable but would not achieve the goal; it does not address the issue of stigma. The self-conscious patient would not be comfortable doing this. A better nursing diagnosis would be related to social stigma. The question asks for an intervention, not analysis.

A patient being treated in an alcohol rehabilitation unit reveals to the nurse, I feel terrible guilt for sexually abusing my 6-year-old child before I was admitted. Based on state and federal law, the best action for the nurse to take is to: a. anonymously report the abuse by telephone to the local child abuse hotline. b. reply, Im glad you feel comfortable talking to me about it c. respect the nurse-patient relationship of confidentiality. d. file a written report on the agency letterhead.

ANS: A Laws regarding reporting child abuse discovered by a professional during a suspected abusers alcohol or drug treatment differ by state. Federal law supersedes state law and prohibits disclosure without a court order except in instances in which the report can be made anonymously or without identifying the abuser as a patient in an alcohol or drug treatment facility. Anonymously reporting the abuse by telephone to the local child abuse hotline meets federal criteria. Respecting nurse-patient confidentiality and replying Im glad you feel comfortable talking to me about it do not accomplish reporting. Filing a written report on agency letterhead violates federal law.

A nurse assesses an inpatient psychiatric unit, noting that exits are free from obstruction, no one is smoking, the janitors closet is locked, and all sharp objects are being used under staff supervision. These observations relate to: a. management of milieu safety. b. coordinating care of patients. c. management of the interpersonal climate. d. use of therapeutic intervention strategies.

ANS: A Members of the nursing staff are responsible for all aspects of milieu management. The observations mentioned in this question directly relate to the safety of the unit. The other options, although part of the nurses concerns, are unrelated to the observations cited.

As Election Day nears, a mental health nurse studies the position statements of various candidates for federal offices. Which candidate's commentary would the nurse interpret as supportive of services or persons diagnosed with mental illness? a. "Full parity insurance coverage for mental illness" b. "Coverage for biologically based mental illnesses" c. "Reimbursement for initial treatment of addictions" d. "Managed care oversight for mental illness services"

ANS: A Mental health parity refers to third-party (insurance) coverage of care for mental illness and addictions similarly to care of physical illness. Federal and state legislation apply, but coverage varies by state. Some states offer full parity or mental illness insurance coverage. (chapter 5, Page 58)

An advanced practice nurse determines a group of patients would benefit from therapy in which peers and interdisciplinary staff all have a voice in determining the level of the patients privileges. The nurse would arrange for: a. milieu therapy b. cognitive therapy c. short-term dynamic therapy d. systematic desensitization

ANS: A Milieu therapy is based on the idea that all members of the environment contribute to the planning and functioning of the setting. The other therapies are all individual therapies that do not fit the description.

Several nurses are concerned that agency policies related to restraint and seclusion are inadequate. Which statement about the relationship of substandard institutional policies and individual nursing practice should guide nursing practice? a. The policies do not absolve an individual nurse of the responsibility to practice according to the professional standards of nursing care. b. Agency policies are the legal standard by which a professional nurse must act and therefore override other standards of care. c. In an institution with substandard policies, the nurse has a responsibility to inform the supervisor and leave the premises. d. Interpretation of policies by the judicial system is rendered on an individual basis and therefore cannot be predicted.

ANS: A Nurses are professionally bound to uphold the American Nurses Association (ANA) standards of practice, regardless of lesser standards established by a health care agency or state. Conversely, if the agency standards are higher than the ANA standards of practice, the agency standards must be upheld. The courts may seek to establish the standard of care through the use of expert witnesses when the issue is clouded.

Three weeks after being assaulted by a patient, a nurse develops headaches, insomnia, and gastrointestinal problems. The nurse has four absences from work over a 2-week period. Which action should the nursing supervisor employ? a. Refer the nurse or counseling and support. b. Ask the nurse about current personal problems. c. Direct the nurse to take paid vacation for the following week. d. Schedule the nurse or administrative tasks rather than patient care.

ANS: A Nurses need to monitor their thoughts and feelings and learn to recognize when they need self-care, support, or professional help. This is especially true in the aftermath of violence. Nurses often suppress their own feelings in order to effectively handle the immediate situation and react later with anxiety (chapter 20, pg 327)

A nurse is communicating with a client who was admitted for treatment of a substance use disorder. Which of the following communication techniques should the nurse identify as a barrier to therapeutic communication? a. Offering advice b. Reflecting c. Listening attentively d. Giving information

ANS: A Offering advice to a client is a barrier to therapeutic communication that the nurse should avoid using. Advice tends to interfere with the client's ability to make personal decisions and choices.The technique of reflection, directs the focus back to the client in order for the client to examine his feelings. The skill of active listening is an important therapeutic technique to help the nurse hear and understand the information and messages the client is trying to convey. Giving information informs the client of needed information to assist in the treatment planning process. (ATI ch 3)

A nurse explains to the family of a patient who is mentally ill how the nurse-patient relationship differs from social relationships. Which is the best explanation? a. The focus is on the patient. Problems are discussed by the nurse and patient, but solutions are implemented by the patient. b. The focus shifts from nurse to patient as the relationship develops. Advice is given by both, and solutions are implemented. c. The focus of the relationship is socialization. Mutual needs are met, and feelings are openly shared. d. The focus is the creation of a partnership in which each member is concerned with the growth and satisfaction of the other.

ANS: A Only the correct response describes the elements of a therapeutic relationship. The remaining responses describe events that occur in social or intimate relationships.

A patient diagnosed with mental illness asks a psychiatric technician, Whats the matter with me? The technician replies, Your wing nuts need tightening. The nurse who overheard the exchange should take action based on: a. violation of the patients right to be treated with dignity and respect. b. the nurses obligation to report caregiver negligence. c. preventing defamation of the patients character. d. supervisory liability.

ANS: A Patients have the right to be treated with dignity and respect. Patients should never be made the butt of jokes about their illness. Patient emotional abuse has been demonstrated, not negligence. The technicians response was not clearly defamation. Patient abuse, not supervisory liability, is the issue.

A nurse at the mental health center prepares to administer a scheduled injection of haloperidol decanoate (Haldol depot) to a patient with schizophrenia. As the nurse swabs the site, the patient shouts, Stop! I dont want to take that medicine anymore. I hate the side effects. Select the nurses best initial action. a. Stop the medication administration procedure and say to the patient, Tell me more about the side effects youve been having. b. Say to the patient, Since Ive already drawn the medication in the syringe, Im required to give it, but lets talk to the doctor about skipping next months dose. c. Proceed with the injection but explain to the patient that other medications are available that may help reduce the unpleasant side effects. d. Notify other staff members to report to the room for a show of force and proceed with the injection, using restraint if necessary.

ANS: A Patients with mental illness retain their civil rights unless clear, cogent, and convincing evidence of dangerousness exists. The patient in this situation presents no evidence of being dangerous. The nurse, an as advocate and educator, should seek more information about the patients decision and should not force the medication.

Which technique will best communicate to a patient that the nurse is interested in listening? a. Restate a feeling or thought the patient has expressed. b. Ask a direct question, such as, "Did you feel angry?" c. Make a judgment about the patients problem. d. Say, "I understand what you're saying."

ANS: A Restating allows the patient to validate the nurses understanding of what has been communicated. Restating is an active listening technique. Judgments should be suspended in a nurse-patient relationship. Closed-ended questions such as "Did you feel angry?" ask for specific information rather than show understanding. When the nurse simply states that he or she understands the patients words, the patient has no way of measuring the understanding.

While conducting the initial interview with a patient in crisis, the nurse should: a. speak in short, concise sentences. b. convey a sense of urgency to the patient. c. be forthright about time limits of the interview. d. let the patient know that the nurse is in control.

ANS: A Severe anxiety narrows perceptions and concentration. By speaking in short, concise sentences, the nurse enables the patient to grasp what is being said. Conveying urgency will increase the patients anxiety. Letting the patient know who is in control and stating that time is limited are nontherapeutic strategies.

A soldier in a combat zone tells the nurse, "I saw a child get blown up over a year ago, and now I keep seeing bits of flesh everywhere. I see something red and the visions race back to my mind." Which phenomenon associated with post-traumatic stress disorder (PTSD) is this soldier describing? a. Re-experiencing b. Hyperarousal c. Avoidance d. Psychosis

ANS: A Spontaneous or cued recurrent, involuntary, and intrusive distressing memories of the traumatic events are often associated with PTSD. The soldier has described intrusive thoughts and visions associated with re-experiencing the traumatic event. This description does not indicate psychosis, hypervigilance, or avoidance.

A victim of spousal violence comes to the crisis center seeking help. The nurse uses crisis intervention strategies that focus on: a. supporting emotional security and reestablishing equilibrium. b. offering long-term resolution of issues precipitating the crisis. c. promoting growth of the individual. d. providing legal assistance.

ANS: A Strategies of crisis intervention are directed toward the immediate cause of the crisis and are aimed at bolstering the emotional security and reestablishing equilibrium, rather than focusing on underlying issues and long-term resolutions. The goal is to return the individual to the pre- crisis level of function. Crisis intervention is, by definition, short term. Promoting growth is a focus of long-term therapy. Providing legal assistance might be applicable.

A student nurse tells the instructor, "I don't need to interact with my patients. I learn what I need to know by observation." The instructor can best interpret the nursing implications of Sullivan's theory to the student by responding: a. Nurses cannot be isolated. We must interact to provide patients with opportunities to practice interpersonal skills. b. Observing patient interactions can help you formulate priority nursing diagnoses and appropriate interventions. c. I wonder how accurate your assessment of the patients needs can be if you do not interact with the patient. d. Noting patient behavioral changes is important because these signify changes in personality.

ANS: A Sullivan believed that the nurses role includes educating patients and assisting them in developing effective interpersonal relationships. Mutuality, respect for the patient, unconditional acceptance, and empathy are cornerstones of Sullivans theory. The nurse who does not interact with the patient cannot demonstrate these cornerstones. Observations provide only objective data. Priority nursing diagnoses usually cannot be accurately established without subjective data from the patient. The third response pertains to Maslows theory. The fourth response pertains to behavioral theory.

17. A nurse talks with the caregiver of a combat veteran diagnosed with severe traumatic brain injuries. The caregiver says, "I don't know how much longer I can do it. My whole life is consumed with taking care of my partner." Select the nurses best response. a. How are you taking care of yourself? b. Let's review your partners diagnostic results. c. I have some web-based programs for you to visit. d. Your partner is lucky to have someone so devoted.

ANS: A The caregiver is the focus of the nurses attention. The caregiver is suffering. The nurse must be empathetic and assess how the caregiver is caring for self. Reassurance and isolated computer activities do not help. The partner is already aware of the diagnostic results.

A professor's 4-year-old child has a fever of 101.6 F, diarrhea, and complains of stomach pain. The professor is scheduled to teach three classes today. Which nursing diagnosis best applies to this scenario? a. Decisional conflict b. Unilateral neglect c. Disabled family coping d. Ineffective management of the therapeutic regimen

ANS: A The caregiver is the focus of the nurses attention. The professor is under stress, related to the conflict between his parenting and professional roles. This scenario presents a decisional conflict. No evidence suggests that family coping is disabled or that a therapeutic regimen is not managed. Unilateral neglect refers to the awareness of the body.

A 4-year-old child grabs toys from siblings, saying, "I want that toy now!" The siblings cry, and the child's parent becomes upset with the behavior. Using the Freudian theory, a nurse can interpret the child's behavior as a product of impulses originating in the: a. id. b. ego. c. superego. d. preconscious.

ANS: A The id operates on the pleasure principle, seeking immediate gratification of impulses. The ego acts as a mediator of behavior and weighs the consequences of the action, perhaps determining that taking the toy is not worth the parents wrath. The superego would oppose the impulsive behavior as not nice. The preconscious is a level of awareness

A client plans to have electroconvulsive therapy (ECT). Which member of the team is responsible for obtaining the client's informed consent? A. Physician B. Psychologist C. Case manager D. Registered nurse

ANS: A The informed consent of a client must be obtained by the physician before a treatment or procedure is performed. (question is from the legal issues ppt)

A nurse assesses soldiers in a combat zone in Afghanistan. When is it most important for the nurse to screen for signs and symptoms of traumatic brain injury (TBI)? a. After a fall, vehicle crash, or exposure to a blast b. Before departing Afghanistan to return to the United States c. One year after returning to the United States from Afghanistan d. Immediately upon return to the United States from Afghanistan

ANS: A The military estimates that up to 20% of the combat veterans in both Afghanistan and Iraq suffer some degree of TBI. TBI exhibits signs shortly after the injury, and these signs usually resolve in days or weeks. Screening after an exposure to an explosion and before returning to the United States is important.

When a female Mexican-American patient and a female nurse sit together, the patient often holds the nurses hand. The patient also links arms with the nurse when they walk. The nurse is uncomfortable with this behavior and thinks the patient is homosexual. Which alternative is a more accurate assessment? a. The patient is accustomed to touch during conversations, as are members of many Hispanic subcultures. b. The patient understands that touch makes the nurse uncomfortable and controls the relationship based on that factor. c. The patient is afraid of being alone. When touching the nurse, the patient is reassured and comforted. d. The nurse is homophobic.

ANS: A The most likely answer is that the patients behavior is culturally influenced. Hispanic women frequently touch women they consider to be their friends. Although the other options are possible, they are less likely.

A nurse uses Maslows hierarchy of needs to plan care for a psychotic patient. Which problem will receive priority? The patient: a. refuses to eat or bathe. b. reports feelings of alienation from family. c. is reluctant to participate in unit social activities. d. needs to be taught about medication action and side effects.

ANS: A The need for food and hygiene is physiological and therefore takes priority over psychological or meta-needs in care planning.

As a nurse discharges a patient, the patient gives the nurse a card of appreciation made in an arts and crafts group. What is the nurse's best action? a. Recognize the effectiveness of the relationship and patients thoughtfulness. Accept the card. b. Inform the patient that accepting gifts violates the policies of the facility. Decline the card. c. Acknowledge the patients transition through the termination phase but decline the card. d. Accept the card and invite the patient to return to participate in other arts and crafts groups.

ANS: A The nurse must consider the meaning, timing, and value of the gift. In this instance, the nurse should accept the patients expression of gratitude.

A patient is hospitalized for depression and suicidal ideation after their spouse asks for a divorce. Select the nurses most caring comment. a. Lets discuss some means of coping other than suicide when you have these feelings. b. I understand why youre so depressed. When I got divorced, I was devastated too. c. You should forget about your marriage and move on with your life. d. How did you get so depressed that hospitalization was necessary?

ANS: A The nurses communication should evidence caring and a commitment to work with the patient. This commitment lets the patient know the nurse will help. Probing and advice are not helpful or therapeutic interventions.

A 40-year-old adult living with parents states, "I'm happy but I don't socialize much. My work is routine. When new things come up, my boss explains them a few times to make sure I understand. At home, my parents make decisions for me, and I go along with them." A nurse should identify interventions to improve this patients: a. self-concept. b. overall happiness. c. appraisal of reality. d. control over behavior.

ANS: A The patient feels the need for multiple explanations of new tasks at work and, despite being 40 years of age, allows both parents to make all decisions. These behaviors indicate a poorly developed self-concept. Although the patient reports being happy, the subsequent comments refute that self-appraisal. The patients comments do not indicate that he/she is out of touch with reality. The patients needs are broader than control over own behavior.

A nurse surveys the medical records for violations of patients rights. Which finding signals a violation? a. No treatment plan is present in record. b. Patient belongings are searched at admission. c. Physical restraint is used to prevent harm to self. d. Patient is placed on one-to-one continuous observation.

ANS: A The patient has the right to have a treatment plan. Inspecting a patients belongings is a safety measure. Patients have the right to a safe environment, including the right to be protected against impulses to harm self that occur as a result of a mental disorder.

A patient diagnosed with major depressive disorder tells the community mental health nurse, "I usually spend all day watching television. I there's nothing good to watch, I just sleep or think about my problems." What is the nurse's best action? a. Refer the patient or counseling with a recreational therapist. b. Ask the patient, "What kinds o program do you like to watch?" c. Suggest to the patient, "Are there some friends you could call instead?" d. Advise the patient, "Watching television and thinking about problems makes depression worse."

ANS: A The patient's comments indicate problems with use of leisure time. Recreational activities improve emotional, physical, cognitive, and social well-being. A recreational therapist is the best member o the treatment team to provide these services. (chapter 5, Page 55 (Box 5-1))

When a nurse assesses an older adult patient, the patients answers seem vague or unrelated to the questions. The patient also leans forward and frowns, listening intently to the nurse. An appropriate question for the nurse to ask would be: a. Are you having difficulty hearing when I speak? b. How can I make this assessment interview easier for you? c. I notice you are frowning. Are you feeling annoyed with me? d. You're having trouble focusing on what I'm saying. What is distracting you?

ANS: A The patients behaviors may indicate difficulty hearing. Identifying any physical need the patient may have at the onset of the interview and making accommodations are important considerations. By asking if the patient is annoyed, the nurse is jumping to conclusions. Asking how to make the interview easier for the patient may not elicit a concrete answer. Asking about distractions is a way of asking about auditory hallucinations, which is not appropriate because the nurse has observed that the patient seems to be listening intently.

An adult has cared for a debilitated parent for 10 years. The parents condition recently declined, and the health care provider recommended placement in a skilled care facility. The adult says, "I've always been able to care for my parents. Nursing home placement goes against everything I believe." Successful resolution of this person's crisis will most closely relate to: a. resolving the feelings associated with the threat to the persons self-concept. b. maintaining the ability to identify situational supports in the community. c. relying on the assistance from role models within the person's culture. d. mobilizing automatic relief behaviors by the person.

ANS: A The patients crisis clearly relates to a loss of (or threatened change in) self-concept. Her capacity to care for her parents, regardless of the deteriorating condition, has been challenged. Crisis resolution involves coming to terms with the feelings associated with this loss. Identifying situational supports is relevant but less so than coming to terms with the threat to self-concept. Reliance on lessons from role models can be helpful but is not the primary factor associated with resolution in this case. Automatic relief behaviors are not helpful and are part of the fourth phase of crisis.

The family of a patient whose insurance will not pay for continuing hospitalization considers transferring the patient to a public psychiatric hospital. The family expresses concern that the patient will never get any treatment. Which reply by the nurse would be most helpful? a. Under the law, treatment must be provided. Hospitalization without treatment violates patients rights. b. Thats a justifiable concern because the right to treatment extends only to the provision of food, shelter, and safety. c. Much will depend on other patients, because the right to treatment for a psychotic patient takes precedence over the right to treatment of a patient who is stable. d. All patients in public hospitals have the right to choose both a primary therapist and a primary nurse.

ANS: A The right to medical and psychiatric treatment was conferred on all patients hospitalized in public mental hospitals with the enactment of the federal Hospitalization of Mentally Ill Act in 1964. Stating that the concern is justifiable supports the familys erroneous belief. The provisions mentioned in the third and fourth options are not part of this or any other statute governing psychiatric care.

Which principle takes priority for the psychiatric inpatient staff when addressing behavioral crises? a. Resolve behavioral crises using the least restrictive intervention possible. b. Rights of the majority of patients supersede the rights of individual patients. c. Swift intervention is justified to maintain the integrity of the therapeutic milieu. d. Allow patients opportunities to regain control without intervention if the safety of other patients is not compromised.

ANS: A The rule of using the least restrictive treatment or intervention possible to achieve the desired outcome is the patients legal right. Planned interventions are nearly always preferable. Intervention may be necessary when the patient threatens harm to self.

A psychiatric technician says, Little of what takes place on the behavioral health unit seems to be theory based. A nurse educates the technician by identifying which common use of Sullivans theory? a. Structure of the therapeutic milieu of most behavioral health units b. Frequent use of restraint and seclusion for behavior modification c. Assessment tools based on age-appropriate versus arrested behaviors d. Use of the nursing process to determine the best sequence for nursing actions

ANS: A The structure of the therapeutic environment has, as its foci, an accepting atmosphere and provision of opportunities for practicing interpersonal skills. Both constructs are directly attributable to Sullivans theory of interpersonal relationships. Sullivans interpersonal theory did not specifically consider the use of restraint or seclusion. Assessment based on the developmental level is associated with Eriksons theories. The nursing process applies concepts from multiple theories.

A veteran of the war in Afghanistan tells the nurse, "Everyday, something happens that makes me feel like I'm still there. My family has grown impatient with me. They say it's time or me to move on from that time in my life but I can't." What is the nurse's first priority? a. Assess the veteran for suicide risk. b. Refer the veteran or specialized mental health services. c. Assess the veteran for evidence of traumatic brain injury. d. Refer the veteran's family to a post-traumatic stress disorder group.

ANS: A The veteran had high risk or post-traumatic stress disorder (PTSD). When PTSD is untreated or under- treated, painful repercussions often occur, particularly marital problems, unemployment, heavy substance abuse, and suicide. The highest priority is an assessment of suicide risk (chapter 10, pg 122-123)

A 26-month-old child displays negative behaviors. The parent says, "My child refuses toilet training and shouts, 'No!' when given direction. What do you think is wrong?" Select the nurses best reply. a. This is normal for your childs age. The child is striving for independence. b. The child needs firmer control. Punish the child for disobedience and say, No. c. There may be developmental problems. Most children are toilet trained by age 2 years. d. Some undesirable attitudes are developing. A child psychologist can help you develop a remedial plan.

ANS: A These negative behaviors are typical of a child around the age of 2 years whose developmental task is to develop autonomy. The incorrect options indicate the childs behavior is abnormal.

While entering the building, an elementary school nurse observes a person in the distance emerging from a forest and approaching the school. The person is dressed in black from head to toe, wearing a backpack and carrying a long, narrow, dark object. Which action should the nurse take first? a. Move to a secure location b. Observe the intruder's features c. Take note of the intruder's location d. Activate the school code or an intruder

ANS: A This scenario presents a potential adventitious crisis in phase one. The nurse must first consider safety. After moving to a secure location, the nurse can activate the school's code or an intruder and describe the intruder to law enforcement. (chapter 20, pg 325 (Figure 20-1))

A multidisciplinary health care team meets 12 hours after an adolescent is hospitalized after a suicide attempt. Members of the team report their assessments. What outcome can be expected from this meeting? a. A treatment plan will be formulated. b. The health care provider will order neuroimaging studies. c. The team will request a court-appointed advocate for the patient. d. Assessment of the patients need for placement outside the home will be undertaken.

ANS: A Treatment plans are formulated early in the course of treatment to streamline the treatment process and reduce costs. It is too early to determine the need for alternative post-discharge living arrangements. Neuroimaging is not indicated for this scenario.

Which employers health plan is required to include parity provisions related to mental illnesses? a. Employer with more than 50 employees b. Cancer thrift shop staffed by volunteers c. Daycare center that employs 7 teachers d. Church that employs 15 people

ANS: A Under federal parity laws, companies with more than 50 employees may not limit annual or lifetime mental health benefits unless they also limit benefits for physical illnesses.

A nurse assesses an older adult patient brought to the emergency department by a family member. The patient was wandering outside, saying, I can't find my way home. The patient is confused and unable to answer questions. Select the nurses best action. a. Document the patients mental status. Obtain other assessment data from the family member. b. Record the patients answers to questions on the nursing assessment form. c. Ask an advanced practice nurse to perform the assessment interview.d. Call for a mental health advocate to maintain the patients rights.

ANS: A When the patient (primary source) is unable to provide information, secondary sources should be used, in this case the family member. Later, more data may be obtained from other relatives or neighbors who are familiar with the patient. An advanced practice nurse is not needed for this assessment; it is within the scope of practice of the staff nurse. Calling a mental health advocate is unnecessary.

A college student has been experiencing significant stress associated with academic demands. Last month, the student began attending yoga sessions three times a week. Which outcome indicates this activity has been successful? a. The student reports improved feelings of well-being. b. The student increases use of caffeine to enhance concentration. c. The student reports, "Now I am sleeping about 10 hours every day." d. The student says, "I withdrew from two courses to reduce my academic load."

ANS: A Yoga and other physical activities can be effective ways to manage stress. These activities deepen breathing, relieve muscle tension, and can elevate levels of the body's own endorphins, which induces a sense of well-being (chapter 10, pg 123 (Box 10-1))

Jon, an emergency department (ED) nurse, has just worked an 8-hour shift in the ED. After a five-car wreck during his midshift, five patients have been transported to the hospital with multiple injuries and one fatality. Jon looks exhausted, and his hands are tremulous. He insists that he is scheduled to work another 6 hours. As the nurse manager, what is your best action? A. Tell him that he needs to end his shift right away. B. Tell him he needs to nap for 30 minutes and then return to work. C. Tell him to go to the cafeteria, relax, and drink strong coffee. D. Tell him to eat a high carbohydrate snack, take a 30-minute break, and then return to work.

ANS: A Your role as nurse manager of patient care includes deciding who is safe to work, and Jon is not safe. Sleeping on the job is not an acceptable practice in a hospital setting. Although food and fluids may help with Jon's exhaustion, strong coffee could contribute to further tremulousness. Although food and a break may help with exhaustion, it is not a predictor that Jon will be safe in his work with patients (from ch 10 ppt).

Which is a responsibility that only an APRN-PMH can perform? A. Conducting psychotherapy B. Administering prescription medications C. Integrating biological and complementary therapies D. Adapting health instruction to a patient's specific needs

ANS: A from ch 7 ppt slide 25, no rationale provided

Which situations qualify as abandonment on the part of a nurse? (Select all that apply.) The nurse: a. allows a patient with acute mania to refuse hospitalization without taking further action. b. terminates employment without referring a seriously mentally ill for aftercare. c. calls police to bring a suicidal patient to the hospital after a suicide attempt. d. refers a patient with persistent paranoid schizophrenia to community treatment. e. asks another nurse to provide a patients care because of concerns about countertransference.

ANS: A, B Abandonment arises when a nurse does not place a patient safely in the hands of another health professional before discontinuing treatment. Calling the police to bring a suicidal patient to the hospital after a suicide attempt and referring a patient with schizophrenia to community treatment both provide for patient safety. Asking another nurse to provide a patients care because of concerns about countertransference demonstrates self-awareness.

An experienced nurse says to a new graduate, When you've practiced as long as I have, you will instantly know how to take care of psychotic patients. What is the new graduates best analysis of this comment? Select all that apply. a. The experienced nurse may have lost sight of patients individuality, which may compromise the integrity of practice. b. New research findings must be continually integrated into a nurses practice to provide the most effective care. c. Experience provides mental health nurses with the tools and skills needed for effective professional practice. d. Experienced psychiatric nurses have learned the best ways to care for psychotic patients through trial and error. e. Effective psychiatric nurses should be continually guided by an intuitive sense of patients needs.

ANS: A, B Evidence-based practice involves using research findings to provide the most effective nursing care. Evidence is continually emerging; therefore, nurses cannot rely solely on experience. The effective nurse also maintains respect for each patient as an individual. Overgeneralization compromises that perspective. Intuition and trial and error are unsystematic approaches to care.

A nurse ends a relationship with a patient. Which actions by the nurse should be included in the termination phase? Select all that apply. a. Focus dialog with the patient on problems that may occur in the future. b. Help the patient express feelings about the relationship with the nurse. c. Help the patient prioritize and modify socially unacceptable behaviors. d. Reinforce expectations regarding the parameters of the relationship. e. Help the patient identify strengths, limitations, and problems.

ANS: A, B The correct actions are part of the termination phase. The other actions are used in the working and orientation phases.

A professors 4-year-old child has a temperature of 101.6 F, diarrhea, and complains of stomach pain. The professor is scheduled to teach three classes today. Which actions by the professor demonstrate effective parenting? Select all that apply. a. Telephoning a grandparent to stay with the child at home for the day. b. Telephoning a colleague to teach his classes and staying home with the sick child. c. Taking the child to the university and keeping the child in a private office for the day. d. Taking the child to a daycare center and hoping daycare workers will not notice the child is sick. e. Giving the child one dose of ibuprofen (Motrin) and taking the child to the daycare center.

ANS: A, B The correct responses demonstrate fulfillment of the role as a parent. The distractors indicate the professor has not cared for the sick child in an effective way. Taking the child to a daycare center exposes other children to a potential infection. Taking the child to ones office does not keep the child comfortable or provide for the child while the professor is teaching.

A charge nurse is discussing mental status examinations with a newly licensed nurse. Which of the following statements by the newly licensed nurse indicates an understanding of the teaching (select all that apply) a. "To access cognitive ability, I should ask the client to count backward by seven." b. "To assess affect, I should observe the client's facial expression." c. "To assess language ability, I should instruct the client to write a sentence." d. "To assess remote memory, I should have the client repeat a list of objects." e. "To assess the client's abstract thinking, I should ask the client to identify our most recent presidents."

ANS: A, B, C Asking the client to repeat a list of objects is appropriate to assess immediate rather than remote memory. Asking the client to identify recent presidents is appropriate to assess cognitive knowledge rather than abstract thinking. (from ATI ch 1)

Which benefits are most associated with the use of telehealth? Select all that apply. a. Cost savings for patients b. Maximization of care management c. Access to services for patients in rural areas d. Prompt reimbursement by third-party payers e. Rapid development of trusting relationships with patients

ANS: A, B, C Use of telehealth technologies has shown that it can maximize health and improve disease management skills and confidence with the disease process. Many rural patients have felt disconnected from services; telehealth technologies can solve these problems. Although telehealths improved health outcomes regularly show cost savings for payers, one significant barrier is the current lack of reimbursement for remote patient monitoring by third-party payers. Telehealth is not associated with rapid development of trusting relationships.

While assessing your patient, Simon, a 63-year-old man in the psychiatric unit with a diagnosis of generalized anxiety disorder, he asks you, "Can you tell me why my family thinks that I am just acting sick to get attention?" Drawing from your knowledge of the impact of mental illness on families, which of the following would you include in your discussion to help Simon see his illness as a real illness? (Select all that apply.) A. Mental health is fundamental to health. B. Mental disorders are real health conditions that have an immense impact on individuals and families. C. The efficacy of mental health treatment is well documented. D. A range of treatments exists for most mental disorders.

ANS: A, B, C, D (question is from ch 2 ppt, no rationale included)

An individual says to the nurse, "I feel so stressed out lately. I think the stress is affecting my body also." Which somatic complaints are most likely to accompany this feeling? Select all that apply. a. Headache b. Neck pain c. Insomnia d. Anorexia e. Myopia

ANS: A, B, C, D When individuals feel stressed-out, they often have accompanying somatic complaints, especially associated with sleep, eating, and headache or back pain. Changes in vision, such as myopia, would not be expected.

A nurse is working in a community mental health facility. Which of the following services does this type of program provide? (select all that apply) a. Educational programs b. Medication dispensing programs c. Individual counseling programs d. Detoxification programs e. Family therapy

ANS: A, B, C, E All choices except for choice D are services provided in a community mental health facilities. Detoxification programs are services provided in a partial hospitalization program. (ATI ch 6)

A nurse can best address factors of critical importance to successful community treatment for persons with mental illness by including assessments related to which of the following? Select all that apply. a. Housing adequacy and stability b. Income adequacy and stability c. Family and other support systems d. Early psychosocial development e. Substance abuse history and current use

ANS: A, B, C, E Early psychosocial developmental history is less relevant to successful outcomes in the community than the assessments listed in the other options. If a patient is homeless or fears homelessness, focusing on other treatment issues is impossible. Sufficient income for basic needs and medication is necessary. Adequate support is a requisite to community placement. Substance abuse undermines medication effectiveness and interferes with community adjustment.

A patient states, Im starting cognitive behavioral therapy. What can I expect from the sessions? Which responses by the nurse are appropriate? Select all that apply. a. The therapist will be active and questioning. b. You may be given homework assignments c. The therapist will ask you to describe your dreams. d. The therapist will help you look at ideas and beliefs you have about yourself. e. The goal is to increase your subjectivity about thoughts that govern your behavior.

ANS: A, B, D Cognitive therapists are active rather than passive during therapy sessions because they help patients to reality test their thinking. Homework assignments are given and completed outside the therapy sessions. Homework is usually discussed at the next therapy session. The goals of cognitive therapy are to assist the patient to identify inaccurate cognitions, to reality test their thinking, and to formulate new, accurate cognitions. Dream describing applies to psychoanalysis, not cognitive behavioral therapy. The desired outcome of cognitive therapy is to assist patients in increasing their objectivity, not subjectivity, about the cognitions that influence behavior.

A nurse is preparing to implement cognitive reframing techniques for a client who has an anxiety disorder. Which of the following techniques should the nurse include in the plan of care? (select all that apply) a. Priority restructuring b. Monitoring thoughts c. Diaphragmatic breathing d. Journal keeping e. Meditation

ANS: A, B, D Diaphragmatic breathing and meditation are forms of behavioral therapy rather than a cognitive reframing technique. (from ATI ch 7)

Which patient statements identify qualities of nursing practice with high therapeutic value? (Select all that apply.) a. "My nurse talks in language I can understand." b. "My nurse helps me keep track of my medications." c. "My nurse is willing to go to social activities with me." d. "My nurse lets me do whatever I choose without interfering." e. "My nurse looks at me as a whole person with different needs."

ANS: A, B, E Each correct answer demonstrates caring is an example of appropriate nursing foci: communicating at a level understandable to the patient, using holistic principles to guide care, and providing medication supervision. The incorrect options suggest a laissez-faire attitude on the part of the nurse, when the nurse should instead provide thoughtful feedback and help patients test alternative solutions or violate boundaries.

A nurse performing an assessment interview for a patient with a substance abuse disorder decides to use a standardized rating scale. Which scales are appropriate? Select all that apply. a. Addiction Severity Index (ASI) b. Brief Drug Abuse Screen Test (B-DAST) c. Abnormal Involuntary Movement Scale (AIMS) d. Cognitive Capacity Screening Examination (CCSE) e. Recovery Attitude and Treatment Evaluator (RAATE)

ANS: A, B, E Standardized scales are useful for obtaining data concerning substance use disorders. The ASI, B-DAST, and RAATE are scales related to substance abuse. The AIMS assesses involuntary movements associated with antipsychotic medications. The CCSE assesses cognitive function.

A community member asks a nurse, "People diagnosed with mental illnesses used to go to a state hospital. Why has that changed?" Select the nurses accurate responses. Select all that apply. a. Science has made significant improvements in drugs for mental illness, so now many people may live in their communities. b. A better selection of less restrictive settings is now available in communities to care for individuals with mental illness. c. National rates of mental illness have declined significantly. The need for state institutions is actually no longer present. d. Most psychiatric institutions were closed because of serious violations of patients rights and unsafe conditions. e. Federal legislation and payment for treatment of mental illness have shifted the focus to community rather than institutional settings.

ANS: A, B, E The community is a less restrictive alternative than hospitals for the treatment of people with mental illness. Funding for treatment of mental illness remains largely inadequate but now focuses on community rather than institutional care. Antipsychotic medications improve more symptoms of mental illness; hence, management of psychiatric disorders has improved. Rates of mental illness have increased, not decreased. Hospitals were closed because funding shifted to the community. Conditions in institutions have improved.

Which descriptors exemplify consistency regarding therapeutic nurse-patient relationships? Select all that apply. a. Having the same nurse care for a patient on a daily basis b. Encouraging a patient to share initial impressions of staff c. Providing a schedule of daily activities to a patient d. Setting a time for regular sessions with a patient e. Offering solutions to a patients problems

ANS: A, C, D Consistency implies predictability. Having the same nurse see the patient daily, providing a daily schedule of patient activities, and setting a regular time for sessions help a patient to predict what will happen during each day and to develop a greater degree of security and comfort. Encouraging a patient to share initial impressions of staff and giving advice are not related to consistency and would not be considered a therapeutic intervention.

A new psychiatric nurse has a parent diagnosed with bipolar disorder. This nurse angrily recalls embarrassing events concerning the parents behavior in the community. Select the best ways for this nurse to cope with these feelings. Select all that apply. a. Seek ways to use the understanding gained from childhood to help patients cope with their own illnesses. b. Recognize that these feelings are unhealthy, and try to suppress them when working with patients. c. Recognize that psychiatric nursing is not an appropriate career choice, and explore other nursing specialties. d. Begin new patient relationships by saying, "My own parent had mental illness, so I accept it without stigma." e. Recognize that the feelings may add sensitivity to the nurses practice, but supervision is important.

ANS: A, E The nurse needs to explore these feelings. An experienced psychiatric nurse is a resource who may be helpful. The knowledge and experience gained from the nurses relationship with a parent who is mentally ill may contribute sensitivity to a compassionate practice. Self-disclosure and suppression are not adaptive coping strategies. The nurse should not give up on this area of practice without first seeking ways to cope with the memories.

A nurse should introduce the matter of a contract during the first session with a new patient because contracts: a. specify what the nurse will do for the patient. b. spell out the participation and responsibilities of each party. c. indicate the feeling tone established between the participants. d. are binding and prevent either party from prematurely ending the relationship.

ANS: B A contract emphasizes that the nurse works with the patient rather than doing something for the patient. Working with is a process that suggests each party is expected to participate and share responsibility for the outcomes. Contracts do not, however, stipulate roles or feeling tone, or that premature termination is forbidden.

A patient comes to the clinic with superficial cuts on the left wrist. The patient paces around the room sobbing but cringes when approached and responds to questions with only shrugs or monosyllables. Select the nurses best initial statement to this patient. a. Everything is going to be all right. You are here at the clinic, and the staff will keep you safe. b. I see you are feeling upset. I am going to stay and talk with you to help you feel better. c. You need to try to stop crying so we can talk about your problems. d. Lets set some guidelines and goals for your visit here.

ANS: B A crisis exists for this patient. The two primary thrusts of crisis intervention are to provide for the safety of the individual and use anxiety-reduction techniques to facilitate the use of inner resources. The nurse offers therapeutic presence, which provides caring, ongoing observation relative to the patients safety. The incorrect responses use nontherapeutic techniques, including false reassurance and giving advice.

A nurse consistently strives to demonstrate caring behaviors during interactions with patients. Which reaction by a patient indicates this nurse is effective? A patient reports feeling: a. distrustful of others. b. connected with others. c. uneasy about the future. d. discouraged with efforts to improve.

ANS: B A patient is likely to respond to caring with a sense of connectedness with others. The absence of caring can make patients feel distrustful, disconnected, uneasy, and discouraged.

A voluntarily hospitalized patient tells the nurse, Get me the forms for discharge against medical advice so I can leave now. What is the nurses best initial response? a. I cant give you those forms without your health care providers knowledge. b. I will get them for you, but lets talk about your decision to leave treatment. c. Since you signed your consent for treatment, you may leave if you desire. d. I'll get the forms for you right now and bring them to your room.

ANS: B A patient who has been voluntarily admitted as a psychiatric inpatient has the right to demand and obtain release in most states. However, as a patient advocate, the nurse is responsible for weighing factors related to the patients wishes and best interests. By asking for information, the nurse may be able to help the patient reconsider the decision. The statement that discharge forms cannot be given without the health care providers knowledge is not true. Facilitating discharge without consent is not in the patients best interest before exploring the reason for the request.

A patient is admitted to the psychiatric hospital. Which assessment finding best indicates that the patient has a mental illness? The patient: a. describes coping and relaxation strategies used when feeling anxious. b. describes mood as consistently sad, discouraged, and hopeless. c. can perform tasks attempted within the limits of own abilities. d. reports occasional problems with insomnia.

ANS: B A patient who reports having a consistently negative mood is describing a mood alteration. The incorrect options describe mentally healthy behaviors and common problems that do not indicate mental illness.

An adolescent comes to the crisis clinic and reports sexual abuse by an uncle. The patient told the parents about the uncles behavior, but the parents did not believe the adolescent. What type of crisis exists? a. Maturational b. Adventitious c. Situational d. Organic

ANS: B An *adventitious* crisis is a crisis of disaster that is not a part of everyday life; it is unplanned or accidental. Adventitious crises include natural disasters, national disasters, and crimes of violence. Sexual molestation falls within this classification. Maturational crisis occurs as an individual arrives at a new stage of development, when old coping styles may be ineffective. Situational crisis arises from an external source such as a job loss, divorce, or other loss affecting self-concept or self-esteem. Organic is not a type of crisis.

A nurse is caring for a group of clients. Which of the following clients should a nurse consider for referral to an assertive community treatment (ACT) group? a. A client in an acute care mental health facility who has fallen several times while running down the hallway b. A client who lives at home and keeps "forgetting" to come in for a scheduled monthly antipsychotic injection for schizophrenia c. A client in a day treatment program who reports increasing anxiety during group therapy d. A client in a weekly grief support group who reports still missing a deceased partner who has been dead for 3 months

ANS: B An ACT group works with clients who are nonadherent with traditional therapy, such as the client in a home setting who keeps "forgetting" his injection.A client in acute care who has been running and falling should be helped by the treatment team on her unit. A client who has anxiety might be referred to his counselor or mental health provider. A client who is grieving for her husband who died 3 months ago is currently involved in an appropriate intervention. (ATI ch 6)

A nurse is talking with a client who reports experiencing increased stress because a new partner is "pressuring me and my kids to go live with him. I love him, but I'm not ready to do that." Which of the following recommendations should the nurse make to promote a change in the client's situation? a. Learn to practice mindfulness b. Use assertiveness techniques c. Exercise regularly d. Rely on the support of a close friend

ANS: B Assertive communication allows the client to assert her feelings and then make a change in the situation.Mindfulness is appropriate to decrease the client's stress. However, it does not promote a change in the client's situation. Regular exercise is appropriate to decrease the client's stress. However, it does not promote a change in the client's situation. Social support is appropriate to decrease the client's stress. However, it does not promote a change in the client's situation. (ATI ch 9)

A nurse asks a patient, "If you had fever and vomiting for 3 days, what would you do?" Which aspect of the mental status examination is the nurse assessing? a. Behavior b. Cognition c. Affect and mood d. Perceptual disturbances

ANS: B Assessing cognition involves determining a patients judgment and decision-making capabilities. In this case, the nurse expects a response of "Call my doctor" if the patients cognition and judgment are intact. If the patient responds, "I would stop eating", or "I would just wait and see what happened," the nurse would conclude that judgment is impaired. The other options refer to other aspects of the examination.

Before assessing a new patient, a nurse is told by another health care worker, I know that patient. No matter how hard we work, there isnt much improvement by the time of discharge. The nurses responsibility is to: a. document the other workers assessment of the patient. b. assess the patient based on data collected from all sources. c. validate the workers impression by contacting the patients significant other. d. discuss the workers impression with the patient during the assessment interview.

ANS: B Assessment should include data obtained from both the primary and reliable secondary sources. Biased assessments by others should be evaluated as objectively as possible by the nurse, keeping in mind the possible effects of countertransference.

After leaving work, a staff nurse realizes that documentation of the administration of a medication to a patient was omitted. This off-duty nurse telephones the unit and tells the nurse, Please document the administration of the medication I forgot to do. My password is alpha1. The nurse should: a. fulfill the request. b. refer the matter to the charge nurse to resolve. c. access the record and document the information. d. report the request to the patients health care provider.

ANS: B At most hospitals, termination is a possible penalty for unauthorized entry into a patient record. Referring the matter to the charge nurse will allow the observance of hospital policy while ensuring that documentation occurs. Making an exception and fulfilling the request places the on-duty staff nurse in jeopardy. Reporting the request to the patients health care provider would be unnecessary. Accessing the record and documenting the information would be unnecessary when the charge nurse can resolve the problem.

A nurse is teaching a client who has an anxiety disorder and is scheduled to begin classical psychoanalysis. Which of the following client statements indicated an understanding of this form of therapy? a. "Even if my anxiety improves, I will need to continue this therapy for 6 weeks." b. "The therapist will focus on my past relationships during our sessions." c. "Psychoanalysis will help me reduce my anxiety by changing my behaviors." d. "This therapy will address my conscious feelings about stressful experiences."

ANS: B Classical psychoanalysis is a therapeutic process that requires many sessions over months to years. Classical psychoanalysis places a common focus on past relationships to identify the cause of the anxiety disorder. Classical psychoanalysis focuses on identifying and resoling the cause of the anxiety rather than changing the behavior. Classical psychoanalysis assesses unconscious, rather than conscious, thoughts and feelings.(from ATI ch 7)

Cortisol is released in response to a patients prolonged stress. Which initial effect would the nurse expect to result from the increased cortisol level? a. Diuresis and electrolyte imbalance b. Focused and alert mental status c. Drowsiness and lethargy d. Restlessness and anxiety

ANS: B Cortisol is the primary stress hormone and is released in response to prolonged stress. Cortisol helps supply cells with amino acids and fatty acids for energy supply, as well as diverting glucose from muscles for use by the brain. As a result, the brain stays alert and focused. The distractors present effects that would not be expected.

Which scenario best demonstrates an example of eustress? An individual: a. loses a beloved family pet. b. prepares to take a 1 week vacation to a tropical island with a group of close friends. c. receives a bank notice there were insufficient funds in their account for a recent rent payment. d. receives notification that their current employer is experiencing financial problems and some workers will be terminated.

ANS: B Eustress is beneficial stress; it motivates people to develop skills they need to solve problems and meet personal goals. Positive life experiences produce eustress. Going on a tropical vacation is an exciting, relaxing experience and an example of eustress. Losing the family pet, worrying about employment security, and having financial problems are examples of distress, a negative experience that drains energy and can lead to significant emotional problems.

Which nursing intervention demonstrates false imprisonment? a. A confused and combative patient says, Im getting out of here and no one can stop me. The nurse restrains this patient without a health care providers order and then promptly obtains an order. b. A patient has been irritating, seeking the attention of nurses most of the day. Now a nurse escorts the patient down the hall, saying, Stay in your room or youll be put in seclusion. c. An involuntarily hospitalized patient with suicidal ideation runs out of the psychiatric unit. A nurse rushes after the patient and convinces the patient to return to the unit. d. An involuntarily hospitalized patient with suicidal ideation attempts to leave the unit. A nurse calls the security team and uses established protocols to prevent the patient from leaving.

ANS: B False imprisonment involves holding a competent person against his or her will. Actual force is not a requirement of false imprisonment. The individual needs only to be placed in fear of imprisonment by someone who has the ability to carry out the threat. The patient in one distractor is not competent, and the nurse is acting beneficently. The patients in the other distractors have been admitted as involuntary patients and should not be allowed to leave without permission of the treatment team.

A patient shows the nurse an article from the Internet about a health problem. Which characteristic of the web sites address most alerts the nurse that the site may have biased and prejudiced information? a. Address ends in .org. b. Address ends in .com. c. Address ends in .gov. d. Address ends in .net.

ANS: B Financial influences on a site are a clue that the information may be biased. .com at the end of the address indicates that the site is a commercial one. .gov indicates that the site is maintained by a government entity. .org indicates that the site is nonproprietary; the site may or may not have reliable information, but it does not profit from its activities. .net can have multiple meanings.

A soldier returned home from active duty in a combat zone in Afghanistan and was diagnosed with post-traumatic stress disorder (PTSD). The soldier says, "If there's a loud noise at night, I get under my bed because I think we're getting bombed." What type of experience has the soldier described? a. Illusion b. Flashback c. Nightmare d. Auditory hallucination

ANS: B Flashbacks are dissociative reactions in which an individual feels or acts as if the traumatic event were recurring. Illusions are misinterpretations of stimuli; although the experience is similar, the more accurate term is flashback because of the diagnosis of PTSD. Auditory hallucinations have no external stimuli. Nightmares commonly accompany PTSD, but this experience is stimulated by an actual environmental sound.

A nurse wants to demonstrate genuineness with a patient diagnosed with schizophrenia. The nurse should: a. restate what the patient says. b. use congruent communication strategies. c. use self-disclosure in patient interactions. d. consistently interpret the patients behaviors.

ANS: B Genuineness is a desirable characteristic involving an awareness of ones own feelings as they arise and the ability to communicate them when appropriate. The incorrect options are undesirable in a therapeutic relationship.

A community mental health nurse has worked for 6 months to establish a relationship with a delusional, suspicious patient. The patient recently lost employment and stopped taking medications because of inadequate money. The patient says, "Only a traitor would make me go to the hospital." Which solution is best? a. Arrange a bed in a local homeless shelter with nightly onsite supervision. b. Negotiate a way to provide medication so the patient can remain at home. c. Hospitalize the patient until the symptoms have stabilized. d. Seek inpatient hospitalization for up to 1 week.

ANS: B Hospitalization may damage the nurse-patient relationship even if it provides an opportunity for rapid stabilization. If medication can be obtained and restarted, the patient can possibly be stabilized in the home setting, even if it takes a little longer. A homeless shelter is inappropriate and unnecessary. Hospitalization may be necessary later, but a less restrictive solution should be tried first because the patient is not dangerous.

A 26-month-old child displays negative behavior, refuses toilet training, and often shouts, "No!" when given directions. Using Freuds stages of psychosexual development, a nurse would assess the childs behavior is based on which stage? a. Oral b. Anal c. Phallic d. Genital

ANS: B In Freuds stages of psychosexual development, the anal stage occurs from age 1 to 3 years and has, as its focus, toilet training and learning to delay immediate gratification. The oral stage occurs between birth and 1 year, the phallic stage occurs between 3 and 5 years, and the genital stage occurs between 13 and 20 years.

The nurse in a high school meets with small groups of students the day after a school bus accident resulted in the death of five students. Which comment should the nurse use to begin the session? a. "Sometimes life is not fair .Yesterday's tragedy is an example of just how unfair it can be." b. "We're grateful that you are safe. Our discussion is to talk about feelings associated with yesterday's tragedy." c. "We've had a terrible loss .I also feel your pain. You need to talk about your feelings associated with the event." d. "Thank you for coming today. As school leaders, we know it is very important to respond to yesterday's tragedy."

ANS: B In phase 1 of a crisis, a person faces a conflict or problem that threatens the self-concept and responds with increased feelings of anxiety. The nurse should first assure students that they are safe and then specify the reason for the session. (chapter 20, pg 327)

A patient comes to the crisis center saying, "I'm in a terrible situation. I don't know what to do." The triage nurse can initially assume that the patient is: a. suicidal. b. anxious and fearful. c. misperceiving reality. d. potentially homicidal.

ANS: B Individuals in crisis are universally anxious. They are often frightened and may be mildly confused. Perceptions are often narrowed.

A nurse decides to put a client who has a psychotic disorder in seclusion overnight because the unit is very short-staffed, and the client frequently fights with other clients. The nurse's actions are an example of which of the following torts? a. Invasion of privacy b. False imprisonment c. Assault d. Battery

ANS: B Invasion of privacy is the sharing or obtaining of the client's confidential information without the client's consent. *False imprisonment is the confining of a client to a specific area if the reason for such confinement is for the convenience of the staff.* Assault is making a threat to the client's person. Battery involves causing intentional, physical harm to clients. (ATI ch 2)

A patient diagnosed with schizophrenia tells the nurse, "The CIA is monitoring us through the fluorescent lights in this room. Be careful what you say." Which response by the nurse would be most therapeutic? a. Lets talk about something other than the CIA. b. It sounds like you're concerned about your privacy. c. The CIA is prohibited from operating in health care facilities. d. You have lost touch with reality, which is a symptom of your illness.

ANS: B It is important not to challenge the patients beliefs, even if they are unrealistic. Challenging undermines the patients trust in the nurse. The nurse should try to understand the underlying feelings or thoughts the patients message conveys. The correct response uses the therapeutic technique of reflection. The other comments are nontherapeutic. Asking to talk about something other than the concern at hand is changing the subject. Saying that the CIA is prohibited from operating in health care facilities gives false reassurance. Stating that the patient has lost touch with reality is truthful but uncompassionate.

A nurse is conducting a family therapy session. The younger child tells the nurse about plans to make the older sibling look bad, believing that this will earn more freedom and privileges. The nurse should identify this dysfunctional behavior as which of the following? a. Placation b. Manipulation c. Blaming d. Distraction

ANS: B Manipulation is the dysfunctional behavior of using dishonesty to support an individual agenda. Blaming is the dysfunctional behavior of blaming others to shift focus away from the individual's own inadequacies. Placation is taking the blame in order to maintain peace. Distraction is inserting irrelevant information into an argument. (from ATI ch 8)

A nurse is conducting chart reviews of multiple clients at a community mental health facility. Which of the following events is an example of client experiencing a maturational crisis? A. Rape B. Marriage C. Severe physical illness D. Job loss

ANS: B Marriage is an example of a maturational crisis, which is a naturally occurring event during the life span. Rape is an example of an adventitious crisis; it is not a part of everyday life. Severe physical illness and loss of a job are examples of situational crises. (ATI ch 29)

Considering Maslow's pyramid, which comment indicates an individual is motivated by the highest level of need? a. "Even though I'm 40 years old, I have returned to college so I can get a better job." b. "I help my community by volunteering at a thrift shop that raises money or the poor." c. "I recently applied or public assistance in order to feed my family, but I hope it's not forever." d. "My children tell me I'm a good parent. I feel happy being part of a family that appreciates me."

ANS: B Maslow's hierarchy of needs are placed conceptually on a pyramid, with the most basic and important needs on the lower level. The higher levels, the more distinctly human needs, occupy the top sections of the pyramid. When lower-level needs are met, higher-level needs are able to emerge. Self-actualization and esthetics are the highest-level needs. (chapter 3, pg 24 (Figure 3-2))

The patient says, "My marriage is just great. My spouse and I usually agree on everything." The nurse observes the patients foot moving continuously as the patient twirls a shirt button. What conclusion can the nurse draw? The patients communication is: a. clear. b. mixed. c. precise. d. inadequate.

ANS: B Mixed messages involve the transmission of conflicting or incongruent messages by the speaker. The patients verbal message that all is well in the relationship is modified by the nonverbal behaviors denoting anxiety. Data are not present to support the choice of the verbal message being clear, explicit, or inadequate.

Because he works with young men in a treatment center for domestic abuse, Ernesto is always careful to be respectful of both patients and staff—"especially women," he tells a coworker. Which educational tool is Ernesto demonstrating most clearly? A. Empathy B. Modeling C. Transference D. Value teaching

ANS: B Modeling (Presents a vivid example of values in action) (from ch 8 ppt).

Which situation demonstrates the use of primary care related to crisis intervention? a. Implementing suicide precautions for a patient with depression. b. Teaching stress reduction techniques to a beginning student nurse. c. Assessing coping strategies used by a patient who has attempted suicide. d. Referring a patient with schizophrenia to a partial hospitalization program.

ANS: B Primary crisis intervention promotes mental health and reduces mental illness. The incorrect options are examples of secondary or tertiary interventions.

Which entry in the medical record best meets the requirement for problem-oriented charting? a. A: Pacing and muttering to self. P: Sensory perceptual alteration, related to internal auditory stimulation. I: Given fluphenazine (Prolixin) 2.5 mg at 0900, and went to room to lie down. E: Calmer by 0930. Returned to lounge to watch TV. b. S: States, "I feel like I'm ready to blow up." O: Pacing hall, mumbling to self. A: Auditory hallucinations. P: Offer haloperidol (Haldol) 2 mg. I: (Haldol) 2 mg at 0900. E: Returned to lounge at 0930 and quietly watched TV. c. Agitated behavior. D: Patient muttering to self as though answering an unseen person. A: Given haloperidol (Haldol) 2 mg and went to room to lie down. E: Patient calmer. Returned to lounge to watch TV. d. Pacing hall and muttering to self as though answering an unseen person. haloperidol (Haldol) 2 mg administered at 0900 with calming effect in 30 minutes. Stated, Im no longer bothered by the voices.

ANS: B Problem-oriented documentation uses the first letter of key words to organize data: S for subjective data, O for objective data, A for assessment, P for plan, I for intervention, and E for evaluation. The distractors offer examples of PIE charting, focus documentation, and narrative documentation.

The acronym QSEN refers to: a. Qualitative Standardized Excellence in Nursing. b. Quality and Safety Education for Nurses. c. Quantitative Effectiveness in Nursing. d. Quick Standards Essential for Nurses.

ANS: B QSEN represents national initiatives centered on patient safety and quality. The primary goal of QSEN is to prepare future nurses with the knowledge, skills, and attitudes to increase the quality, care, and safety in the health care setting in which they work.

Which outcome, focused on recovery, would be expected in the plan of care for a patient living in the community and diagnosed with serious and persistent mental illness? Within 3 months, the patient will: a. deny suicidal ideation. b. report a sense of well-being. c. take medications as prescribed. d. attend clinic appointments on time.

ANS: B Recovery emphasizes managing symptoms, reducing psychosocial disability, and improving role performance. The goal of recovery is to empower the individual with mental illness to achieve a sense of meaning and satisfaction in life and to function at the highest possible level of wellness. The incorrect options focus on the classic medical model rather than recovery.

Which research evidence would most influence a group of nurses to change their practice? a. Expert committee report of recommendations for practice b. Systematic review of randomized controlled trials c. Nonexperimental descriptive study d. Critical pathway

ANS: B Research findings are graded using a hierarchy of evidence. A systematic review of randomized controlled trials is Level A and provides the strongest evidence for changing practice. Expert committee recommendations and descriptive studies lend less powerful and influential evidence. A critical pathway is not evidence; it incorporates research findings after they have been analyzed.

Which step in SOAPIE involves interpretation of two kinds of data in identifying a problem or a nursing diagnosis? A. "S" B. "A" C. "P" D. "I"

ANS: B S: Subjective data (patient statement) O: Objective data (nurse observations) *A: Assessment (nurse interprets S and O and describes either a problem or a nursing diagnosis)* P: Plan (proposed intervention) I: Interventions (nurse's response to problem) E: Evaluation (patient outcome)

A community psychiatric nurse assesses that a patient diagnosed with a mood disorder is more depressed than on the previous visit a month ago; however, the patient says, "I feel the same." Which intervention supports the nurses assessment while preserving the patients autonomy? a. Arrange for a short hospitalization. b. Schedule weekly clinic appointments. c. Refer the patient to the crisis intervention clinic. d. Call the family and ask them to observe the patient closely.

ANS: B Scheduling clinic appointments at shorter intervals will give the opportunity for more frequent assessment of symptoms and allow the nurse to use early intervention. If the patient does not admit to having a crisis or problem, a referral would be useless. The remaining options may produce unreliable information, violate the patients privacy, and waste scarce resources.

A nurse wants to enhance the growth of a patient by showing positive regard. The action consistent with this wish is: a. making rounds daily. b. staying with a tearful patient. c. administering daily medication as prescribed. d. examining personal feelings about a patient.

ANS: B Staying with a crying patient offers support and shows positive regard. Administering daily medication and making rounds are tasks that could be part of an assignment and do not necessarily reflect positive regard. Examining feelings regarding a patient addresses the nurses ability to be therapeutic.

Which organization actively seeks to reduce the stigma associated with mental illness through public presentations such as In Our Own Voice (IOOV)? a. American Psychiatric Association (APA) b. National Alliance on Mental Illness (NAMI) c. United States Department of Health and Human Services (USDHHS) d. North American Nursing Diagnosis Association International (NANDA-I)

ANS: B Stigma represents the bias and prejudice commonly held regarding mental illness. NAMI actively seeks to dispel misconceptions about mental illness. NANDA-I defines approved nursing diagnoses. The APA publishes the DSM 5. The USDHHS regulates and administers health policies.

A patient diagnosed with liver failure has been on the transplant waiting list 8 months. The patient says, "Why is it taking so long to have the surgery? Maybe I'm meant to die for all the bad things I've done." The nurse should document the patients comment in which section of the assessment? a. Physical b. Spiritual c. Financial d. Psychological

ANS: B Stress can be evident in a persons spirituality. This patients comment indicates questioning of ones place in the universe and consequences for wrongdoing, both of which are elements of spirituality. Stress can be related to psychological, physical, or psychosocial well-being, but spirituality is the best answer.

An individual lives in a community adjacent to a military base. Loud jets fly overhead multiple times daily. The person tells the nurse, "They're so loud I can't hear myself think." What is the nurse's best first action? a. Direct the individual to report the jet noise to local authorities. b. Teach relaxation and stress reduction techniques to the individual. c. Assess the individual or sensory impairments, particularly auditory. d. Encourage the individual to form a community action group to oppose noise pollution.

ANS: B Stress can be psychological (e.g., anxiety, guilt, or joy) or physical (e.g., stress ul environment, such as loud noises, extreme heat or cold, or other dis- turbing physical condition). Stress is a part of everyday life for everyone. Skills in stress reduction will assist the individual to cope with the jet noise. Later, the individual may consider the other options. (chapter 10, pg 121, (Figure 10-1), pg 123 (Box 10-1))

A woman says, "I can't take anymore! Last year my husband had an affair and now we don't communicate. Three months ago, I found a lump in my breast. Yesterday my daughter said she's quitting college." If this person's immediate family is unable to provide sufficient situational support, the nurse should: a. suggest hospitalization for a short period. b. ask what other relatives or friends are available for support. c. tell the patient, "You must be strong. Don't let this crisis overwhelm you." d. foster insight by relating the present situation to earlier situations involving loss.

ANS: B The assessment of situational supports should continue. Although the patients nuclear family may not be supportive, other situational supports may be available. If they are adequate, admission to an inpatient unit will be unnecessary. Psychotherapy is not appropriate for crisis intervention. Advice is usually nontherapeutic.

During the initial interview at the crisis center, a patient says, "I've been served with divorce papers. I'm so upset and anxious that I can't think clearly." What could the nurse say to assess personal coping skills? a. What would you like us to do to help you feel more relaxed? b. In the past, how did you handle difficult or stressful situations c. Do you think you deserve to have things like this happen to you? d. I can see you are upset. You can rely on us to help you feel better.

ANS: B The correct answer is the only option that assesses coping skills. The incorrect options offer unrealistic reassurance, are concerned with self-esteem, and ask the patient to decide on treatment at a time when he or she cannot think clearly.

A soldier returns to the United States from active duty in a combat zone in Afghanistan. The soldier is diagnosed with post-traumatic stress disorder (PTSD). Which comment by the soldier requires the nurses immediate attention? a. It's good to be home. I missed my family and friends. b. I saw my best friend get killed by a roadside bomb. It should have been me instead. c. Sometimes I think I hear bombs exploding, but it's just the noise of traffic in my hometown. d. I want to continue my education but I'm not sure how I will fit in with other college students.

ANS: B The correct response indicates the soldier is thinking about death and feeling survivors guilt. These emotions may accompany suicidal ideation, which warrants the nurses follow- up assessment. Suicide is a high risk among military personnel diagnosed with PTSD. One distractor indicates flashbacks, which is common with individuals with PTSD but is not solely indicative of further problems. The other distractors are normal emotions associated with returning home and change.

What is the desirable outcome for the orientation stage of a nurse-patient relationship? The patient will demonstrate behaviors that indicate: a. great sense of independence b. rapport and trust with the nurse. c. self-responsibility and autonomy. d. resolution of feelings of transference.

ANS: B The development of rapport and trust is necessary before the relationship can progress to the working phase. Behaviors indicating a greater sense of independence, self-responsibility, and resolved transference occur in the working phase.

A patient diagnosed with schizophrenia believes evil spirits are being summoned by a local minister and verbally threatens to bomb a local church. The psychiatrist notifies the minister. The psychiatrist has: a. released information without proper authorization. b. demonstrated the duty to warn and protect. c. violated the patients confidentiality. d. avoided charges of malpractice.

ANS: B The duty of a health care professional is to warn or notify an intended victim after a threat of harm has been made. Informing a potential victim of a threat is a legal responsibility of the health care professional and not considered a violation of confidentiality.

A patient diagnosed with schizophrenia has been stable in the community. Today, the spouse reports the patient is expressing delusional thoughts. The patient says, "I'm willing to take my medicine, but I forgot to get my prescription refilled." Which outcome should the nurse add to the plan of care? a. Nurse will obtain prescription refills every 90 days and deliver them to the patient. b. Patients spouse will mark dates for prescription refills on the family calendar. c. Patient will report to the hospital for medication follow-up every week. d. Patient will call the nurse weekly to discuss medication-related issues.

ANS: B The nurse should use the patient's support system to meet patient needs whenever possible. Delivery of medication by the nurse should be unnecessary for the nurse to do if the patient or a significant other can be responsible. The patient may not need more intensive follow-up as long as he or she continues to take the medications as prescribed. No patient issues except failure to obtain medication refills were identified.

Which communication technique is used more in crisis intervention than traditional counseling? a. Role modeling b. Giving direction c. Information giving d. Empathic listening

ANS: B The nurse working in crisis intervention must be creative and flexible in looking at the patients situation and suggesting possible solutions to the patient. Giving direction is part of the active role a crisis intervention therapist takes. The other options are used equally in crisis intervention and traditional counseling roles.

Two nursing students discuss career plans after graduation. One student wants to enter psychiatric nursing. The other student asks, "Why would you want to be a psychiatric nurse? All they do is talk. You will lose your skills." Select the best response by the student interested in psychiatric nursing. a. Psychiatric nurses practice in safer environments than other specialties. Nurse-to-patient ratios must be better because of the nature of patients problems. b. Psychiatric nurses use complex communication skills, as well as critical thinking, to solve multidimensional problems. I'm challenged by those situations. c. I think I will be good in the mental health field. I do not like clinical rotations in school, so I do not want to continue them after I graduate. d. Psychiatric nurses do not have to deal with as much pain and suffering as medical surgical nurses. That appeals to me.

ANS: B The practice of psychiatric nursing requires a different set of skills than medical surgical nursing, although substantial overlap does exist. Psychiatric nurses must be able to help patients with medical and mental health problems, reflecting the holistic perspective these nurses must have. Nurse-patient ratios and workloads in psychiatric settings have increased, similar to other specialties. Psychiatric nursing involves clinical practice, not simply documentation. Psychosocial pain is real and can cause as much suffering as physical pain.

After formulating the nursing diagnoses for a new patient, what is the next action a nurse should take? a. Design interventions to include in the plan of care. b. Determine the goals and outcome criteria. c. Implement the nursing plan of care. d. Complete the spiritual assessment.

ANS: B The third step of the nursing process is planning and outcome identification. Outcomes cannot be determined until the nursing assessment is complete and the nursing diagnoses have been formulated.

A nurse is in the working phase of a therapeutic relationship with a client who has methamphetamine use disorder. Which of the following actions indicates transference behavior? a. The client asks the nurse asks the nurse if they will go out to dinner together. b. The client accuses the nurse of being controlling just like an ex-partner. c. The client reminds the nurse of a friend who died from substance toxicity. d. The client becomes angry and threatens to engage in self harm.

ANS: B When a client views the nurse as having characteristics of another person who has been significant to his personal life, such as his ex-girlfriend, this indicates transference.The client asking the nurse to dinner indicates the need to discuss boundaries but does not indicate transference. The client reminding the nurse of a friend who died from substance toxicity indicates countertransference rather than transference. The client becoming angry and threatening to engage in self-harm indicates the need for safety intervention but does not indicate transference. (ATI ch 5)

A nurse hears a newly licensed nurse discussing a client's hallucinations in the hallway with another nurse. Which of the following actions should the nurse take first? a. Notify the nurse manager. b. Tell the nurse to stop discussing the behavior. c. Provide an in-service program about confidentiality. d. Complete an incident report.

ANS: B i. Notify the nurse manager if the client's right to privacy is violated but the keyword in this question is *first. The greatest risk to this client is an invasion of privacy through the sharing of confidential information in a public place.* Answers C and D are also correct actions to take however, they are not the first actions to be taken. (ATI ch 2)

A patient cries as the nurse explores the patients relationship with a deceased parent. The patient says, "I shouldn't be crying like this. It happened a long time ago." Which responses by the nurse will facilitate communication? Select all that apply. a. Why do you think you are so upset? b. I can see that you feel sad about this situation. c. The loss of your parent is very painful for you. d. Crying is a way of expressing the hurt you're experiencing. e. Let's talk about something else because this subject is upsetting you.

ANS: B, C, D Reflecting (I can see that you feel sad or This is very painful for you) and giving information (Crying is a way of expressing hurt) are therapeutic techniques. Why questions often imply criticism or seem intrusive or judgmental, and they are difficult to answer. Changing the subject is a barrier to communication.

A new nurse tells a mentor, "I want to convey to my patients that I am interested in them and that I want to listen to what they have to say." Which behaviors are helpful in meeting the nurses goal? Select all that apply. a. Sitting behind a desk, facing the patient. b. Introducing self to a patient and identifying own role. c. Using facial expressions that convey interest and encouragement. d. Assuming an open body posture and sometimes mirror imaging. e. Maintaining control of the topic under discussion by asking direct questions.

ANS: B, C, D Trust is fostered when the nurse gives an introduction and identifies his or her role. Facial expressions that convey interest and encouragement support the nurses verbal statements to that effect and strengthen the message. An open body posture conveys openness to listening to what the patient has to say. Mirror imaging enhances patient comfort. A desk would place a physical barrier between the nurse and patient. A face-to-face stance should be avoided when possible, and a less intense, 90- or 120-degree angle is used to permit either party to look away without discomfort. Once introductions have been made, the nurse focuses the interview on the patient by using open-ended questions, such as, Where should we start?

A patient is very suspicious and states, The FBI has me under surveillance. Which strategies should a nurse use when gathering initial assessment data about this patient? Select all that apply. a. Tell the patient that medication will help this type of thinking. b. Ask the patient, Tell me about the problem as you see it. c. Seek information about when the problem began. d. Tell the patient, Your ideas are not realistic. e. Reassure the patient, You are safe here.

ANS: B, C, E During the assessment interview, the nurse should listen attentively and accept the patients statements in a nonjudgmental way. Because the patient is suspicious and fearful, reassuring safety may be helpful, although trust is unlikely so early in the relationship. Saying that medication will help or telling the patient that the ideas are not realistic will undermine the development of trust between the nurse and patient.

A nurse is involved in a serious and prolonged mass casualty incident in the emergency department. Which of the following strategies should the nurse use to help prevent developing a trauma‑related disorder? (Select all that apply) A. Avoid thinking about the incident when it is over. B. Take breaks during the incident for food and water. C. Debrief with others following the incident D. Hold emotions in check in the days following the incident. E. Take advantage of offered counseling.

ANS: B, C, E Taking breaks and remembering to drink water and eat nutritious foods while working during a traumatic incident can help prevent development of a trauma‑related disorder. Debriefing with others following a traumatic incident can help prevent development of a trauma‑related disorder.Thinking and talking about a traumatic incident can help prevent development of a trauma‑related disorder. Displaying emotions following a traumatic incident can help prevent development of a trauma‑related disorder. (ATI ch 12)

A patient in the emergency department reports, "I hear voices saying someone is stalking me. They want to kill me because I found the cure for cancer. I will stab anyone that threatens me." Which aspects of mental health have the greatest immediate concern to a nurse? Select all that apply. a. Happiness b. Appraisal of reality c. Control over behavior d. Effectiveness in work e. Healthy self-concept

ANS: B, C, E The aspects of mental health of greatest concern are the patients appraisal of and control over behavior. The patients appraisal of reality is inaccurate, and auditory hallucinations are evident, as well as delusions of persecution and grandeur. In addition, the patients control over behavior is tenuous, as evidenced by the plan to stab anyone who seems threatening. A healthy self-concept is lacking. Data are not present to suggest that the other aspects of mental health (happiness and effectiveness in work) are of immediate concern.

A nurse assesses a patient who reluctantly participates in activities, answers questions with minimal responses, and rarely makes eye contact. What information should be included when documenting the assessment? Select all that apply. a. Uncooperative patient b. Patients subjective responses c. Only data obtained from the patients verbal responses d. Description of the patients behavior during the interview e. Analysis of why the patient is unresponsive during the interview

ANS: B, D Both the content and process of the interview should be documented. Providing only the patients verbal responses creates a skewed picture of the patient. Writing that the patient is uncooperative is subjectively worded. An objective description of patient behavior is preferable. Analysis of the reasons for the patients behavior is speculation, which is inappropriate.

A nurse if planning a peer group discussion about the Diagnostic and Statistical Manual of Mental Disorders, 5th Edition (DSM-5). Which of the following information is appropriate to include in the discussion? (select all that apply) a. The DSM-5 includes client education handouts for mental health disorders. b. The DSM-5 establishes diagnostic criteria for individual mental health disorders. c. The DSM-5 indicates recommended pharmacological treatment for mental health disorders. d. The DSM-5 assists nurses in planning care for client's who have mental health disorders. e. The DSM-5 indicates expected assessment findings of mental health disorders.

ANS: B, D, E The DSM-5 is used by mental health professionals however, it does not include client education handouts. The DSM-5 does not indicate pharmacological treatment for mental health disorders. (from ATI ch 1)

A nurse is caring for a client who is experiencing a crisis. Which of the following medications might the provider prescribe? (Select all that apply.) A. Lithium carbonate B. Paroxetine C. Risperidone D. Haloperidol E. Lorazepam

ANS: B, E SSRI antidepressants, such as paroxetine, may be prescribed to decrease the anxiety and depression of a client who is experiencing a crisis. Benzodiazepines, such as lorazepam, may be prescribed to decrease the anxiety of a client who is experiencing a crisis. Mood stabilizers, such as lithium carbonate, are prescribed for bipolar disorder and are not indicated in a short-term crisis situation. Antipsychotic medications, such as risperidone and haloperidol, may be prescribed for disturbed thought processes, usually when accompanied by other psychotic symptoms (hallucinations, delusions, blunt affect). Antipsychotics are not indicated in a short-term crisis situation (ATI ch 29)

A nurse is talking with a client who is at risk for suicide following their partner's death. Which of the following statements should the nurse make? a. "I feel very sorry for the loneliness you must be experiencing." b. "suicide is not the appropriate way to cope with loss." c. "Losing someone close to you must be very upsetting." d. "I know how difficult it is to lose a loved one."

ANS: C "Losing someone close to you must be very upsetting." is an empathetic response that attempts to understand the client's feelings."I feel very sorry for the loneliness you must be experiencing." focuses on the nurse's feelings and is sympathetic rather than empathetic. "Suicide is not the appropriate way to cope with loss." implies judgment and is therefore not an empathetic or therapeutic response. "I know how difficult it is to lose a loved one." focuses on the nurse's experiences rather than the client's and is therefore not therapeutic. (ATI ch 5)

After celebrating a 40th birthday, an individual becomes concerned with the loss of youthful appearance. What type of crisis has occurred? a. Reactive b. Situational c. Maturational d. Adventitious

ANS: C *Maturational* crises occur when a person arrives at a new stage of development and finds that old coping styles are ineffective but has not yet developed new strategies. Situational crises arise from sources external to the individual, such as divorce and job loss. No classification called reactive crisis exists. Adventitious crises occur when disasters such as natural disasters (e.g., floods, hurricanes), war, or violent crimes disrupt coping styles.

A community mental health nurse is planning care to address the issue of depression among older adult clients in the community. Which of the following interventions should the nurse implement as a method of tertiary prevention? a. Educating clients on health promotion techniques to reduce the risk of depression. b. Performing screenings for depression at community health programs c. Establishing rehab programs to decrease the effects of depression d. Providing support groups for clients at risk for depression

ANS: C A and D are primary interventions. B is a secondary intervention. C is a tertiary intervention as it deals with prevention of further problems in clients already diagnoses with mental illness. (ATI ch 6)

To provide comprehensive care to patients, which competency is more important for a nurse who works in a community mental health center than a psychiatric nurse who works in an inpatient unit? a. Problem-solving skills b. Calm and caring manner c. Ability to cross service systems d. Knowledge of psychopharmacology

ANS: C A community mental health nurse must be able to work with schools, corrections facilities, shelters, health care providers, and employers. The mental health nurse working in an inpatient unit needs only to be able to work within the single setting. Problem-solving skills are needed by all nurses. Nurses in both settings must have knowledge of psychopharmacology.

A nurse in an acute mental health facility is assisting with discharge planning for a client who has a severe mental illness and requires supervision. The client's partner works all day but is home by late afternoon. Which of the following strategies should the nurse suggest for follow-up care? a. Receiving daily care from a home health aid b. Having a weekly visit from a nurse case worker c. Attending a partial hospitalization program d. Visiting a community mental health center on a daily basis

ANS: C A partial hospitalization program can provide treatment during the day while allowing the client to spend nights at home, as long as a responsible family member is present (as this client's partner is). The other options will not provide consistent supervision for this client (ATI ch 6)

A woman says, "I can't take anymore! Last year my husband had an affair and now we don't communicate. Three months ago, I found a lump in my breast. Yesterday my daughter said she's quitting college." What type of crisis is this person experiencing? a. Maturational b. Adventitious c. Situational d. Recurring

ANS: C A situational crisis arises from an external source and involves a loss of self-concept or self- esteem. An adventitious crisis is a crisis of disaster, such as a natural disaster or crime of violence. Maturational crisis occurs as an individual arrives at a new stage of development, when old coping styles may be ineffective. No classification of recurring crisis exists.

As a nurse escorts a patient being discharged after treatment for major depressive disorder, the patient gives the nurse a gold necklace with a heart pendant and says, "Thank you for helping mend my broken heart." Which is the nurses best response? a. Accepting gifts violates the policies and procedures of the facility. b. I'm glad you feel so much better now. Thank you for the beautiful necklace. c. I'm glad I could help you, but I can't accept the gift. My reward is seeing you with a renewed sense of hope. d. Helping people is what nursing is all about. It's rewarding to me when patients recognize how hard we work.

ANS: C Accepting a gift creates a social rather than a therapeutic relationship with the patient and blurs the boundaries of the relationship. A caring nurse will acknowledge the patients gesture of appreciation, but the gift should not be accepted.

A patient discloses several concerns and associated feelings. If the nurse wants to seek clarification, which comment would be appropriate? a. What are the common elements here? b. Tell me again about your experiences. c. Am I correct in understanding that...? d. Tell me everything from the beginning.

ANS: C Asking, "Am I correct in understanding that...?" permits clarification to ensure that both the nurse and patient share mutual understanding of the communication. Asking about common elements encourages comparison rather than clarification. The remaining responses are implied questions that suggest the nurse was not listening.

A client tells a nurse, "Don't tell anyone, but I hid a sharp knife under my mattress in order to protect myself from my roommate, who is always threatening me." Which of the following actions should the nurse take? a. Keep the client's communication confidential, but talk to the client daily, using therapeutic communication to convince them to admit to hiding the knife. b. Keep the client's communication confidential but watch the client and their roommate closely. c. Tell the client that this must be reported to the health care team because it concerns the health and safety of the client and others. d. Report the incident to the health care team, but do not inform the client of the intention to do so.

ANS: C Based on the nature of the information, the nurse cannot keep the information confidential from everyone despite the client's request. *The information presented by the client is a serious safety issue that the nurse must report to the health care team. Using the ethical principle of veracity, the nurse tells the client truthfully what must be done regarding the issue.* (ATI ch 2)

A nurse must assess several new patients at a community mental health center. Conclusions concerning current functioning should be made on the basis of: a. the degree of conformity of the individual to societys norms. b. the degree to which an individual is logical and rational. c. a continuum from mentally healthy to unhealthy. d. the rate of intellectual and emotional growth.

ANS: C Because mental health and mental illness are relative concepts, assessment of functioning is made by using a continuum. Mental health is not based on conformity; some mentally healthy individuals do not conform to societys norms. Most individuals occasionally display illogical or irrational thinking. The rate of intellectual and emotional growth is not the most useful criterion to assess mental health or mental illness.

A patient tells the nurse, I dont think I will ever get out of here. Select the nurses most therapeutic response. a. Don't talk that way. Of course you will leave here! b. Keep up the good work and you certainly will. c. You don't think youre making progress? d. Everyone feels that way sometimes.

ANS: C By asking if the patient does not believe that progress has been made, the nurse is reflecting by putting into words what the patient is hinting. By making communication more explicit, issues are easier to identify and resolve. The remaining options are nontherapeutic techniques. Telling the patient not to talk that way is disapproving. Saying that everyone feels that way at times minimizes feelings. Telling the patient that good work will always result in success is falsely reassuring.

A cognitive strategy a nurse could use to assist a very dependent patient would be to help the patient: a. reveal dream content. b. take prescribed medications. c. examine thoughts about being autonomous. d. role model ways to ask for help from others.

ANS: C Cognitive theory suggests that ones thought processes are the basis of emotions and behavior. Changing faulty learning makes the development of new adaptive behaviors possible. Revealing dream content would be used in psychoanalytical therapy. Taking prescribed medications is an intervention associated with biological therapy. A dependent patient needs to develop independence.

During which phase of the nurse-patient relationship can the nurse anticipate that identified patient issues will be explored and resolved? a. Preorientation b. Orientation c. Working d. Termination

ANS: C During the working phase, the nurse strives to assist the patient in making connections among dysfunctional behaviors, thinking, and emotions and offers support while alternative coping behaviors are tried.

The parent of a 4-year-old rewards and praises the child for helping a younger sibling, being polite, and using good manners. A nurse supports the use of praise because, according to the Freudian theory, these qualities will likely be internalized and become part of the childs: a. id. b. ego. c. superego. d. preconscious.

ANS: C In the Freudian theory, the superego contains the thou shalts or moral standards internalized from interactions with significant others. Praise fosters internalization of desirable behaviors. The id is the center of basic instinctual drives, and the ego is the mediator. The ego is the problem-solving and reality-testing portion of the personality that negotiates solutions with the outside world. The preconscious is a level of awareness from which material can be easily retrieved with conscious effort.

The nurse presents a class about mental health and mental illness to a group of fourth graders. One student asks, "Why do people get mentally ill?" Select the nurse's best response. a. "There are many reasons why mental illness occurs." b. "The cause of mental illness is complicated and very hard to understand." c. "Sometimes a person's brain does not work correctly because something bad happens or they inherit a brain problem." d. "Most mental illnesses result from genetically transmitted abnormalities in cerebral structure; however, some are a consequence of traumatic life experiences."

ANS: C In the correct response, the nurse answers rather than evades the question, provides accurate information, and uses terminology a 9- or 10-year-old child can understand. Many of the most prevalent and disabling mental disorders have been found to have strong biological influences, including genetic transmission (chapter 2, pg 13-14)

A patient should be considered for involuntary commitment for psychiatric care when he or she: a. is noncompliant with the treatment regimen. b. sells and distributes illegal drugs. c. threatens to harm self and others. d. fraudulently files for bankruptcy.

ANS: C Involuntary commitment protects patients who are dangerous to themselves or others and cannot care for their own basic needs. Involuntary commitment also protects other individuals in society. The behaviors described in the other options are not sufficient to require involuntary hospitalization.

When a new patient is hospitalized, a nurse takes the patient on a tour, explains the rules of the unit, and discusses the daily schedule. The nurse is engaged in: a. counseling. b. health teaching. c. milieu management. d. psychobiologic intervention.

ANS: C Milieu management provides a therapeutic environment in which the patient can feel comfortable and safe while engaging in activities that meet the patients physical and mental health needs. Counseling refers to activities designed to promote problem solving and enhanced coping and includes interviewing, crisis intervention, stress management, and conflict resolution. Health teaching involves identifying health educational needs and giving information about these needs. Psychobiologic interventions involve medication administration and monitoring response to medications.

A patient says, "I'm still on restriction, but I want to attend some off-unit activities. Would you ask the doctor to change my privileges?" What is the nurses best response? a. Why are you asking me when youre able to speak for yourself? b. I will be glad to address it when I see your doctor later today. c. That's a good topic for you to take up with your doctor. d. Do you think you cant speak to a doctor?

ANS: C Nurses should encourage patients to work at their optimal level of functioning. A nurse does not act for the patient unless it is necessary. Acting for a patient increases feelings of helplessness and dependency.

Operant conditioning will be used to encourage speech in a child who is nearly mute. Which technique would a nurse include in the treatment plan? a. Ignore the child for using silence. b. Have the child observe others talking. c. Give the child a small treat for speaking. d. Teach the child relaxation techniques, then coax speech.

ANS: C Operant conditioning involves giving positive reinforcement for a desired behavior. Treats are rewards to reinforce speech. Ignoring the child will not change the behavior. Having the child observe others describes modeling. Teaching relaxation techniques and then coaxing speech is an example of systematic desensitization.

Two hospitalized patients fight when they are in the same room. During a team meeting, a nurse asserts that safety is of paramount importance and therefore the treatment plans should call for both patients to be secluded to prevent them from injuring each other. This assertion: a. reveals that the nurse values the principle of justice. b. reinforces the autonomy of the two patients. c. violates the civil rights of the two patients. d. represents the intentional tort of battery.

ANS: C Patients have a right to treatment in the least restrictive setting. Less restrictive measures should be tried first. Unnecessary seclusion may result in a charge of false imprisonment. Seclusion removes the patients autonomy. The principle by which the nurse is motivated is beneficence, not justice. The tort represented is false imprisonment, not battery.

A new staff nurse completes orientation to the psychiatric unit. This nurse will expect to ask an advanced practice nurse to perform which action for patients? a. Perform mental health assessment interviews. b. Establish therapeutic relationships. c. Prescribe psychotropic medications. d. Individualize nursing care plans.

ANS: C Prescriptive privileges are granted to Masters-prepared nurse practitioners who have taken special courses on prescribing medications. The nurse prepared at the basic level performs mental health assessments, establishes relationships, and provides individualized care planning.

An adult plans to attend an upcoming tenth high school reunion. This person says to the nurse, "I am embarrassed to go. I will not look as good as my classmates. I haven't been successful in my career." Which comment by the nurse addresses this cognitive distortion? a. "You look fine to me. Do think you will have fun at your reunion?" b. "Everyone ages. Other classmates have had more problems than you." c. "Do you think you are the only person who has aged and faced difficulties in life?" d. "I think you are doing well in the face of the numerous problems you have endured."

ANS: C Rapid, unthinking responses are known as automatic thoughts. Often these automatic thoughts, or cognitive distortions, are irrational because people make false assumptions and misinterpretations. Once the negative patterns of thought that lead to negative emotions are identified, they can be replaced with rational thoughts (chapter 3, pg 25 (Table 3-3))

Which issues should a nurse address during the first interview with a patient diagnosed with a psychiatric disorder? a. Trust, congruence, attitudes, and boundaries b. Goals, resistance, unconscious motivations, and diversion c. Relationship parameters, the contract, confidentiality, and termination d. Transference, countertransference, intimacy, and developing resources

ANS: C Relationship parameters, the contract, confidentiality, and termination are issues that should be considered during the orientation phase of the relationship. The remaining options are issues that are dealt with later.

A newly admitted patient with major depressive disorder has lost 20 pounds over the past month and has suicidal ideation. The patient has taken an antidepressant medication for 1 week without remission of symptoms. Select the priority nursing diagnosis. a. Imbalanced nutrition: Less than body requirements b. Chronic low self-esteem c. Risk for suicide d. Hopelessness

ANS: C Risk for suicide is the priority diagnosis when the patient has both suicidal ideation and a plan to carry out the suicidal intent. Imbalanced nutrition, Hopelessness, and Chronic low self-esteem may be applicable nursing diagnoses, but these problems do not affect patient safety as urgently as a suicide attempt.

Which principle should guide the nurse in determining the extent of silence to use during patient interview sessions? a. Nurses are responsible for breaking silences. b. Patients withdraw if silences are prolonged. c. Silence can provide meaningful moments for reflection. d. Silence helps patients know that what they said is understood.

ANS: C Silence can be helpful to both participants by giving each an opportunity to contemplate what has transpired, weigh alternatives, and formulate ideas. A nurse breaking silences is not a principle related to silences. Saying that patients withdraw during long silences or that silence helps patients know that they are understood are both inaccurate statements. Feedback helps patients know they have been understood.

A nurse is caring for a client who has derealization disorder. Which of the following findings should the nurse identify as an indication of derealization? A. The client explains that her body seems to be floating above the ground. B. The client has the idea that someone is trying to kill her and steal her money. C. The client states that the furniture in the room seems to be small and far away. D. The client cannot recall anything that happened during the past 2 weeks.

ANS: C Stating that one's surroundings are far away or unreal in some way is an example of derealization.Feeling that one's body is floating above the ground is an example of depersonalization, in which the person seems to observe her own body from a distance. Having the idea that others are trying to hurt or kill her is an example of a paranoid delusion. Being unable to recall any events from the past 2 weeks is an example of amnesia. (ATI ch 12)

A mentally ill gunman opens fire in a crowded movie theater, killing six people and injuring others. Which comment about this event by a member of the community most clearly shows the stigma of mental illness? a. "Gun control laws are inadequate in our country." b. "It's frightening to feel that it is not safe to go to a movie theater." c. "All these people with mental illness are violent and should be locked up." d. "These events happen because American amilies no longer go to church together."

ANS: C Stigma refers to the array of negative attitudes and beliefs regarding mental illness. Bias, prejudice, fear, and misinformation contribute to stigma (chapter 2, pg 8-9)

After several therapeutic encounters with a patient who recently attempted suicide, which occurrence should cause the nurse to consider the possibility of countertransference? a. The patients reactions toward the nurse seem realistic and appropriate. b. The patient states, "Talking to you feels like talking to my parents." c. The nurse feels unusually happy when the patients mood begins to lift. d. The nurse develops a trusting relationship with the patient.

ANS: C Strong positive or negative reactions toward a patient or an overidentification with a patient signals possible countertransference. Nurses must carefully monitor their own feelings and reactions to detect countertransference and then seek supervision. Realistic and appropriate reactions from a patient toward a nurse are desirable. One incorrect response suggests transference. A trusting relationship with the patient is desirable.

A patient hurriedly tells the community mental health nurse, "Everything's a disaster! I can't concentrate. My disability check didn't come. My roommate moved out, and I cant afford the rent. My therapist is moving away. I feel like Im coming apart." Which nursing diagnosis applies? a. Decisional conflict, related to challenges to personal values b. Spiritual distress, related to ethical implications of treatment regimen c. Anxiety, related to changes perceived as threatening to psychological equilibrium d. Impaired environmental interpretation syndrome, related to solving multiple problems affecting security needs

ANS: C Subjective and objective data obtained by the nurse suggest the patient is experiencing anxiety caused by multiple threats to security needs. Data are not present to suggest Decisional conflict, Spiritual distress caused by ethical conflicts, or Impaired environmental interpretation syndrome.

Termination of a therapeutic nurse-patient relationship with a patient has been successful when the nurse: a. avoids upsetting the patient by shifting focus to other patients before the discharge. b. gives the patient a personal telephone number and permission to call after discharge. c. discusses with the patient changes that have happened during the relationship and evaluates the outcomes. d. offers to meet the patient for coffee and conversation three times a week after discharge.

ANS: C Summarizing and evaluating progress help validate the experience for the patient and the nurse and facilitate closure. Termination must be discussed; avoiding the discussion by spending little time with the patient promotes feelings of abandonment. Successful termination requires that the relationship be brought to closure without the possibility of dependency-producing ongoing contact.

A person received an invitation to be in the wedding of a friend who lives across the country. The individual is afraid of flying. What type of therapy should the nurse recommend? a. Psychoanalysis b. Milieu therapy c. Systematic desensitization d. Short-term dynamic therapy

ANS: C Systematic desensitization is a type of therapy aimed at extinguishing a specific behavior, such as the fear of flying. Psychoanalysis and short-term dynamic therapy are aimed at uncovering conflicts. Milieu therapy involves environmental factors.

A person with a fear of heights drives across a high bridge. Which division of the autonomic nervous system is stimulated in response to this experience? a. Limbic system b. Peripheral nervous system c. Sympathetic nervous system d. Parasympathetic nervous system

ANS: C The autonomic nervous system is made up of the sympathetic (fight-or-flight response) and parasympathetic (relaxation response) nervous systems. In times of stress, the sympathetic nervous system is stimulated. A person fearful of heights would experience stress associated with the experience of driving across a high bridge. The peripheral nervous system responds to messages from the sympathetic nervous system. The limbic system processes emotional responses but is not specifically part of the autonomic nervous system.

A patient is hospitalized for a reaction to a psychotropic medication and then is closely monitored for 24 hours. During a predischarge visit, the case manager learns the patient received a notice of eviction on the day of admission. The most appropriate intervention for the case manager is to: a. cancel the patients discharge from the hospital. b. contact the landlord who evicted the patient to discuss the situation. c. arrange a temporary place for the patient to stay until new housing can be arranged. d. document that the adverse medication reaction was feigned because the patient had nowhere to live.

ANS: C The case manager should intervene by arranging temporary shelter for the patient until suitable housing can be found. This is part of the coordination and delivery of services that falls under the case manager role. The other options are not viable alternatives.

A patient visiting the crisis clinic for the first time asks, "How long will I be coming here?" The nurses reply should consider that the usual duration of crisis intervention is _______ weeks. a. 1 to 2 b. 3 to 4 c. 4 to 6 d. 6 to 12

ANS: C The disorganization associated with crisis is so distressing that it usually cannot be tolerated for more than 4 to 6 weeks. If the crisis is not resolved by that time, the individual usually adopts dysfunctional behaviors that reduce anxiety without solving the problem. Crisis intervention can shorten the duration.

Which patient is likely to achieve maximum benefit from cognitive behavioral therapy (CBT)? a. Older adult diagnosed with stage 3 Alzheimer's disease b. Adult diagnosed with schizophrenia and experiencing delusions c. Adult experiencing feelings of failure after losing the fourth job in 2 years d. School-age child diagnosed with attention deficit/ hyperactivity disorder (ADHD)

ANS: C The goal of cognitive behavioral therapy (CBT) is to identify the negative patterns of thought that lead to negative emotions. Once the maladaptive patterns are identified, they can be replaced with rational thoughts. A person must be able to engage in meaningful dialogue to benefit from CBT (chapter 3, pg 25)

A person with a fear of heights drives across a high bridge. Which structure will stimulate a response from the autonomic nervous system? a. Thalamus b. Parietal lobe c. Hypothalamus d. Pituitary gland

ANS: C The individual will find this experience stressful. The hypothalamus functions as the command-and-control center when receiving stressful signals. The hypothalamus responds to signals of stress by engaging the autonomic nervous system. The parietal lobe is responsible for the interpretation of other sensations. The thalamus processes messages associated with pain and wakefulness. The pituitary gland may be involved in other aspects of the persons response but would not stimulate the autonomic nervous system.

A nurse is planning care for several clients who are attending community-based mental health programs. Which of the following clients should the nurse visit first? a. A client who received a burn on the arm while using a hot iron at home. b. A client who request a change of antipsychotic medication due to some new adverse effects. c. A client who reports hearing a voice saying that life is not worth living anymore. d. A client who tells the nurse about experiencing manifestations of severe anxiety before and during a job interview.

ANS: C The key word is "first." All the client need to be seen but the priority is the client who poses a threat to himself. (ATI ch 6)

A nurse works with a patient to establish outcomes. The nurse believes that one outcome suggested by the patient is not in the patients best interest. What is the nurses best action? a. Remain silent. b. Educate the patient that the outcome is not realistic. c. Explore with the patient possible consequences of the outcome. d. Formulate a more appropriate outcome without the patients input.

ANS: C The nurse should not impose outcomes on the patient; however, the nurse has a responsibility to help the patient evaluate what is in his or her best interest. Exploring possible consequences is an acceptable approach.

A nurse caring for a client who has anorexia nervosa. Which of the following examples demonstrates the nurse's use of interpersonal communication? a. The nurse discusses the client's weight loss during a health care team meeting b. The nurse examines their own personal feelings about client who have anorexia nervosa c. The nurse asks the client about personal body image perception d. The nurse presents an educational session about anorexia nervosa to a large group of adolescents

ANS: C The nurse's one‑on‑one communication with the client is an example of interpersonal communication.The nurse's discussion of client information with members of the healthcare team is an example of small‑group communication. The nurse's self‑assessment of feelings is an example of intrapersonal communication. The nurse's educational presentation to a large group of adolescents is an example of public communication. (ATI ch 3)

An adolescent asks a nurse conducting an assessment interview, "Why should I tell you anything? You'll just tell my parents whatever you find out." Select the nurses best reply. a. That is not true. What you tell us is private and held in strict confidence. Your parents have no right to know. b. Yes, your parents may find out what you say, but it is important that they know about your problems. c. What you say about feelings is private, but some things, like suicidal thinking, must be reported to the treatment team. d. It sounds as though you are not really ready to work on your problems and make changes.

ANS: C The patient has a right to know that most information will be held in confidence but that certain material must be reported or shared with the treatment team, such as threats of suicide, homicide, use of illegal drugs, or issues of abuse. The first response is not strictly true. The second response will not inspire the confidence of the patient. The fourth response is confrontational.

A student nurse prepares to administer oral medications to a patient diagnosed with major depressive disorder, but the patient refuses the medication. The student nurse should: a. tell the patient, "I'll get an unsatisfactory grade if I don't give you the medication." b. tell the patient, "Refusing your medication is not permitted. You are required to take it." c. discuss the patients concerns about the medication, and report to the staff nurse. d. document the patients refusal of the medication without further comment.

ANS: C The patient has the right to refuse medication in most cases. The patients reason for refusing should be ascertained, and the refusal should be reported to a unit nurse. Sometimes refusals are based on unpleasant side effects that can be ameliorated. Threats and manipulation are inappropriate. Medication refusal should be reported to permit appropriate intervention.

The goal for a patient is to increase resiliency. Which outcome should a nurse add to the plan of care? Within 3 days, the patient will: a. describe feelings associated with loss and stress. b. meet own needs without considering the rights of others. c. identify healthy coping behaviors in response to stressful events. d. allow others to assume responsibility for major areas of own life.

ANS: C The patients ability to identify healthy coping behaviors indicates adaptive, healthy behavior and demonstrates an increased ability to recover from severe stress. Describing feelings associated with loss and stress does not move the patient toward adaptation. The remaining options are maladaptive behaviors.

A nurse in an emergency mental health facility is caring for a group of clients. The nurse should identify that which of the following clients requires a temporary emergency admission? a. A client who has schizophrenia with delusions of grandeur. b. A client who has manifestations of depression and attempted suicide a year ago. c. A client who has borderline personality disorder and assaulted a homeless man with a metal rod. d. A client who has bipolar disorder and paces quickly around the room while talking to themselves.

ANS: C The presence of delusions does not constitute a clear reason for emergency admission unless they present a danger for the client or others. Clinical findings of depression do not constitute a clear reason for a temporary emergency admission unless the client is currently at risk for suicide. *A client who is a current danger to self or others is a candidate for a temporary emergency admission.* The presence of pacing does not constitute a clear reason for a temporary emergency admission. (ATI ch 2)

A patient has been disruptive to the therapeutic milieu or two days. A certified nursing assistant says to the nurse, "We need to seclude this patient because this behavior is upsetting everyone on the unit." Considering patients' rights, the nurse should respond, a. "Seclusion is not part of this patient's plan of care." b. "Let's think of some new ways to help this patient be less disruptive." c. "Thank you or that suggestion. I will discuss it with the health care provider." d. "Disruptive behavior is expected with mental illness. We must respond therapeutically."

ANS: C The scenario offers no indication that the patient is dangerous or out of control; therefore less restrictive interventions should be employed. The nurse has a responsibility to provide guidance to the CNA (chapter 6, page 65-66)

2. An adult has had long-term serious medical problems resulting in decreased libido and sexual per ormance. The adult's spouse privately says to the nurse, "I don't feel loved anymore. I feel sexual urges but my partner is not interested." Select the nurse's therapeutic response. a. "Tell me about how your partner shows love for you." b. "You're describing a scenario that many couples face." c. "Let's consider some other ways you can satisfy your needs." d. "I'm glad you are able to talk about and accept your situation."

ANS: C The scenario presents a maturational crisis. Helping the spouse to consider other options is the nurse's most therapeutic action (chapter 20, pg 326)

A patient comments, "I never know the right answer" and "My opinion is not important." Using Erikson's theory, which psychosocial crisis did the patient have difficulty resolving? a. Initiative versus Guilt b. Trust versus Mistrust c. Autonomy versus Shame and Doubt d. Generativity versus Self-Absorption

ANS: C These statements show severe self-doubt, indicating that the crisis of gaining control over the environment is not being successfully met. Unsuccessful resolution of the crisis of Initiative versus Guilt results in feelings of guilt. Unsuccessful resolution of the crisis of Trust versus Mistrust results in poor interpersonal relationships and suspicion of others. Unsuccessful resolution of the crisis of Generativity versus Self-Absorption results in self- absorption that limits the ability to grow as a person.

An adult tells the nurse, "I can't take anymore! Last year my husband had an affair and now we don't communicate. Three months ago, I found a lump in my breast. Yesterday my daughter told me she's quitting college and moving in with her boyfriend." What is the priority nursing diagnosis? a. Fear, related to impending breast surgery b. Deficient knowledge, related to breast lesion c. Ineffective coping, related to perceived loss of daughter d. Impaired verbal communication, related to spousal estrangement

ANS: C This nursing diagnosis is the priority because it reflects the precipitating event associated with the patients crisis. Data are not present to make the other diagnoses of Deficient knowledge, Fear, or Impaired verbal communication.

Which individual with a mental illness may need emergency or involuntary hospitalization for mental illness? The individual who: a. resumes using heroin while still taking methadone. b. reports hearing angels playing harps during thunderstorms. c. throws a heavy plate at a waiter at the direction of command hallucinations. d. does not show up for an outpatient appointment with the mental health nurse.

ANS: C Throwing a heavy plate is likely to harm the waiter and is evidence of being dangerous to others. This behavior meets the criteria for emergency or involuntary hospitalization for mental illness. The behaviors in the other options evidence mental illness but not dangerousness.

The nurse interacts with a veteran of World War II. The veteran says, "Veterans of modern wars whine and complain all the time. Back when I was in service, you kept your feelings to yourself." Select the nurse's best response. a. "American society in the 1940s expected World War II soldiers to be strong." b. "World War II was fought in a traditional way but the enemy is more difficult to identify in today's wars." c. "We now have a better understanding of how trauma affects people and the importance of research-based, compassionate care." d. "Intermittent explosive devices (IEDs), which were not in use during World War II, produce traumatic brain injuries that must be treated."

ANS: C Trauma occurs in many forms, including physical, sexual, and emotional abuse; war; natural disasters; and other harmful experiences. Trauma-informed care provides guidelines or integrating an understanding of how trauma affects patients into clinical programming (chapter 1, pg 5-6)

Documentation in a patients chart shows, "Throughout a 5-minute interaction, patient fidgeted and tapped left foot, periodically covered face with hands, and looked under chair while stating, 'I enjoy spending time with you.'" Which analysis is most accurate? a. Patient is giving positive feedback about the nurses communication techniques. b. Nurse is viewing the patients behavior through a cultural filter. c. Patients verbal and nonverbal messages are incongruent. d. Patient is demonstrating psychotic behaviors.

ANS: C When a verbal message is not reinforced with nonverbal behavior, the message is confusing and incongruent. Some clinicians call it a mixed message. It is inaccurate to say that the patient is giving positive feedback about the nurses communication techniques. The concept of a cultural filter is not relevant to the situation; a cultural filter determines what a person will pay attention to and what he or she will ignore. Data are insufficient to draw the conclusion that the patient is demonstrating psychotic behaviors.

A charge nurse is discussing the characteristics of a nurse-client relationship with a newly licensed nurse. Which of the following characteristics should the nurse include in the discussion? (select all that apply) a. The needs of both participants are met b. An emotional commitment exists between the participants c. It is goal-directed d. Behavioral change is encouraged e. A termination date is established

ANS: C, D, E A therapeutic nurse-client relationship is goal-directed, encourages positive behavioral change, and has an established termination date.A therapeutic nurse-client relationship focuses on the needs of the client, not on the nurse. An emotional commitment between the participants is characteristic of an intimate or social relationship rather than one that is therapeutic. (ATI ch 5)

Which experiences are most likely to precipitate post-traumatic stress disorder (PTSD)? Select all that apply. a. An 8-year-old child watches an R-rated movie with both parents. b. A young adult jumps from a bridge with a bungee cord with a best friend. c. An adolescent is kidnapped and held for 2 years in the home of a sexual predator. d. A passenger is in a bus that overturns on a sharp curve in the road, tumbling down an embankment. e. An adult is trapped for 3 hours at an angle in an elevator after a portion of the supporting cable breaks.

ANS: C, D, E PTSD usually follows a traumatic event that is outside the range of usual experience. Examples are childhood physical abuse, torture or kidnapping, military combat, sexual assault, and natural disasters such as floods, tornados, earthquakes, and tsunamis; human disasters such as a bus or elevator accident or crime-related events such being taken hostage are additional examples. The common element in these experiences is the individuals extraordinary helplessness or powerlessness in the face of such stressors. Bungee jumping by adolescents is part of the developmental task and might be frightening but in an exhilarating way rather than a harmful way. A child may be disturbed by an R-rated movie, but the presence of the parents would modify the experience in a positive way.

A nurse volunteers for a committee that must revise the hospital policies and procedures for suicide precautions. Which resources would provide the best guidance? Select all that apply. a. Diagnostic and Statistical Manual of Mental Disorders (fifth edition) (DSM-5) b. States nurse practice act c. State and federal regulations that govern hospitals d. Summary of common practices of several local hospitals e. American Nurses Association Scope and Standards of Practice for PsychiatricMental Health Nursing

ANS: C, E Regulations regarding hospitals provide information about the minimal standard. The American Nurses Association (ANA) national standards focus on elevating practice by setting high standards for nursing practice. The DSM-5 and the states nurse practice act would not provide relevant information. A summary of common practices of several local hospitals cannot be guaranteed to be helpful because the customs may or may not comply with laws or best practices.

Which behavior shows that a nurse values autonomy? The nurse: a. sets limits on a patients romantic overtures toward the nurse. b. suggests one-on-one supervision for a patient who is suicidal. c. informs a patient that the spouse will not be in during visiting hours. d. discusses available alternatives and helps the patient weigh the consequences.

ANS: D A high level of valuing is acting on ones belief. Autonomy is supported when the nurse helps a patient weigh alternatives and their consequences before the patient makes a decision. Autonomy or self-determination is not the issue in any of the other behaviors.

Which belief by a nurse supports the highest degree of patient advocacy during a multidisciplinary patient care planning session? a. All mental illnesses are culturally determined. b. Schizophrenia and bipolar disorder are cross-cultural disorders. c. Symptoms of mental disorders are constant from culture to culture. d. Some symptoms of mental disorders may reflect a persons cultural patterns.

ANS: D A nurse who understands that a patients symptoms are influenced by culture will be able to advocate for the patient to a greater degree than a nurse who believes that culture is of little relevance. All mental illnesses are not culturally determined. Schizophrenia and bipolar disorder are cross-cultural disorders, but this understanding has little relevance to patient advocacy. Symptoms of mental disorders change from culture to culture.

Which scenario is an example of a tort? a. The primary nurse does not complete the plan of care for a patient within 24 hours of the patients admission. b. An advanced practice nurse recommends that a patient who is dangerous to self and others be voluntarily hospitalized. c. A patients admission status is changed from involuntary to voluntary after the patients hallucinations subside. d. A nurse gives an as-needed dose of an antipsychotic drug to a patient to prevent violence because a unit is short staffed.

ANS: D A tort is a civil wrong against a person that violates his or her rights. Giving unnecessary medication for the convenience of staff members controls behavior in a manner similar to secluding a patient; thus false imprisonment is a possible charge. The other options do not exemplify torts.

An adult expresses the wish to be taken care of and often behaves in a helpless fashion. This adult has needs related to which of Freuds stages of psychosexual development? a. Latency b. Phallic c. Anal d. Oral

ANS: D According to Freud, fixation at the oral stage sometimes produces dependent infantile behaviors in adults. Latency fixations often result in a difficulty identifying with others and developing social skills, resulting in a sense of inadequacy and inferiority. Phallic fixations result in having difficulty with authority figures and poor sexual identity. Anal fixation sometimes results in retentiveness, rigidity, messiness, destructiveness, and cruelty.

The following patients are seen in the emergency department. The psychiatric unit has one bed available. Which patient should the admitting officer recommend for admission to the hospital? The patient who: a. is experiencing dry mouth and tremor related to side effects of haloperidol (Haldol). b. is experiencing anxiety and a sad mood after a separation from a spouse of 10 years. c. self-inflicted a superficial cut on the forearm after a family argument. d. is a single parent and hears voices saying, "Smother your infant."

ANS: D Admission to the hospital would be justified by the risk of patient danger to self or others. The other patients have issues that can be handled with less restrictive alternatives than hospitalization.

A patient on an acute psychiatric unit removed the cap from the ceiling sprinkler, resulting in rapid flooding of the unit. After moving patients to a safe area, which action should the nurse take next? a. Conduct individual sessions with patients regarding the experience. b. Increase the volume of overhead music to distract patients from the event. c. Implement a psychomotor activity to reduce anxiety associated with the event. d. Lead a group session with patients to discuss feelings associated with the event.

ANS: D After addressing safety concerns, the nurse should take steps to help patients feel safe and lower anxiety, such as providing a quiet environment, building rapport, and acknowledging their crisis experience. A group session will allow patients who are unable to articulate their feelings to hear from patients who are able to discuss it. (chapter 20, pg 330 (Table 20-2))

Which health care worker should be referred to critical incident stress debriefing? a. Nurse who works at an oncology clinic where patients receive chemotherapy b. Case manager whose patients are seriously mentally ill and are being cared for at home c. Health care employee who worked 8 hours at the information desk of an intensive care unit d. Emergency medical technician (EMT) who treated victims of a car bombing at a department store

ANS: D Although each of the individuals mentioned experiences job-related stress on a daily basis, the person most in need of critical incident stress debriefing is the EMT, who experienced an adventitious crisis event by responding to a bombing and provided care to victims of trauma.

A newly admitted patient who is acutely psychotic is a private patient of the senior psychiatrist. To whom does the psychiatric nurse who is assigned to this patient owe the duty of care? a. Health care provider b. Profession c. Hospital d. Patient

ANS: D Although the nurse is accountable to the health care provider, the agency, the patient, and the profession, the duty of care is owed to the patient.

A patients nursing diagnosis is Insomnia. The desired outcome is: Patient will sleep for a minimum of 5 hours nightly by October 31. On November 1, a review of the sleep data shows the patient sleeps an average of 4 hours nightly and takes a 2-hour afternoon nap. Which evaluation should be documented? a. Consistently demonstrated b. Often demonstrated c. Sometimes demonstrated d. Never demonstrated

ANS: D Although the patient is sleeping 6 hours daily, the total is not in one uninterrupted session at night. Therefore the outcome must be evaluated as never demonstrated.

A nurse is planning care for a client who has a mental health disorder. Which of the following actions should the nurse include as a psychobiological intervention? a. Assist the client with systematic desensitization therapy. b. Teach the client appropriate coping mechanisms. c. Assess the client for comorbid health conditions. d. Monitor the client for adverse effects of medications.

ANS: D Assisting with systematic desensitization therapy is cognitive and behavioral. Teaching appropriate coping mechanisms is counseling or health teaching. Assessing for comorbid health conditions is health promotion and maintenance (from ATI ch 1)

A nurse assesses a newly admitted patient diagnosed with major depressive disorder. Which statement is an example of attending? a. We all have stress in life. Being in a psychiatric hospital isnt the end of the world. b. Tell me why you felt you had to be hospitalized to receive treatment for your depression. c. You will feel better after we get some antidepressant medication started for you. d. I'd like to sit with you a while so you may feel more comfortable talking with me.

ANS: D Attending is a technique that demonstrates the nurses commitment to the relationship and reduces feelings of isolation. This technique shows respect for the patient and demonstrates caring. Generalizations, probing, and false reassurances are non-therapeutic.

A nurse listens to a group of recent retirees. One says, "I volunteer with Meals on Wheels, coach teen sports, and do church visitation." Another laughs and says, "I'm too busy taking care of myself to volunteer. I don't have time to help others." These comments contrast which developmental tasks? a. Trust versus Mistrust b. Industry versus Inferiority c. Intimacy versus Isolation d. Generativity versus Self-Absorption

ANS: D Both retirees are in middle adulthood, when the developmental crisis to be resolved is Generativity versus Self-Absorption. One exemplifies generativity; the other embodies self- absorption. The developmental crisis of Trust versus Mistrust would show a contrast between relating to others in a trusting fashion and being suspicious and lacking trust. Failure to negotiate the developmental crisis of Industry versus Inferiority would result in a sense of inferiority or difficulty learning and working as opposed to the ability to work competently. Behaviors that would be contrasted in the crisis of Intimacy versus Isolation would be emotional isolation and the ability to love and commit to oneself.

An adolescent hospitalized after a violent physical outburst tells the nurse, Im going to kill my father, but you cant tell anyone. Select the nurses best response. a. Youre right. Federal law requires me to keep that information private. b. Those kinds of thoughts will make your hospitalization longer. c. You really should share this thought with your psychiatrist. d. I am required to share information with the treatment team.

ANS: D Breach of nurse-patient confidentiality does not pose a legal dilemma for the nurse in this circumstance because a team approach to the delivery of psychiatric care presumes communication of patient information to other staff members to develop treatment plans and outcome criteria. The patient should know that the team may have to warn the father of the risk for harm.

An informal group of patients discuss their perceptions of nursing care. Which comment best indicates a patients perception that his or her nurse is caring? a. My nurse always asks me which type of juice I want to help me swallow my medication. b. My nurse explained my treatment plan to me and asked for my ideas about how to make it better. c. My nurse told me that if I take all the medicines the doctor prescribes I will get discharged soon. d. My nurse spends time listening to me talk about my problems. That helps me feel like I'm not alone.

ANS: D Caring evidences empathic understanding, as well as competency. It helps change pain and suffering into a shared experience, creating a human connection that alleviates feelings of isolation. The incorrect options give examples of statements that demonstrate advocacy or giving advice.

A troubled adolescent opened fire in a high school cafeteria, fatally shooting three people and injuring many others. Hundreds of parents come to the high school after hearing the news reports. After the police arrest the shooter, which action should occur next? a. Ask the police to encircle the school campus with yellow tape to prevent parents from entering. b. Announce over the loudspeakers, "The campus is now secure. Please return to your classrooms." c. Require parents to pass through metal detectors and then allow them to look for their children in the school. d. Designate zones according to the alphabet, and direct students to the zones based on their surnames to facilitate reuniting them with their parents.

ANS: D Chaos is likely among students and desperate parents. A directive approach is best. Once the scene is secure, creative solutions are needed. Creating zones by letters of the alphabet helps anxious parents and their children to unite. Preventing parents from uniting with their children would further incite the situation.

A single parent is experiencing feelings of inadequacy related to work and family since one teenaged child ran away several weeks ago. The parent seeks the help of a therapist specializing in cognitive therapy. The psychotherapist who uses cognitive therapy will treat the patient by: a. discussing ego states. b. focusing on unconscious mental processes. c. negatively reinforcing an undesirable behavior. d. helping the patient identify and change faulty thinking.

ANS: D Cognitive therapy emphasizes the importance of changing erroneous ways people think about themselves. Once faulty thinking changes, the individuals behavior changes. Focusing on unconscious mental processes is a psychoanalytic approach. Negatively reinforcing undesirable behaviors is behavior modification, and discussing ego states relates to transactional analysis.

A nurse's sibling happily says,"I want to introduce you to my fiancé. We're getting married in six months." The nurse has encountered the fiancé in a clinical setting and is aware of the fiancé's diagnosis of schizophrenia. What is the nurse's best response? a. In private, tell the sibling about the fiancé's diagnosis. b. Encourage the sibling to postpone the wedding or at least a year . c. Ask the fiancé, "Have you told my sibling about your mental illness?" d. Say to the sibling and fiancé, "I hope you will be very happy together."

ANS: D Despite personal misgivings, the nurse must maintain the fiancé's confidentiality (chapter 6, page 66)

A nurse is discussing free association as a therapeutic tool with a client who has major depressive disorder. Which of the following client statements indicated understanding of this technique? a. "I will write down my dreams as soon as I wake up." b. "I might begin to associate my therapist with important people in my life." c. "I can learn to express myself in a nonaggressive manner." d. "I should say the first thing that comes to my mind."

ANS: D Dream analysis and interpretation are therapeutic tools. However, they are not an example of free association. Associating the therapist with significant persons in the client's life is an example of transference. Learning to express feelings and solve problems in a nonaggressive manner is an example of assertiveness training. Free association is spontaneous, uncensored verbalization of whatever comes to a client's mind. (from ATI ch 7)

A woman says, "I can't take anymore! Last year my husband had an affair and now we don't communicate. Three months ago, I found a lump in my breast. Yesterday my daughter said she's quitting college." What is the nurses priority assessment? a. Identifying measures useful to help improve the couples communication b. Discussing the patients feelings about the possibility of having a mastectomy c. Determining whether the husband is still engaged in an extramarital affair d. Clarifying what the patient means by "I can't take it anymore!"

ANS: D During crisis intervention, the priority concern is patient safety. This question helps assess personal coping skills. The other options are incorrect because the focus of crisis intervention is on the event that occurred immediately before the patient sought help.

A soldier returned 3 months ago from Afghanistan and was diagnosed with post-traumatic stress disorder (PTSD). Which social event would most likely be disturbing for this soldier? a. Halloween festival with neighborhood children b. Singing carols around a Christmas tree c. Family outing to the seashore d. Fireworks display on July 4th

ANS: D Exploding noises associated with fireworks are most likely to provoke exaggerated responses for this soldier. The other distractors are not associated with offensive sounds.

A nurse in an acute mental health facility is planning care for a client who has dissociative fugue. Which of the following interventions should the nurse add to the plan of care? A. Teach the client to recognize how stress brings on a personality change in the client. B. Repeatedly present the client with information about past events. C. Make decisions for the client regarding routine daily activities. D. Work with the client on grounding techniques.

ANS: D Grounding techniques, such as stomping the feet, clapping the hands, or touching physical objects, are useful for clients who have a dissociative disorder and are experiencing manifestations of derealization.The client who has dissociative identity disorder displays multiple personalities, while the client who has dissociative fugue has amnesia regarding her identity and past. The nurse should avoid flooding the client with information about past events, which can increase the client's level of anxiety. The nurse should encourage the client to make decisions regarding routine daily activities in order to promote improved self‑esteem and decrease the client's feelings of powerlessness. (ATI ch 12)

In the majority culture of the United States, which individual is at greatest risk to be incorrectly labeled mentally ill? a. Person who is usually pessimistic but strives to meet personal goals b. Wealthy person who gives $20 bills to needy individuals in the community c. Person with an optimistic viewpoint about life and getting his or her own needs met d. Person who attends a charismatic church and describes hearing Gods voice

ANS: D Hearing voices is generally associated with mental illness; however, in charismatic religious groups, hearing the voice of God or a prophet is a desirable event. In this situation, cultural norms vary, making it more difficult to make an accurate DSM-5 diagnosis. The individuals described in the other options are less likely to be labeled as mentally ill.

In a staff meeting at an inpatient mental health facility for persons, the administrator announces that psychiatric technicians will now be supervised by the milieu director rather than by nurses. What is the nurse's best action? a. Confer with colleagues about their opinions regarding the proposed change. b. Volunteer to participate on a committee charged with defining job responsibilities of unlicensed assistive personnel. c. Ask the administrator to delay implementation of this change until the decision can be reviewed by an interdisciplinary team. d. Advise the administrator of regulations in the state nurse practice act regarding supervision of unlicensed assistive personnel.

ANS: D Institutional policies and practices do not absolve an individual nurse of responsibility to practice on the basis of professional standards of nursing care. State nurse practice acts specify that unlicensed assistive personnel (UAP) work under a nurse's supervision (chapter 6, page 69)

A person tells a nurse, I was the only survivor in a small plane crash, but three business associates died. I got anxious and depressed and saw a counselor three times a week for a month. We talked about my feelings related to being a survivor, and now Im fine, back to my old self. Which type of therapy was used? a. Milieu therapy b. Psychoanalysis c. Behavior modification d. Interpersonal therapy

ANS: D Interpersonal therapy returns the patient to the former level of functioning by helping the patient come to terms with the loss of friends and guilt over being a survivor. Milieu therapy refers to environmental therapy. Psychoanalysis calls for a long period of exploration of unconscious material. Behavior modification focuses on changing a behavior rather than helping the patient understand what is going on in his or her life.

The spouse of a patient diagnosed with schizophrenia says, "I don't understand why childhood experiences have anything to do with this disabling illness." Select the nurses response that will best help the spouse understand this condition. a. Psychological stress is actually at the root of most mental disorders. b. We now know that all mental illnesses are the result of genetic factors. c. It must be frustrating for you that your spouse is sick so much of the time. d. Although this disorder more likely has a biological rather than psychological origin, the support and involvement of caregivers is very important.

ANS: D Many of the most prevalent and disabling mental disorders have been found to have strong biological influences. Helping the spouse understand the importance of his or her role as a caregiver is also important. Empathy is important but does not increase the spouses level of knowledge about the cause of the patients condition. Not all mental illnesses are the result of genetic factors. Psychological stress is not at the root of most mental disorders.

A Puerto Rican-American patient uses dramatic body language when describing emotional discomfort. Which analysis most likely explains the patients behavior? The patient: a. likely has a histrionic personality disorder. b. believes dramatic body language is sexually appealing c. wishes to impress staff with the degree of emotional pain. d. belongs to a culture in which dramatic body language is the norm.

ANS: D Members of Hispanic-American subcultures tend to use high affect and dramatic body language as they

A patient tells a nurse, "I have psychiatric problems and am in and out of hospitals all the time. Not one of my friends or relatives has these problems." Select the nurses best response. a. Comparing yourself with others has no real advantages. b. Why do you blame yourself for having a psychiatric illness? c. Mental illness affects 50% of the adult population in any given year. d. It sounds like you are concerned that others don't experience the same challenges as you.

ANS: D Mental illness affects many people at various times in their lives. No class, culture, or creed is immune to the challenges of mental illness. The correct response also demonstrates the use of reflection, a therapeutic communication technique. It is not true that mental illness affects 50% of the population in any given year. Asking patients if they blame themselves is an example of probing.

Select the most appropriate label to complete this nursing diagnosis: ___________, related to feelings of shyness and poorly developed social skills as evidenced by watching television alone at home every evening. a. Deficient knowledge b. Ineffective coping c. Powerlessness d. Social isolation

ANS: D Nursing diagnoses are selected on the basis of the etiologic factors and assessment findings or evidence. In this instance, the evidence shows social isolation that is caused by shyness and poorly developed social skills.

A nurse interacts with a newly hospitalized patient. Select the nurses comment that applies the communication technique of offering self. a. I've also had traumatic life experiences. Maybe it would help if I told you about them. b. Why do you think you had so much difficulty adjusting to this change in your life? c. I hope you will feel better after getting accustomed to how this unit operates. d. I'd like to sit with you for a while to help you get comfortable talking to me.

ANS: D Offering self is a technique that should be used in the orientation phase of the nurse-patient relationship. Sitting with the patient, an example of offering self, helps build trust and conveys that the nurse cares about the patient. Two incorrect responses are ineffective and nontherapeutic. The other incorrect response is therapeutic but an example of offering hope.

A soldier returned home last year after deployment to a war zone. The soldiers spouse complains, "We were going to start a family but now he won't talk about it. He will not look at children. I wonder if we're going to make it as a couple." Select the nurses best response. a. Post-traumatic stress disorder often changes a persons sexual functioning. b. I encourage you to continue to participate in social activities where children are present. c. Have you talked with your spouse about these reactions? Sometimes we just need to confront behavior. d. Post-traumatic stress disorder often strains relationships. I will suggest some community resources for help and support.

ANS: D PSTD precipitates changes that often lead to divorce. Providing support to both the veteran and spouse is important. Confrontation will not be effective. Although providing information is important, ongoing support is more effective.

Which is an example of patient advocacy? A. The nurse provides empathic listening. B. The nurse combines competence and caring. C. The nurse allows the patient to defend his preferences to the nurse. D. The nurse respects the patient's decision to refuse treatment.

ANS: D Patient Advocacy can occur on many levels including direct patient care, pleading for a course of action, and supporting change in institutional, global, and legislative arenas. The following are examples of patient advocacy activities: Informed consent, including refusal of treatment. Respecting patient decisions, even those we disagree with. Protecting against threats to well-being. Being informed about best practices. Patients are afforded protection through: Privacy and confidentiality during participation in research, using standards and reviews, and taking action against questionable or impaired practice.

The relapse of a patient diagnosed with schizophrenia is related to medication noncompliance. The patient is hospitalized for 5 days, medication is restarted, and the patients thoughts are now more organized. The patients family members are upset and say, "It's too soon for discharge. Hospitalization is needed for at least a month." The nurse should: a. call the psychiatrist to come explain the discharge rationale. b. explain that health insurance will not pay for a longer stay for the patient. c. call security to handle the disturbance and escort the family off the unit. d. explain that the patient will continue to improve if medication is taken regularly.

ANS: D Patients no longer stay in the hospital until every vestige of a symptom disappears. The nurse must assume responsibility to advocate for the patients right to the least restrictive setting as soon as the symptoms are under control and for the right of citizens to control health care costs. The health care provider will use the same rationale. Shifting blame will not change the discharge. Calling security is unnecessary. The nurse can handle this matter.

Which assessment finding for a patient living in the community requires priority intervention by the nurse? The patient: a. receives Social Security disability income plus a small check from a trust fund. b. lives in an apartment with two patients who attend day hospital programs. c. has a sibling who is interested and active in care planning. d. purchases and uses marijuana on a frequent basis.

ANS: D Patients who regularly buy illegal substances often become medication noncompliant. Medication noncompliance, along with the disorganizing influence of illegal drugs on cellular brain function, promotes relapse. The remaining options do not suggest problems.

An adult experienced a spinal cord injury resulting in quadriplegia 3 years ago and now lives permanently in a skilled care facility. Which comment by this person best demonstrates resiliency? a. "I often pray or a miracle that will heal my paralysis so I will be whole again." b. "I don't know what I did to deserve this fate or whether I am tough enough to endure it." c. "My accident was a twist of fate. I suppose there are worse things than being paralyzed." d/ "Being paralyzed has taken things from me but it hasn't kept me from being mentally involved in life."

ANS: D Resiliency is the ability to recover from or adjust success fully to trauma or change. A successful transition through a crisis builds resiliency or the next difficult trial. In the correct response, the person demonstrates acceptance of the paralysis and a focus on his or her abilities and assets (chapter 2, pg 12)

A nurse in an acute mental health facility is communicating with a client. The client states," I can't sleep. I stay up all night." The nurse responds, "You are having difficulty sleeping?" which of the following therapeutic communication technqiues is the nurse demonstrating? a. Offering general leads b. Summarizing c. Focusing d. Restating

ANS: D Restating allows the nurse to repeat the main idea expressed.Offering general leads allows the nurse to take the direction of the discussion. Summarizing enables the nurse to bring together important points of discussion to enhance understanding. Focusing concentrates the attention on one single point. (ATI ch 3)

A nurse working in an acute care unit or adolescents diagnosed with mental illness says, "Our patients have so much energy. We need some physical activities or them." In recognition of needs for safety and exercise, which activity could the treatment team approve? a. Badminton tournament b. Competitive soccer matches c. Intramural basketball games d. Line dancing to popular music

ANS: D Safety is a key consideration in selection of activities. The correct response identifies an activity likely to appeal to the population but without physical contact between patients or equipment, which may be associated with injury (chapter 5, Page 56)

A colleague tells the nurse, "I have not been able to sleep for the past three days. I feel like a robot." What is the nurse's best action? a. Direct the colleague to leave the facility immediately. b. Observe the colleague closely for evidence of impaired practice. c. Offer to administer medications to patients assigned to the colleague. d. Confer with the supervisor about the nurse's ability to safely deliver care.

ANS: D Sleep deprivation causes impaired practice, which jeopardizes patient safety. The colleague's comments indicate that impairment is likely. The nurse should confer with the supervisor to determine the appropriate action (chapter 6, page 70)

A patients nursing diagnosis is Insomnia. The desired outcome is: Patient will sleep for a minimum of 5 hours nightly by October 31. On November 1, a review of the sleep data shows the patient sleeps an average of 4 hours nightly and takes a 2-hour afternoon nap. What is the nurses next action? a. Continue the current plan without changes. b. Remove this nursing diagnosis from the plan of care. c. Write a new nursing diagnosis that better reflects the problem. d. Revise the outcome target date and interventions.

ANS: D Sleeping a total of 5 hours at night remains a reasonable outcome. Extending the time frame for attaining the outcome is appropriate. Examining interventions might result in planning an activity during the afternoon rather than permitting a nap. Continuing the current plan without changes is inappropriate. At the very least, the time in which the outcome is to be attained must be extended. Removing this nursing diagnosis from the plan of care could be used when the outcome goal has been met and the problem resolved. Writing a new nursing diagnosis is inappropriate because no other nursing diagnosis relates to the problem.

Which patient is a least likely candidate for telehealth/teletherapy? A. A depressed mother in an underserved area B. A soldier in Afghanistan C. An adolescent in a rural family D. A violent man who is threatening to harm his family

ANS: D Telehealth offers distinct advantages to isolated individuals and is making a difference in reaching untreated patients with mental health problems. However, a disadvantage for patients who may need physical intervention is that it is, of course, long distance care (from ch 8 ppt).

A nurse at a behavioral health clinic sees an unfamiliar psychiatric diagnosis on a patients insurance form. Which resource should the nurse consult to discern the criteria used to establish this diagnosis? a. A psychiatric nursing textbook b. NANDA International (NANDA-I ) c. A behavioral health reference manual d. Diagnostic and Statistical Manual of Mental Disorders (DSM-5)

ANS: D The DSM-5 gives the criteria used to diagnose each mental disorder. The NANDA-I focuses on nursing diagnoses. A psychiatric nursing textbook or behavioral health reference manual may not contain diagnostic criteria.

A nurse wants to find a description of diagnostic criteria for a person diagnosed with schizophrenia. Which resource should the nurse consult? a. U.S. Department of Health and Human Services b. Journal of the American Psychiatric Association c. North American Nursing Diagnosis Association International (NANDA-I) d. Diagnostic and Statistical Manual of Mental Disorders (DSM-5)

ANS: D The DSM-5 identifies diagnostic criteria for psychiatric diagnoses. The other sources have useful information but are not the best resources for finding a description of the diagnostic criteria for a psychiatric disorder.

A community psychiatric nurse facilitates medication compliance for a patient by having the health care provider prescribe depot medications by injection every 3 weeks at the clinic. For this plan to be successful, which factor will be of critical importance? a. Attitude of significant others toward the patient b. Nutritional services in the patients neighborhood c. Level of trust between the patient and the nurse d. Availability of transportation to the clinic

ANS: D The ability of the patient to get to the clinic is of paramount importance to the success of the plan. The depot medication relieves the patient of the necessity to take medication daily, but if he or she does not receive the injection at 3-week intervals, noncompliance will again be the issue. Attitude toward the patient, trusting relationships, and nutrition are important but not fundamental to this particular problem.

A Filipino-American patient had this nursing diagnosis: Situational low self-esteem, related to poor social skills as evidenced by lack of eye contact. Interventions were used to raise the patients self-esteem; however, after 3 weeks, the patients eye contact did not improve. What is the most accurate analysis of this scenario? a. The patients eye contact should have been directly addressed by role-playing to increase comfort with eye contact. b. The nurse should not have independently embarked on assessment, diagnosis, and planning for this patient. c. The patients poor eye contact is indicative of anger and hostility that remain unaddressed. d. The nurse should have assessed the patients culture before making this diagnosis and plan.

ANS: D The amount of eye contact in which a person engages is often culturally determined. In some cultures, eye contact is considered insolent, whereas in other cultures, eye contact is expected. Asian Americans, including persons from the Philippines, often prefer not to engage in direct eye contact.

A nurse is collecting an admission history for a client who has acute stress disorder (ASD). Which of the following information should the nurse expect to collect? A. The client remembers many details about the traumatic incident. B. The client expresses heightened elation about what is happening. C. The client states he first noticed manifestations of the disorder 6 weeks after the traumatic incident occurred. D. The client expresses a sense of unreality about the traumatic incident.

ANS: D The client who has ASD often expresses dissociative manifestations regarding the event, which includes a sense of unreality.The client who has ASD tends to be unable to remember details about the incident and can block the entire incident from memory. The client who has ASD reacts to what is happening with negative emotions such as anger, guilt, depression, and anxiety. Elation is an emotion that can occur in clients who have mania. Manifestations of ASD occur immediately to a few days following the event. (ATI ch 12)

The nurse prepares outcomes to the plan of care for an adult diagnosed with mental illness. Which strategy recognizes the current focus of treatment services or this population? a. The patient's diagnoses are confirmed using advanced neuro-imaging techniques. b. The nurse confers with the treatment team to verify the patient's most significant disability. c. The nurse prioritizes the patient's problems in accordance with Maslow's hierarchy of needs. d. The patient and family participate actively in establishing priorities and selecting interventions.

ANS: D The correct response recognizes the recovery model, which has the following tenets: Mental health care is consumer and family driven, with patients being partners in all aspects of care; care must focus on increasing the consumer's success in coping with life's challenges and building resilience; and an individualized care plan is at the core of consumer-centered recovery (chapter 1, pg 5)

A 26-month-old child displays negative behavior, refuses toilet training, and often shouts, "No!" when given direction. The nurse's counseling with the parent should be based on the premise that the child is engaged in which of Eriksons psychosocial crises? a. Trust versus Mistrust b. Initiative versus Guilt c. Industry versus Inferiority d. Autonomy versus Shame and Doubt

ANS: D The crisis of Autonomy versus Shame and Doubt is related to the developmental task of gaining control of self and environment, as exemplified by toilet training. This psychosocial crisis occurs during the period of early childhood. Trust versus Mistrust is the crisis of the infant, Initiative versus Guilt is the crisis of the preschool and early school-aged child, and Industry versus Inferiority is the crisis of the 6- to 12-year-old child.

During the first interview, a nurse notices that the patient does not make eye contact. The nurse can correctly analyze that: a. the patient is not truthful. b. the patient is feeling sad. c. the patient has a poor self-concept. d. more information is needed to draw a conclusion.

ANS: D The data are insufficient to draw a conclusion. The nurse must continue to assess.

A nurse documents: Patient is mute, despite repeated efforts to elicit speech. Makes no eye contact. Is inattentive to staff. Gazes off to the side or looks upward rather than at the speaker. Which nursing diagnosis should be considered? a. Defensive coping b. Decisional conflict c. Risk for other-directed violence d. Impaired verbal communication

ANS: D The defining characteristics are more related to the nursing diagnosis of Impaired verbal communication than to the other nursing diagnoses.

Which documentation of a patients behavior best demonstrates a nurses observations? a. Isolates self from others. Frequently fell asleep during group. Vital signs stable. b. Calmer and more cooperative. Participated actively in group. No evidence of psychotic thinking. c. Appeared to hallucinate. Patient frequently increased volume on television, causing conflict with others d. Wears four layers of clothing. States, I need protection from dangerous bacteria trying to penetrate my skin.

ANS: D The documentation states specific observations of the patients appearance and the exact statements made. The other options are vague or subjective statements and can be interpreted in different ways.

A nurse cares for an older adult patient admitted for treatment of depression. The health care provider prescribes an antidepressant medication, but the dose is more than the usual adult dose. The nurse should: a. implement the order. b. consult a drug reference. c. give the usual geriatric dosage. d. hold the medication and consult the health care provider.

ANS: D The dose of an antidepressant medication for older adult patients is often less than the usual adult dose. The nurse should withhold the medication and consult the health care provider who wrote the order. The nurses duty is to intervene and protect the patient. Consulting a drug reference is unnecessary because the nurse already knows the dose is excessive. Implementing the order is negligent. Giving the usual geriatric dose would be wrong; a nurse without prescriptive privileges cannot change the dose.

The spouse of a patient who has delusions asks the nurse, Are there any circumstances under which the treatment team is justified in violating the patients right to confidentiality? The nurse must reply that a. under no circumstances. b. at the discretion of the psychiatrist. c. when questions are asked by law enforcement. d. if the patient threatens the life of another person.

ANS: D The duty to warn a person whose life has been threatened by a patient under psychiatric treatment overrides the patients right to confidentiality. The right to confidentiality is not suspended at the discretion of the therapist or for legal investigations.

A woman says, "I cant' take anymore! Last year my husband had an affair and now we don't communicate. Three months ago, I found a lump in my breast. Yesterday my daughter said she's quitting college and moving in with her boyfriend." Which issue should be the focus for crisis intervention? a. Possible mastectomy b. Disordered family communication c. Effects of the husbands infidelity d. Coping with the reaction to the daughters events

ANS: D The focus of crisis intervention is on the most recent problem the straw that broke the camel's back. The patient has coped with the breast lesion, the husbands infidelity, and the disordered communication. Disequilibrium occurs only with the introduction of the daughter leaving college and moving.

A patients history shows intense and unstable relationships with others. The patient initially idealizes an individual and then devalues the person when the patients needs are not met. Which aspect of mental health is a problem? a. Effectiveness in work b. Communication skills c. Productive activities d. Fulfilling relationships

ANS: D The information provided centers on relationships with others, which are described as intense and unstable. The relationships of mentally healthy individuals are stable, satisfying, and socially integrated. Data are not present to describe work effectiveness, communication skills, or activities.

A critical care nurse asks a psychiatric nurse about the difference between a diagnosis in the Diagnostic and Statistical Manual of Mental Disorders (DSM-5) and a nursing diagnosis. Select the psychiatric nurses best response. a. No functional difference exists between the two diagnoses. Both serve to identify a human deviance. b. The DSM-5 diagnosis disregards culture, whereas the nursing diagnosis includes cultural variables. c. The DSM-5 diagnosis profiles present distress or disability, whereas a nursing diagnosis considers past and present responses to actual mental health problems. d. The DSM-5 diagnosis influences the medical treatment; the nursing diagnosis offers a framework to identify interventions for problems a patient has or may experience.

ANS: D The medical diagnosis, defined according to the DSM-5, is concerned with the patients disease state, causes, and cures, whereas the nursing diagnosis focuses on the patients response to stress and possible caring interventions. Both the DSM-5 and a nursing diagnosis consider culture. Nursing diagnoses also consider potential problems.

A nurse caring for a withdrawn, suspicious patient recognizes the development of feelings of anger toward the patient. The nurse should: a. suppress the angry feelings. b. express the anger openly and directly with the patient. c. tell the nurse manager to assign the patient to another nurse. d. discuss the anger with a clinician during a supervisory session.

ANS: D The nurse is accountable for the relationship. Objectivity is threatened by strong positive or negative feelings toward a patient. Supervision is necessary to work through a countertransference of feelings.

In a team meeting a nurse says, Im concerned whether we are behaving ethically by using restraint to prevent one patient from self-mutilation while the care plan for another patient who has also self-mutilated calls for one-on-one supervision. Which ethical principle most clearly applies to this situation? a. Beneficence b. Autonomy c. Fidelity d. Justice

ANS: D The nurse is concerned about justice, that is, the fair treatment with the least restrictive methods for both patients. Beneficence means promoting the good of others. Autonomy is the right to make ones own decisions. Fidelity is the observance of loyalty and commitment to the patient.

A patient states, Im not worth anything. I have negative thoughts about myself. I feel anxious and shaky all the time. Sometimes I feel so sad that I want to go to sleep and never wake up. Which nursing intervention should have the highest priority? a. Self-esteembuilding activities b. Anxiety self-control measures c. Sleep enhancement activities d. Suicide precautions

ANS: D The nurse should place priority on monitoring and reinforcing suicide self-restraint because it relates directly and immediately to patient safety. Patient safety is always a priority concern. The nurse should monitor and reinforce all patient attempts to control anxiety, improve sleep patterns, and develop self-esteem while giving priority attention to suicide self-restraint.

Select the best outcome for a patient with this nursing diagnosis: Impaired social interaction, related to sociocultural dissonance as evidenced by stating, "Although I'd like to, I don't join in because I don't speak the language very well." The patient will: a. demonstrate improved social skills. b. express a desire to interact with others. c. become more independent in decision making. d. select and participate in one group activity per day.

ANS: D The outcome describes social involvement on the part of the patient. Neither cooperation nor independence has been an issue. The patient has already expressed a desire to interact with others. Outcomes must be measurable. Two of the distractors are not measurable.

A patient is brought to the emergency department after a motorcycle accident. The patient is alert, responsive, and diagnosed with a broken leg. The patients vital signs are temperature (T), 98.6 F; pulse (P), 72 beats per minute (bpm); and respirations (R), 16 breaths per minute. After being informed that surgery is required for the broken leg, which vital sign readings would be expected? a. T, 98.6; P, 64; R, 14 b. T, 98.6; P, 68; R, 12 c. T, 98.6; P, 62; R, 16 d. T, 98.6; P, 84; R, 22

ANS: D The patient would experience stress associated with the anticipation of surgery. In times of stress, the sympathetic nervous system takes over (fight-or-flight response) and sends signals to the adrenal glands, thereby releasing norepinephrine. The circulating norepinephrine increases the heart rate. Respirations increase, bringing more oxygen to the lungs.

A participant at a community education conference asks, "What is the most prevalent type of mental disorder in the United States?" Select the nurses best response. a. Why do you ask? b. Schizophrenia c. Affective disorders d. Anxiety disorders

ANS: D The prevalence for schizophrenia is 1.1% per year. The prevalence of all affective disorders (e.g., depression, dysthymic disorder, bipolar) is 9.5%. The prevalence of anxiety disorders is 13.3%.

An example of a breach of a patients right to privacy occurs when a nurse: a. asks a family to share information about a patients prehospitalization behavior. b. discusses the patients history with other staff members during care planning. c. documents the patients daily behaviors during hospitalization. d. releases information to the patients employer without consent.

ANS: D The release of information without patient authorization violates the patients right to privacy. The other options are acceptable nursing practices.

Which scenario is an example of an adventitious crisis? a. Death of a child from sudden infant death syndrome b. Being fired from a job because of company downsizing c. Retirement of a 55-year-old d. Riot at a rock concert

ANS: D The rock concert riot is unplanned, accidental, violent, and not a part of everyday life. The incorrect options are examples of situational or maturational crises.

An adult required a heart transplant 5 years ago. Multiple medical complications followed, resulting in persistent irritability, depression, and insomnia. The adult's spouse says, "I've walked on eggshells for five years, never knowing when something else will go wrong." What is the nurse's priority intervention regarding the spouse? a. Explore the spouse's feelings, showing care and compassion. b. Encourage the spouse to attend a community support group. c. Teach stress reduction and relaxation techniques to the spouse. d. Refer the spouse to the primary care provider for health assessment.

ANS: D The scenario suggests that the spouse has experienced the effects of long-term stress. When stress is prolonged, the body stays alert. Chemicals produced by the stress response can have damaging effects on the body, causing physical diseases. While all of the actions may be indicated, obtaining a health assessment from the primary care provider has the first priority. (chapter 10, pg Page 122 (Figure 10-2))

Which statement made by a patient during an initial assessment interview should serve as the priority focus for the plan of care? a. I can always trust my family. b. It seems like I always have bad luck. c. You never know who will turn against you. d. I hear evil voices that tell me to do bad things.

ANS: D The statement regarding evil voices tells the nurse that the patient is experiencing auditory hallucinations. The other statements are vague and do not clearly identify the patients chief symptom.

A patient says to the nurse, "I dreamed I was stoned. When I woke up, I felt emotionally drained, as though I hadn't rested well." Which comment would be appropriate if the nurse seeks clarification? a. It sounds as though you were uncomfortable with the content of your dream. b. I understand what youre saying. Bad dreams leave me feeling tired, too. c. So, all in all, you feel as though you had a rather poor nights sleep? d. Can you give me an example of what you mean by stoned?

ANS: D The technique of clarification is therapeutic and helps the nurse examine the meaning of the patients statement. Asking for a definition of stoned directly asks for clarification. Restating that the patient is uncomfortable with the dreams content is parroting, a nontherapeutic technique. The other responses fail to clarify the meaning of the patients comment.

Which remark by a patient indicates passage from the orientation phase to the working phase of a nurse-patient relationship? a. I don't have any problems. b. It is so difficult for me to talk about my problems. c. I don't know how talking about things twice a week can help. d. I want to find a way to deal with my anger without becoming violent.

ANS: D Thinking about a more constructive approach to dealing with anger indicates a readiness to make a behavioral change. Behavioral change is associated with the working phase of the relationship. Denial is often seen in the orientation phase. It is common early in the relationship, before rapport and trust are firmly established, for a patient to express difficulty in talking about problems. Stating skepticism about the effectiveness of the nurse- patient relationship is more typically a reaction during the orientation phase.

Which patient would a nurse refer to partial hospitalization? An individual who: a. spent yesterday in the 24-hour supervised crisis care center and continues to be actively suicidal. b. because of agoraphobia and panic episodes needs psychoeducation for relaxation therapy. c. has a therapeutic lithium level and reports regularly for blood tests and clinic follow-up. d. states, "I'm not sure I can avoid using alcohol when my spouse goes to work every morning."

ANS: D This patient could profit from the structure and supervision provided by spending the day at the partial hospitalization program. During the evening, at night, and on weekends, the spouse could assume supervision responsibilities. The patient who is actively suicidal needs inpatient hospitalization. The patient in need of psychoeducation can be referred to home care. The patient who reports regularly for blood tests and clinical follow-up can continue on the same plan.

A nurse is caring for a client who states, "I'm so stressed at work because of my coworker. I am expected to finish others' work because of their laziness!" When discussing effective communication, which of the following statements by the client to the coworker indicates client understanding? a. "You really should complete your own work. I don't think it's right to expect me to complete your responsibilities." b. "Why do you expect me to finish your work? You must realize I have my own responsibilities." c. "It is not fair to expect me to complete your work. If you continue, then I will report your behavior to our supervisor." d. "When I have to pick up extra work, I feel very overwhelmed. I need to focus on my own responsibilities."

ANS: D This response demonstrates assertive communication, which allows the client to state his feelings about the behavior and then promote a change.This statement is an example of disapproving/disagreeing, which can prompt a defensive reaction and is therefore nontherapeutic. This statement uses a "why" question, which implies criticism and can prompt a defensive reaction and is therefore nontherapeutic. This statement is aggressive and threatening, which can prompt a defensive reaction and is therefore nontherapeutic. (ATI ch 9)

A patient says, "People should be allowed to commit suicide without interference from others." A nurse replies, "You're wrong. Nothing is bad enough to justify death." What is the best analysis of this interchange? a. The patient is correct. b. The nurse is correct. c. Neither person is totally correct. d. Differing values are reflected in the two statements.

ANS: D Values guide beliefs and actions. The individuals stating their positions place different values on life and autonomy. Nurses must be aware of their own values and be sensitive to the values of others.

During an interview, a patient attempts to shift the focus from self to the nurse by asking personal questions. The nurse should respond by saying: a. You've turned the tables on me. b. Nurses direct the interviews with patients. c. Do not ask questions about my personal life. d. The time we spend together is to discuss your concerns.

ANS: D When a patient tries to focus on the nurse, the nurse should refocus the discussion back onto the patient. Telling the patient that interview time should be used to discuss patient concerns refocuses discussion in a neutral way. Telling patients not to ask about the nurses personal life shows indignation. Saying that nurses prefer to direct the interview reflects superiority. Saying Youve turned the tables on me states the fact but does not refocus the interview.

A nurse is talking with a patient, and 5 minutes remain in the session. The patient has been silent for most of the session. Another patient comes to the door of the room, interrupts, and says to the nurse, "I really need to talk to you right now." The nurse should: a. say to the interrupting patient, "I am not available to talk with you at the present time." b. end the unproductive session with the current patient and spend time with the patient who has just interrupted. c. invite the interrupting patient to join in the session with the current patient. d. tell the patient who interrupted, "This session is 5 more minutes; then, I will talk with you."

ANS: D When a specific duration for a session has been set, the nurse must adhere to the schedule. Leaving the first patient would be equivalent to abandonment and would destroy any trust the patient had in the nurse. Adhering to the contract demonstrates that the nurse can be trusted and that the patient and the sessions are important. The incorrect responses preserve the nurse-patient relationship with the silent patient but may seem abrupt to the interrupting patient, abandon the silent patient, or fail to observe the contract with the silent patient.

At one point in an assessment interview a nurse asks, "How does your faith help you in stressful situations?" This question would be asked during the assessment of: a. childhood growth and development. b. substance use and abuse. c. educational background. d. coping strategies.

ANS: D When discussing coping strategies, the nurse might ask what the patient does when upset, what usually relieves stress, and to whom the patient goes to talk about problems. The question regarding whether the patients faith helps deal with stress fits well here. It would seem out of place if introduced during exploration of the other topics.

Legal and patients' rights are suspended when a client is hospitalized involuntarily. True or False?

ANS: False Involuntary admission requires that the client retain freedom from unreasonable bodily restraints, the right to informed consent, and the right to refuse medications, including psychotropic or antipsychotic medications. Other rights are preserved as well. (question is from the legal issues ppt)

The role of a psychiatric nurse on an inpatient unit would include which one of the following? A. Prescribing medication B. Maintaining a therapeutic milieu C. Analyzing patient behavior D. Providing psychotherapy

ANS: B The *therapeutic milieu* is essential to successful inpatient treatment. Milieu refers to the environment in which holistic treatment occurs and includes all members of the treatment team, a positive physical setting, interactions between those who are hospitalized, and activities that promote recovery (from ch 5 ppt)

A nurses neighbor asks, Why arent people with mental illness kept in state institutions anymore? What is the nurses best response? a. Many people are still in psychiatric institutions. Inpatient care is needed because many people who are mentally ill are violent. b. Less restrictive settings are now available to care for individuals with mental illness. c. Our nation has fewer persons with mental illness; therefore fewer hospital beds are needed. d. Psychiatric institutions are no longer popular as a consequence of negative stories in the press.

ANS: B The community is a less restrictive alternative than hospitals for the treatment of people with mental illness. The remaining options are incorrect and part of the stigma of mental illness.

As a patient diagnosed with mental illness is being discharged from a facility, a nurse invites the patient to the annual staff picnic. What is the best analysis of this scenario? a. The invitation facilitates dependency on the nurse. b. The nurses action blurs the boundaries of the therapeutic relationship. c. The invitation is therapeutic for the patients diversional activity deficit. d. The nurses action assists the patients integration into community living.

ANS: B The invitation creates a social relationship rather than a therapeutic relationship.

A patient with severe depression states, "God is punishing me for my past sins." What is the nurses best response? a. Why do you think that? b. You sound very upset about this. c. You believe God is punishing you for your sins? d. If you feel this way, you should talk to a member of your clergy.

ANS: B The nurse reflects on the patients comment, a therapeutic technique to encourage sharing for perceptions and feelings. The incorrect responses reflect probing, closed-ended comments, and giving advice, all of which are nontherapeutic.

A nurse uses Peplaus interpersonal therapy while working with an anxious, withdrawn patient. Interventions should focus on: a. changing the patients perceptions about self. b. improving the patients interactional skills. c. using medications to relieve anxiety. d. reinforcing specific behaviors.

ANS: B The nurse-patient relationship is structured to provide a model for adaptive interpersonal relationships that can be generalized to others. Changing the patients perceptions about his- or herself would be appropriate for cognitive therapy. Reinforcing specific behaviors would be used in behavioral therapy. Using medications is the focus of biological therapy.

During the first interview with a parent whose child died in a car accident, the nurse feels empathic and reaches out to take the patients hand. Select the correct analysis of the nurses behavior. a. It shows empathy and compassion. It will encourage the patient to continue to express feelings. b. The gesture is premature. The patients cultural and individual interpretation of touch is unknown. c. The patient will perceive the gesture as intrusive and overstepping boundaries. d. The action is inappropriate. Patients in a psychiatric setting should not be touched.

ANS: B Touch has various cultural and individual interpretations. Nurses should refrain from using touch until an assessment can be made regarding the way in which the patient will perceive touch. The other options present prematurely drawn conclusions.

The relationship between a nurse and patient as it relates to status and power is best described by which term? a. Symmetric b. Complementary c. Incongruent d. Paralinguistic

ANS: B When a difference in power exists, as between a student and teacher or between a nurse and patient, the relationship is said to be complementary. Symmetrical relationships exist between individuals of like or equal status. Incongruent and paralinguistic are not terms used to describe relationships.

In which scenario is it most urgent for the nurse to act as a patient advocate? a. An adult cries and experiences anxiety after a near-miss automobile accident on the way to work. b. A homeless adult diagnosed with schizophrenia lives in a community expecting a category 5 hurricane. c. A 14-year-old girl's grades decline because she consistently focuses on her appearance and social networking. d. A parent allows the prescription to lapse for 1 day for their 8-year-old child's medication for attention-deficit/hyperactivity disorder.

ANS: B While all of the scenarios present opportunities for a nurse to intervene, the correct response presents an imminent danger to the patient's safety and well-being. (chapter 1, pg 6-7)

A nurse is caring for a client who is in mechanical restraints. Which of the following statements should the nurse include in the documentation? (select all that apply) a. "Client ate most of their breakfast." b. "Client was offered 8 oz of water every hr." c. "Client shouted obscenities at assistive personnel." d. "Client received chlorpromazine 15 mg by mouth at 1000." e. "Client acted out after lunch."

ANS: B, C, D Document objective information regarding intake in the client's medical record ("the client ate 70% of their breakfast.). The amount and frequency of fluids offered, description of the client's verbal communication, and the dosage and time of medication administration are all objective data that should be documented when caring for a client in mechanical restraints. (ATI ch 2)

A nurse is discussing acute vs. prolonged stress with a client. Which of the following effects should the nurse identify as an acute stress response? a. Chronic pain b. Depressed immune system c. Increased blood pressure d. Panic attack Unhappiness

ANS: B, C, E A depressed immune system, increased blood pressure, and unhappiness are indicators of acute stress. Chronic pain and panic attacks indicate a prolonged or maladaptive stress response. (ATI ch 9)

Which is the moral component of personality structure, according to Freud? A.The Id B.The Ego C.The Superego D.The Unconscious

ANS: C

A nurse is working with an established group and identifies various member roles. Which of the following should the nurse identify as an individual role? a. A member who praises input from other members b. A member who follows the direction of other members c. A member who brags about accomplishments d. A member who evaluates the group's performance

ANS: C (a) & (b) Maintenance roles. (d) task role (from ATI ch 8)

A nurse wants to use democratic leadership with a group whose purpose is to learn appropriate conflict resolution techniques. The nurse is correct in implementing this form of group leadership when demonstrating with of the following actions? a. Observes group techniques without interfering with the group process b. Discusses a technique and then directs members of the group c. Asks for group suggestions of technique and then supports discussion d. Suggest techniques and asks group members to reflect on their use

ANS: C (a) is laissez-faire leadership. (b) and (d) are autocratic leadership (from ATI ch 8)

A nurse is assisting with systematic desensitization for a client who has an extreme fear of elevators. Which of the following actions should the nurse implement with this form of therapy? a. Demonstrate riding in the elevator, and then ask the client to imitate the behavior. b. Advise the client to say "stop" out loud every time they begin to feel an anxiety response related to an elevator. c. Gradually expose the client to an elevator while practicing relaxation techniques. d. Stay with the client in the elevator until the anxiety response diminishes.

ANS: C (a) is modeling. (b) is thought stopping. (d) is flooding (from ATI ch 7)

A nurse on an acute mental health unit forms a group to focus on self-management of medication. At each of the meetings, two of the members conspire together to exclude the rest of the group. This is an example of which of the following concepts? a. Triangulation b. Group process c. Subgroup d. Hidden agenda

ANS: C A subgroup is a small number of people within a larger group who function separately from that group. Triangulation is if a third party is drawn into the relationship with two members whose relationship is unstable. This is not an example of group process. If this subgroup conspire to not only exclude the rest of the group but also have a stated, separate goal from the rest of the group, this would be hidden agenda. (from ATI ch 8)

A suspicious and socially isolated patient lives alone, eats one meal a day at a nearby shelter, and spends the remaining daily food allowance on cigarettes. Select the community psychiatric nurses best initial action. a. Report the situation to the manager of the shelter.b. Tell the patient, You must stop smoking to save money.c. Assess the patients weight; determine the foods and amounts eaten.d. Seek hospitalization for the patient while a new plan is being formulated.

ANS: C Assessment of biopsychosocial needs and general ability to live in the community is called for before any action is taken. Both nutritional status and income adequacy are critical assessment parameters. A patient may be able to maintain adequate nutrition while eating only one meal a day. Nurses assess before taking action. Hospitalization may not be necessary.

Which individual with mental illness may need involuntary hospitalization? a. A person with alcoholism who has been sober for 6 months but begins drinking again b. An individual with schizophrenia who stops taking prescribed antipsychotic drugs c. An individual with bipolar disorder, manic phase, who has not eaten in 4 days d. Someone who repeatedly phones a national TV broadcasting service with news tips

ANS: C Because the individual has not eaten in 4 days, this may indicate that the individual poses a threat to themself (question is from the legal issues ppt)

The nurse admits a patient experiencing hallucinations and delu- sional thinking to an inpatient mental health unit. The plan of care will require which service occurs first? a. Social history b. Psychiatric history c. Medical assessment d. Psychological evaluation

ANS: C Begins with a medical assessment to rule out or consider co-occurring/comorbid conditions (chapter 5, Page 56)

Which theory is most likely to help the nurse respond appropriately to the needs of an aging patient experiencing frustration after a stroke? A. Behavioral B. Cognitive C. Psychosocial development D. Hierarchy of needs

ANS: C Behavioral: Promoting adaptive behaviors through reinforcement Cognitive: Helping patients identify negative thought patterns *Psychosocial development*: Providing structure for understanding critical junctures in development Hierarchy of needs: Prioritizing nursing care

A nurse is orienting a new client to a mental health unit. When explaining the unit's community meetings, which of the following statements should the nurse make? a. "You and a group of other clients will meet to discuss your treatment plans." b. "Community meeting have a specific agenda that is established by staff." c. "You and the other clients will meet with staff to discuss common problems." d. "Community meetings are an excellent opportunity to explore your personal mental health issues."

ANS: C Community meetings are an opportunity for clients to discuss common problems or issues affecting all members of the unit.Individual treatment plans are discussed during individual therapy rather than a community meeting. Community meetings may be structured so that they are client-led with decisions made by the group as a whole. Personal mental health issues are discussed during individual therapy rather than a community meeting. (ATI ch 5)

Eileen works with youth in a residential treatment center. When Amber, a new patient of hers, does not want her parents to visit, Eileen says, "I know how you feel. Did your dad molest you too?" Eileen's reaction is an example of which behavior? A. Empathy B. Verbal tracking C. Countertransference D. Positive reinforcement

ANS: C Countertransference: Tendency of the nurse to displace feelings related to people in his or her past onto a patient (from ch 8 ppt).

Which scenario meets the criteria for "normal" behavior? a. An 8-year-old child's only verbalization is "No no no." b. A 16-year-old girl usually sleeps or 3 or 4 hours per night. c. A 43-year-old man cries privately or 1 month after the death of his wive. d. A 64-year-old woman has difficulty remembering the names of her grandchildren.

ANS: C Many biological, cultural, and environmental factors influence mental health. Persons who are normal also may experience dysfunction during their lives. The death of a spouse is a difficult experience, so crying is expected (chapter 2, pg 10)

The parent of a child diagnosed with schizophrenia tearfully asks a nurse, What could I have done differently to prevent this illness? Select the nurses most caring response. a. Although schizophrenia is caused by impaired family relationships, try not to feel guilty. No one can predict how a child will respond to parental guidance. b. Most of the damage is done, but there is still hope. Changing your parenting style can help your child learn to cope more effectively with the environment. c. Schizophrenia is a biological illness with similarities to diabetes and heart disease. You are not to blame for your childs illness. d. Most mental illnesses result from genetic inheritance. Your genes are more at fault than your parenting.

ANS: C Patients and families need reassurance that the major mental disorders are biological in origin and are not the fault of parents. Knowing the biological nature of the disorder relieves feelings of guilt over being responsible for the illness. The incorrect responses are neither wholly accurate nor reassuring; they fall short of being reassuring and place the burden of having faulty genes on the shoulders of the parents.

A soldier who served in a combat zone returned to the United States. The soldiers spouse complains to the nurse, "We had planned to start a family, but now he won't talk about it. He won't even look at children." The spouse is describing which symptom associated with post-traumatic stress disorder (PTSD)? a. Reexperiencing b. Hyperarousal c. Avoidance d. Psychosis

ANS: C Physiologic reactions to reminders of the event include a persistent avoidance of the stimuli associated with the trauma; the individual avoids talking about the event or avoids activities, people, or places that arouse memories of the trauma. Avoidance is exemplified by a sense of foreshortened future and estrangement. No evidence suggests that this soldier is having a hyperarousal reaction or is re-experiencing war-related traumas. Psychosis is not evident.

Which agency provides coordination in the event of a terrorist attack? a. U.S. Food and Drug Administration (FDA) b. Environmental Protection Agency (EPA) c. National Incident Management System (NIMS) d. Federal Emergency Management Agency (FEMA)

ANS: C The NIMS provides a systematic approach to guide departments and agencies at all levels of government, nongovernmental organizations, and the private sector during disaster situations.

An experienced nurse in a major medical center requests a transfer from a general medical unit to an acute care psychiatric unit. Which organizational feature would best support this nurse's successful transition? a. Assignment to medication administration or the first 6 months b. Working with a seasoned mental health technician for the first month c. Co-assignment with a knowledgeable psychiatric nurse or an extended orientation d. Staff development activities focused on developing therapeutic communication skills

ANS: C The nurse's skills rom the medical unit will be valuable, but this nurse will need to expand his or her skill set to effectively care for a psychiatric population. Working with an experienced psychiatric nurse will provide opportunities or learning (chapter 5, Page 59)

At the last contracted visit in the crisis intervention clinic, an adult says, "I've emerged from this a stronger person. You helped me feel like my life is back in balance." The nurse responds, "I think it would be worthwhile to have two more sessions to explore why your reactions were so intense." Which analysis applies? a. The patient is experiencing transference. b. The patient demonstrates a need for continuing support. c. The nurse is having difficulty terminating the relationship. d. The nurse is empathizing with the patients feelings of dependency.

ANS: C The nurses remark is clearly an invitation to work on other problems and prolong contact with the patient. The focus of crisis intervention is on the problem that precipitated the crisis, not other issues. The scenario does not describe transference. The patients need for continuing support is not demonstrated in the scenario. The scenario does not describe dependency needs.

What information is conveyed by nursing diagnoses? Select all that apply. a. Medical judgments about the disorder b. Goals and outcomes for the plan of care c. Unmet patient needs currently present d. Supporting data that validate the diagnoses e. Probable causes that will be targets for nursing interventions

ANS: C, D, E Nursing diagnoses focus on phenomena of concern to nurses rather than on medical diagnoses. Goals and outcomes are part of the planning phase.

Which statements most clearly reflect the stigma of mental illness? Select all that apply. a. Many mental illnesses are hereditary. b. Mental illness can be evidence of a brain disorder. c. People claim mental illness so they can get disability checks. d. If people with mental illness went to church, they would be fine. e. Mental illness is a result of the breakdown of the American family.

ANS: C, D, E Stigma is represented by judgmental remarks that discount the reality and validity of mental illness. Many mental illnesses are genetically transmitted. Neuroimaging can show changes associated with some mental illnesses.

A nurse assesses the health status of soldiers returning from Afghanistan. Screening will be a priority for signs and symptoms of which health problems? Select all that apply. a. Schizophrenia b. Eating disorder c. Traumatic brain injury d. Seasonal affective disorder e. Post-traumatic stress disorder

ANS: C, E TBI and PTSD each occur in approximately 20% of soldiers returning from Afghanistan. Some soldiers have both problems. The incidence of disorders identified in the distractors would be expected to parallel the general population.

Private insurance typically covers which one of the following? A. All illnesses regardless of duration B. All illnesses except mental illness C. All mental illnesses over a lifetime D. Mental illness with a lifetime cap

ANS: D (from ch 5 ppt, no rationale provided)

A nurse is talking with the caregiver of a child who has demonstrated recent changes in behavior and mood. When the caregiver of the child asks the nurse for reassurance about their child's condition, which of the following responses should the nurse make? a. "I think your child is getting better. What have you noticed?" b. "I'm sure everything will be okay. It just takes time to heal." c. "I'm not sure what's wrong. Have you asked the doctor about your concerns?" d. "I understand you're concerned. Let's discuss what concerns your specifically."

ANS: D "I understand you're concerned. Let's discuss what concerns you specifically." reflects upon, and accepts, the parents' feelings, and it allows them to clarify what they are feeling. It is a therapeutic response. The rest are nontherapeutic responses. "I think your son is getting better. What have you noticed?" interjects the nurse's opinion and can cause the parents to withhold their thoughts and feelings. "I'm sure everything will be okay. It just takes time to heal." interjects the nurse's opinion and provides false reassurance which can cause the parents to withhold their thoughts and feelings. "I'm not sure what's wrong. Have you asked the doctor about your concerns?" avoids addressing the parent's concerns directly and indicates disinterest by the nurse for wanting to discuss the concerns with the parents. (ATI ch 3)

Cognitive therapy was provided for a patient who frequently said, "I'm stupid." Which statement by the patient indicates the therapy was effective? a. I'm disappointed in my lack of ability. b. I always fail when I try new things. c. Things always go wrong for me. d. Sometimes I do stupid things.

ANS: D "I'm stupid" is a cognitive distortion or irrational thought. A more rational thought is, "Sometimes I do stupid things." The latter thinking promotes emotional self-control. The incorrect options reflect irrational thinking.

Supplemental Security Income (SSI) benefits typically cover needs based on which one of the following? A. Mental illness B. Unemployment C. Homelessness D. Economic need

ANS: D (from ch 5 ppt, no rationale provided)

A patient comes to the clinic with superficial cuts on the left wrist. The patient is pacing and sobbing. After a few minutes with the nurse, the patient is calmer. What should the nurse ask to determine the patients perception of the precipitating event? a. Tell me why you were crying. b. How did your wrist get injured? c. How can I help you feel more comfortable? d. What was happening just before you started feeling this way?

ANS: D A clear definition of the immediate problem provides the best opportunity to find a solution. Asking about recent upsetting events permits the assessment of the precipitating event. Asking why questions is a nontherapeutic communication technique.

In the shift-change report, an off-going nurse criticizes a patient who wears heavy makeup. Which comment by the nurse who receives the report best demonstrates advocacy? a. This is a psychiatric hospital. Craziness is what we are all about. b. Lets all show acceptance of this patient by wearing lots of makeup too. c. Your comments are inconsiderate and inappropriate. Keep the report objective. d. Our patients need our help to learn behaviors that will help them get along in society.

ANS: D Accepting patients needs for self-expression and seeking to teach skills that will contribute to their well-being demonstrate respect and are important parts of advocacy. The on- coming nurse needs to take action to ensure that others are not prejudiced against the patient. Humor can be appropriate within the privacy of a shift report but not at the expense of respect for patients. Judging the off-going nurse in a critical way will create conflict. Nurses must show compassion for each other.

A psychiatric nurse best implements the ethical principle of autonomy when he or she: a. intervenes when a self-mutilating patient attempts to harm self. b. stays with a patient who is demonstrating a high level of anxiety. c. suggests that two patients who are fighting be restricted to the unit. d. explores alternative solutions with a patient, who then makes a choice.

ANS: D Autonomy is the right to self-determination, that is, to make ones own decisions. When the nurse explores alternatives with the patient, the patient is better equipped to make an informed, autonomous decision. Staying with a highly anxious patient or intervening with a self-mutilating patient demonstrates beneficence and fidelity. Suggesting that two fighting patients be restricted to the unit demonstrates the principles of fidelity and justice.

Social stigma related to mental illness refers to which one of the following statements? A. Mental illness is a disease process. B. Mental illness is inherited. C. Mental illness is a complex problem. D. Mental illness is due to wrong thinking.

ANS: D Having a psychiatric illness is often stigmatized as being flawed in some way rather than being acknowledged as illness. (from ch 5 ppt)

A husband attending family therapy tells his wife, "I'm listening. I want to support you in this," but he glares out the window and taps his foot when she starts to cry and explain her feelings. Which is the best description of his response? A. Active listening B. Reflective listening C. Use of a patient filter D. Double-bind message

ANS: D He is sending a mixed message, which creates confusion by being a mix of content—what is said—and process—what is transmitted nonverbally—usually a hurtful or negative message (from ch 8 ppt)

A nurse assessing a new patient asks, "What is meant by the saying, You cant judge a book by its cover?" Which aspect of cognition is the nurse assessing? a. Mood b. Attention c. Orientation d. Abstraction

ANS: D Patient interpretation of proverbial statements gives assessment information regarding the patients ability to abstract, which is an aspect of cognition. Mood, orientation, and attention span are assessed in other ways.

A nurse psychotherapist works with an anxious, dependent patient. The therapeutic strategy most consistent with the framework of psychoanalytic psychotherapy is: a. emphasizing medication compliance. b. identifying the patients strengths and assets. c. offering psychoeducational materials and groups. d. focusing on feelings developed by the patient toward the nurse.

ANS: D Positive or negative feelings of the patient toward the nurse or therapist represent transference. Transference is a psychoanalytic concept that can be used to explore previously unresolved conflicts. Emphasizing medication compliance is more related to biological therapy. Identifying patient strengths and assets would be consistent with supportive psychotherapy. The use of psychoeducational materials is a common homework assignment used in cognitive therapy.

An adult seeks counseling after the spouse is murdered. The adult angrily says, "I hate the beast that did this. It has ruined my life. During the trial, I don't know what I'll do if the jury doesn't return a guilty verdict." What is the nurses highest priority question? a. Would you like to talk to a psychiatrist about some medication to help you cope during the trial? b. What resources do you need to help you cope with this situation? c. Do you have enough support from your family and friends? d. Are you having thoughts of hurting yourself or others?

ANS: D The highest nursing priority is safety. The nurse should assess suicidal and homicidal potentials. The incorrect options may be important but not the highest priority.

A nurse supports parental praise of a child who is behaving in a helpful way. When the individual behaves with politeness and helpfulness in adulthood, which feeling will most likely result? a. Guilt b. Anxiety c. Loneliness d. Self-esteem

ANS: D The individual will be living up to the ego ideal, which will result in positive feelings about self. The other options are incorrect; each represents a negative feeling.

A patient reports to a primary care provider about sleeplessness, constant fatigue, and sadness. In our current health care climate, what is the most likely treatment approach that will be offered to the patient? a. Group therapy b. Individual psychotherapy c. Complementary therapy d. Psycho-pharmacological treatment

ANS: D The patient's report suggests that depression is occurring. With the increased understanding of the biology of psychiatric illnesses, treatment approaches have evolved rapidly into more scientifically grounded methods, particularly psycho-pharmacology (chapter 1, pg 4)

Which of the following are reasons for the utilization of the DSM-5 in the mental health are system? Select all that apply. 1. Is is a convenient format for organizing and communicating clinical data. 2. Is if a means for considering the complexity of clinical situations 3. It is a means for describing the unique symptoms of psychiatric clients 4. It is a format for evaluating clients based on regulated approach. 5. It is a means to better understand the etiology of man psychiatric disorders

ANS: 1, 2 #1. The DSM-5 is a convenient format for organizing and communicating clinical data. It includes a list of psychiatric and medical conditions and facilitates a compressive and systematic evaluation #3. DSM-5 is a means for considering the complexity of clinical situations. It addresses behavioral and physical symptoms, long-term problems, stressors and functioning.

In the emergency department, the nurse assesses a client who is aggressive and experiencing auditory hallucinations. The client states, "The CIA is plotting to kill me." To which mental health setting would the nurse expect this client to be admitted? 1. Long-term, in-patient facility 2. Day treatment 3. Short-term, in-patient, locked unit 4. Psychiatric case management

ANS: 3 A short-term, in-patient, locked unit would be most appropriate for this client. This setting provides containment and structure for clients who are at risk for harming themselves or others

A nurse is caring for a client who has a new prescription for disulfiram for treatment of alcohol use disorder. The nurse informs the client that this medication can cause nausea and vomiting when alcohol is consumed. Which of the following types of treatment is this method an example? a. Aversion therapy b. Flooding c. Biofeedback d. Dialectical behavior therapy

ANS: A Aversion therapy pairs maladaptive behavior with punishment or unpleasant stimuli to promote a change in behavior. Flooding is exposing a client to a great deal of an undesirable stimulus in an attempt to turn off the anxiety response. Biofeedback is a behavioral therapy to control pain, tensions, and anxiety. Dialectical behavior therapy is a cognitive-behavioral therapy for clients with a personality disorder and exhibiting self-injurious behavior. (from ATI ch 7)

A soldier returns to the United States from active duty in a combat zone in Afghanistan. The soldier is diagnosed with post-traumatic stress disorder (PTSD). The nurses highest priority is to screen this soldier for which problem? a. Major depressive disorder b. Bipolar disorder c. Schizophrenia d. Dementia

ANS: A Major depressive disorder frequently co-occurs with PTSD. The incidence of the disorders identified in the distractors is similar to the general population.

A distraught 8-year-old girl tells the nurse, "I had a horrible nightmare and was so scared. I tried to get in bed with my parents but they said, 'No.' I think I could have gone back to sleep if I had been with them." Which family dynamic is likely the basis of this child's comment? a. Boundaries in the family are rigid. b. The family has poor differentiation of roles. c. The girl is enmeshed in part of a family triangle. d. Generational boundaries in the family are diffuse.

ANS: A Rigid or disengaged boundaries are those in which the rules and roles are followed despite the consequences (chapter 3, pg 31 (Box 3-2))

At what point in the nurse-patient relationship should a nurse plan to first address termination? a. In the orientation phase b. During the working phase c. In the termination phase d. When the patient initially brings up the topic

ANS: A The patient has a right to know the conditions of the nurse-patient relationship. If the relationship is to be time limited, then the patient should be informed of the number of sessions. If it is open ended, then the termination date will not be known at the outset and the patient will know that the issue will be negotiated at a later date. The nurse is responsible for bringing up the topic of termination early in the relationship, usually during the orientation phase.

A nurse says, "I am the only one who truly understands this patient. Other staff members are too critical." The nurses statement indicates: a. boundary blurring. b. sexual harassment. c. positive regard. d. advocacy.

ANS: A When the role of the nurse and the role of the patient shift, boundary blurring may arise. In this situation, the nurse is becoming overinvolved with the patient as a probable result of unrecognized countertransference. When boundary issues occur, the need for supervision exists. The situation does not describe sexual harassment. Data are not present to suggest positive regard or advocacy.

A nurse driving home after work comes upon a serious automobile accident. The driver gets out of the car with no apparent physical injuries. Which assessment findings would be expected from the driver immediately after this event? Select all that apply. a. Difficulty using a cell phone b. Long-term memory losses c. Fecal incontinence d. Rapid speech e. Trembling

ANS: A, D, E Immediate responses to crisis commonly include shock, numbness, denial, confusion, disorganization, difficulty with decision making, and physical symptoms such as nausea, vomiting, tremors, profuse sweating, and dizziness associated with anxiety. Incontinence and long-term memory losses would not be expected.

An adult comes to the crisis clinic after being terminated from a job of 15 years. The patient says, "I don't know what to do. How can I get another job? Who will pay the bills? How will I feed my family?" Which nursing diagnosis applies? a. Hopelessness b. Powerlessness c. Chronic low self-esteem d. Disturbed thought processes

ANS: B The patient describes feelings of the lack of control over life events. No direct mention is made of hopelessness or chronic low self-esteem. The patients thought processes are not shown to be altered at this point.

A nurse is planning group therapy for clients dealing with bereavement. Which of the following activities should the nurse include in the initial phase? (select all that apply) a. Encourage the group to work toward goals b. Define the purpose of the group c. Discuss termination of the group d. Identify informal roles of members within the group e. Establish an expectation of confidentiality within the group

ANS: B, C, E (a) and (d) are during the working phase (from ATI ch 8)

Which is most likely to be a precursor to developing agoraphobia? A.Social phobia B.Bipolar disorder C.Panic disorder D.Anxiety disorder

ANS: C Panic disorders typically develop in adolescence or early adulthood; about one in three people with a panic disorder develops agoraphobia. (See table 2-2 in the text)

Nursing behaviors associated with the implementation phase of the nursing process are concerned with: a. participating in the mutual identification of patient outcomes. b. gathering accurate and sufficient patient-centered data. c. comparing patient responses and expected outcomes. d. carrying out interventions and coordinating care.

ANS: D Nursing behaviors relating to implementation include using available resources, performing interventions, finding alternatives when necessary, and coordinating care with other team members.

A soldier served in combat zones in Iraq in 2010 and was deployed to Afghanistan in 2013. When is it most important for the nurse to screen for signs and symptoms of post-traumatic stress disorder (PTSD)? a. Immediately upon return to the United States from Afghanistan b. Before departing Afghanistan to return to the United States c. One year after returning from Afghanistan d. Screening should be ongoing

ANS: D PTSD can have a long lag timemonths to years. Screening should be ongoing.

An 86-year-old, previously healthy and independent, falls after an episode of vertigo. Which behavior by this patient best demonstrates resilience? The patient: a. says, "I knew this would happen eventually." b. stops attending her weekly water aerobics class. c. refuses to use a walker and says, I dont need that silly thing. d. says, "Maybe some physical therapy will help me with my balance."

ANS: D Resiliency is the ability to recover from or adjust to misfortune and change. The correct response indicates that the patient is hopeful and thinking positively about ways to adapt to the vertigo. Saying I knew this would happen eventually and discontinuing healthy activities suggest a hopeless perspective on the health change. Refusing to use a walker indicates denial.

After completing the contracted number of visits to the crisis clinic, an adult says, "I've emerged from this as a stronger person. You supported me while I worked through my feelings of loss and helped me find community resources. I'm benefiting from a support group." The nurse can evaluate the patients feelings about the care received as: a. not at all satisfied. b. somewhat satisfied. c. moderately satisfied. d. very satisfied.

ANS: D The patient mentions a number of indicators that suggest a high degree of satisfaction with the Nursing Outcomes Classification of patient satisfaction: psychological care. No indicators express low-to-moderate satisfaction.

A patient underwent psychotherapy weekly for 3 years. The therapist used free association, dream analysis, and facilitated transference to help the patient understand unconscious processes and foster personality changes. Which type of therapy was used? a. Short-term dynamic psychotherapy b. Transactional analysis c. Cognitive therapy d. Psychoanalysis

ANS: D The therapy described is traditional psychoanalysis. Short-term dynamic psychotherapy would last less than 1 year. Neither transactional analysis nor cognitive therapy makes use of the techniques described.

A nurse was the case manager for a client with serious mental illness for 6 years. The client died by suicide 1 week ago. Today, the client's spouse asks, "I always wondered if my spouse was a victim of sexual abuse in childhood. What can you tell me about that?" The nurse disclose information to the surviving spouse. True or false?

ANS: False A person's reputation can be damaged even after death. Therefore, it is important that nurses not divulge information after a person's death that could not be legally shared before the death (question is from the legal issues ppt)


Conjuntos de estudio relacionados

Fundamentals of Nursing Ch 2: Theory, Research, and Evidence-Based Practice

View Set

Previous Quiz Questions CSCE 221

View Set

Physics flashcards for exam 2- examples and steps

View Set